SlideShare a Scribd company logo
ĐẠI HỌC THÁI NGUYÊN
TRƢỜNG ĐẠI HỌC KHOA HỌC
-------------------------------
vTải tài liệu tại sividoc.com
Viết đề tài giá sinh viên – ZALO:0973.287.149-TEAMLUANVAN.COM
ĐỖ LAN HƢƠNG
ĐỊNH LÝ ZSIGMONDY VÀ
TÍNH CHẤT SỐ HỌC CỦA ĐA THỨC
LUẬN VĂN THẠC SĨ TOÁN HỌC
THÁI NGUYÊN - 2018
ĐẠI HỌC THÁI NGUYÊN
TRƢỜNG ĐẠI HỌC KHOA HỌC
-------------------------------
vTải tài liệu tại sividoc.com
Viết đề tài giá sinh viên – ZALO:0973.287.149-TEAMLUANVAN.COM
ĐỖ LAN HƢƠNG
ĐỊNH LÝ ZSIGMONDY VÀ
TÍNH CHẤT SỐ HỌC CỦA ĐA THỨC
Chuyên ngành: Phƣơng pháp Toán sơ cấp
Mã số 84 60 113
LUẬN VĂN THẠC SĨ TOÁN HỌC
NGƯỜI HƯỚNG DẪN KHOA HỌC
PGS.TS. ĐÀM VĂN NHỈ
THÁI NGUYÊN - 2018
1
Viết đề tài giá sinh viên – ZALO:0973.287.149-TEAMLUANVAN.COM
Mnc lnc
1 Định lj Zsigmondy 4
1.1 Đa thác và so phác . . . . . . . . . . . . . . . . . . . . . 4
1.1.1 Khái ni m đa thác, phép toán . . . . . . . . . . . 4
1.1.2 Thu t toán Euclid . . . . . . . . . . . . . . . . . 5
1.1.3 Xây dựng trường so phác C . . . . . . . . . . . . 6
1.2 Đa thác chia đường tròn....................................................... 13
1.2.1 Đa thác chia đường tròn........................................... 13
1.2.2 V n dụng..............................................................19
1.3 Định lý Zsigmondy................................................................. 21
1.3.1 Định lý Zsigmondy ..................................................... 21
1.3.2 V n dụng Định lý Zsigmondy ...............................23
2 Tính chat so hoc của đa thfíc 27
2.1 Tính chat đ c bi t của đa thác thu®c Z[x]..........................27
2.1.1 Định lý Bézout.........................................................27
2.1.2 V n dụng..............................................................29
2.2 Đa thác Hilbert và bieu dien Mahler................................... 38
2.3 V n dụng giải bài toán thi hoc sinh giỏi.............................40
Ket lu n 44
Tài li u tham khảo 45
2
Viết đề tài giá sinh viên – ZALO:0973.287.149-TEAMLUANVAN.COM
L i cảm ơn
Lu n văn này được thực hi n tại Trường Đại hoc Khoa hoc - Đại hoc
Thái Nguyên và hoàn thành với sự hướng dan của PGS.TS. Đàm Văn
Nhỉ. Tác giả xin được bày tỏ lòng biet ơn chân thành và sâu sac tới
người hướng dan khoa hoc của mình, người đã đ t van đe nghiên cáu,
dành nhieu thời gian hướng dan và t n tình giải đáp nhǎng thac mac
của tác giả trong suot quá trình làm lu n văn.
Tác giả xin trân trong cảm ơn Ban Giám hi u Trường Đại hoc Khoa
hoc - Đại hoc Thái Nguyên, Ban Chủ nhi m Khoa Toán - Tin, cùng các
giảng viên đã tham gia giảng dạy, đã tạo moi đieu ki n tot nhat đe tác
giả hoc t p và nghiên cáu.
Tác giả muon gải nhǎng lời cảm ơn tot đep nhat tới t p the Lớp B,
cao hoc Toán khóa 10 (2016 - 2018) đã đ®ng viên và giúp đơ tác giả rat
nhieu trong suot quá trình hoc t p.
Nhân dịp này, tác giả cũng xin chân thành cảm ơn Sở Giáo dục và Đào
tạo Hải Phòng, Ban Giám hi u và các đong nghi p ở Trường THPT Lý
Thường Ki t, Huy n Thủy Nguyên, Thành pho Hải Phòng đã tạo đieu
ki n cho tác giả hoàn thành tot nhi m vụ hoc t p và công tác của mình.
Cuoi cùng, tác giả muon dành nhǎng lời cảm ơn đ c bi t nhat đen bo
me và đại gia đình đã luôn đ®ng viên và chia sẻ nhǎng khó khăn đe tác
giả hoàn thành tot lu n văn này.
3
Viết đề tài giá sinh viên – ZALO:0973.287.149-TEAMLUANVAN.COM
L i nói đau
Đa thác có vị trí rat quan trong trong Toán hoc vì nó không nhǎng là
m®t đoi tượng nghiên cáu trong tâm của Đại so mà còn là m®t công
cụ đac lực của Giải tích trong lý thuyet xap xỉ, lý thuyet bieu dien,
lý thuyet n®i suy,... Ngoài ra, đa thác còn được sả dụng nhieu trong
tính toán và áng dụng. Trong các kì thi hoc sinh giỏi toán quoc gia và
Olympic toán quoc te thì các bài toán ve đa thác cũng thường được đe
c p đen và được xem như nhǎng bài toán khó của b c phő thông.
Đã có nhieu đe tài viet ve đa thác nhưng trong lu n văn của mình tôi
muon t p trung xét vi c v n dụng đa thác trong so hoc.
Mục đích của lu n văn này là giới thi u Định lý Zsigmondy - m®t định
lý rat mạnh trong xả lý các bài toán khó ve so nguyên to và giới thi u
tính chat đ c bi t của đa thác thu®c Z[x].
Lu n văn gom phan mở đau, ket lu n và hai chương.
Chương 1. Định lj Zsigmondy. Chương này gom ba mục chính:
Mục 1.1 trình bày ve m®t so tính chat cơ bản ve đa thác và so phác.
Mục 1.2 trình bày ve đa thác chia đường tròn.
Mục 1.3 trình bày ve Định lý Zsigmondy và v n dụng Định lý Zsigmondy
trong giải m®t so bài toán thi hoc sinh giỏi.
Chương 2. Tính chat so hoc của đa thfíc. Chương này được chia
thành ba mục chính:
Mục 2.1 trình bày ve tính chat đ c bi t của đa thác thu®c Z[x].
Mục 2.2 trình bày ve đa thác Hilbert và bieu dien Mahler.
Mục 2.3 trình bày ve cách v n dụng đa thác Hilbert.
4
Viết đề tài giá sinh viên – ZALO:0973.287.149-TEAMLUANVAN.COM
Σ }
Σ
Σ Σ
Chương 1
Định lj Zsigmondy
Trước khi giới thi u ve định lý Zsigmondy, phan đau của chương này
lu n văn trình bày các kien thác cơ sở ve đa thác, trường so phác và đa
thác chia đường tròn. Các kien thác trong chương này được tham khảo
tà tài li u [1] và [3].
1.1 Đa thfíc và so phfíc
1.1.1 Khái ni m đa thfíc, phép toán
Mục này t p trung nghiên cáu vành các đa thác m®t bien trên m®t
trường. Trường K có the là trường Q, R, C. Ký hi u t p đa thác trên K
n
K[x] = {a0 + a1x + · · · + anxn
|ai ∈ K, n ∈ N} = aixi
| ai ∈ K .
i=0
Moi phan tả thu®c K[x] được viet là f(x) ho c đơn giản f. Phan tả
n
f =
i=0
aix
i
với quy ước x0
= 1, được goi là m®t đa thúc của bien x với
các h tả thu®c K. Khi an 0 và n là so tự nhiên thì n được goi là b c
của đa thác f và được ký hi u n = deg f ; an được goi là h tủ cao nhat; a0
được goi là h tủ tự do hay so hạng tự do. Trường hợp f = a /= 0, a ∈ K
,
được goi là đa thác b¾c 0. Đ c bi t, khi f = 0 thì đa thác này được quy
ước có b c −1 ho c −∞, tùy theo vi c sả dụng b c vào lĩnh vực nào.
Đa thác dạng f = a0 + a1x + · · · + an−1xn−1
+ xn
được goi là đa thúc
monic. Các phép toán trong K[x] : Với f =
n
i=0
aixi
, g =
m
i=0
bix
i
∈ K[x]
5
Viết đề tài giá sinh viên – ZALO:0973.287.149-TEAMLUANVAN.COM
Σ Σ
ta định nghĩa
f = g khi và chỉ khi
m = n
ai = bi, i = 0, 1, . . . , n
f + g =
Σ
(ai + bi)xi
, fg =
m
Σ
+n Σ
i
ai−jbj xi
.
i=0 i=0 j=0
M nh đe 1.1. Với các phép toán trên, K[x] l¾p thành m®t vành giao
hoán có đơn v .
M nh đe 1.2. Với hai đa thúc f, g ∈ K[x] ta có các ket quả ve b¾c:
(1) deg(f + g) ≤ max{deg f, deg g}.
(2) deg(fg) = deg f. deg g.
Chfíng minh. (1) Giả sả f =
n
i=0
aix
i
và g =
m
i=0
bixi
. Không hạn che
có the coi m ≤ n. Neu m < n thì deg(f + g) = n ≤ max{n, m}. Neu
m = n và an + bn =
/ 0 thì deg(f + g) = n = max{n, n}. Neu m = n
và an + bn = 0 thì deg(f + g) < n = max{n, n}. Tóm lại, ta luôn có
deg(f + g) ≤ max{deg f, deg g}.
(2) Vì an, bm /= 0 nên an.bm 0. Do v y deg(fg) = m.n = deg f. deg g.
1.1.2 Thu t toán Euclid
Cho hai đa thác f(x) và g(x) với b c n = deg f(x) và m = deg g(x).
Giả thiet m > 0. Neu có đa thác h(x) đe f(x) = h(x)g(x) thì ta nói
rang f (x) chia het cho g(x) với thương h(x). Neu không có đa thác h(x)
nào đe f (x) = h(x)g(x) thì ta nói rang đa thác f (x) không chia het cho
g(x). Ta có hai đa thác duy nhat h(x), r(x) đe
f(x) = h(x)g(x) + r(x), deg r(x) < m.
Đa thác r(x) được goi là đa thác dư trong phép chia đa thác f(x) cho
đa thác g(x).
6
Viết đề tài giá sinh viên – ZALO:0973.287.149-TEAMLUANVAN.COM
−
Định lj 1.1. Với các đa thúc f(x), g(x) thu®c vành K[x] và g(x) /= 0
có hai đa thúc duy nhat q(x), r(x) sao cho f (x) = q(x)g(x) + r(x), trong
đó deg r(x) < deg g(x).
Chfíng minh. Sự ton tại: Giả sả f(x) = anxn
+an−1xn−1
+· · ·+a2x2
+
a1x + a0 và g(x) = bmxm
+ bm−1xm−1
+ · · · + b1x + b0.
Neu n < m thì chon q(x) = 0, r(x) = f(x).
Neu n ≥ m thì ta xét hi u f1
(x) = f(x)
an
xn−m
g(x). Khi đó n =
bm
1
deg f1 (x) ≤ n−1. Neu n1
< m thì chon q(x) =
an
xn−m
và r(x) = f
bm
1(x).
Neu n1 ≥ m ta tiep tục quá trình trên. Sau m®t so hǎu hạn bước, ta
đạt được q(x) và r(x) thỏa mãn các yêu cau đ t ra.
Tính duy nhat: Giả sả có các đa thác q1(x), q2(x), r1(x), r2(x) thỏa mãn
q1(x)g(x)+r1(x) = f (x) = q2(x)g(x)+r2(x) với deg r1(x), deg r2(x) < m.
Tà đây suy ra
[q1(x) − q2(x)]g(x) = r1(x) − r2(x).
Neu q1(x)−q2(x) /= 0 thì deg[q1(x)−q2(x)]g(x) ≥ m > deg[r1(x)−r2(x)],
vô lý. Tà đó suy ra q1(x) = q2(x) và r1(x) = r2(x).
Định nghĩa 1.1. Đa thác d(x) được goi là nhân tủ chung của hai đa
thác f(x) và g(x) neu f(x) và g(x) cùng chia het cho đa thác d(x). Hai
đa thác f (x) và g(x) được goi là nguyên to cùng nhau neu chúng chỉ có
ước chung là các đa thác b c 0.
Định lj 1.2. [Bézout] Hai đa thúc f(x) và g(x) nguyên to cùng nhau
khi và chí khi có hai đa thúc p(x), q(x) đe p(x)f(x) + q(x)g(x) = 1.
Định lj 1.3. Vành K[x] là m®t vành chính và nó là vành nhân tủ hóa.
1.1.3 Xây dfing trư ng so phfíc C
Xét tích T = RxR = {(a, b) |a, b ∈ R}. Với kí hi u i ∈
/ R ta đong nhat
c p (a, b) với a + bi và tích Carte T = RxR được coi như t p
7
Viết đề tài giá sinh viên – ZALO:0973.287.149-TEAMLUANVAN.COM
√
}
T = {(a + bi) |a, b ∈ R}. Định nghĩa các phép toán trong T:
a + bi = c + di khi và chỉ khi a = c, b = d
(a + bi) + (c + di) = (a + c) + (b + d)i
(a + bi).(c + di) = (ac − bd) + (ad + bc)i
a = a + 0i, i = 0 + bi, bi = ib.
Đe đơn giản, ta quy ước viet (a + bi)(c + di) thay cho (a + bi).(c + di)
Tà định nghĩa, ta có :
(1) Với i = 0 + 1i ∈ T có i2
= (0 + 1i)(0 + 1i) = −1 + 0i = −1
(2) (a + bi)(1 + 0i) = a + bi = (1 + 0i)(a + bi).
Ký hi u C là t p T cùng với các phép toán đã nêu ra ở trên. Ta có:
Bo đe 1.1. Ánh xạ φ : R → C, a ›→ (a, 0), là m®t đơn ánh và nó thóa
mãn φ(a + a′) = φ(a) + φ(a′), φ(aa′) = φ(a)φ(a′) với moi a, a′ ∈ R.
Đong nhat (a, 0) ∈ C với a ∈ R. Khi đó ta có the viet
(a, b) = (a, 0) + (b, 0)(0, 1) = a + bi với i2
= (−1, 0) = −1.
Do đó i hay a ho c a+ bi là bình đȁng trong C.
Như v y C = (a + bi) |a, b ∈ R, i2
= −1 và trong C ta có các ket quả:
a + bi = c + di khi và chỉ khi a = c, b = d
a + bi + c + di = a + c + (b + d)i
(a + bi)(c + di) = ac − bd + (ad + bc)i.
Moi phan tả z = a + bi ∈ C được goi là m®t so phác với phan thực a, ký
hi u Re(z), và phan ảo b, ký hi u Im(z); còn i được goi là đơn vị ảo. So
phác a − bi được goi là so phác liên hợp của của z = a + bi và được ký
hi u là z = a + bi. Ta có zz = (a + bi) (a − bi) = a2
+ b2
, z1z2 = z1 z2 và
goi |z| = zz là mô-đun của z. So đoi của z′ = c + di là −z′ = −c − di
và hi u z − z′ = (a + bi) − (c + di) = a − c + (b − d)i.
Xét m t phȁng toa đ® (Oxy). Moi so phác z = a + bi ta cho tương áng
với điem M (a;b). Tương áng này là m®t song ánh:
C → R × R, z = a + bi → M (a; b) .
8
Viết đề tài giá sinh viên – ZALO:0973.287.149-TEAMLUANVAN.COM
a
2
√
−
−
Khi đong nhat C với (Oxy) qua vi c đong nhat z với M, m t phȁng
toa đ® bieu dien so phác như the goi là m¾t phȁng phúc hay m t phȁng
Gauss, ghi công C. F. Gauss-người đau tiên đưa ra bieu dien.
M nh đe 1.3. C là trường chúa trường R như m®t trường con.
Chfíng minh. De dàng kiem tra C là m®t vành giao hoán với đơn vị 1.
Giả sả z = a + bi =
/ 0. Khi đó a2
+ b2
> 0. Giả sả z′ =x + yi ∈ C
thỏa mãn zz’=1 hay (
ax − by = 1
. Giải h ta được x = a2 + b2
b
bx + ay = 0
y = −
a2 + b2
V y z′ =
a
a2 + b2
b
—
a2 + b2
i là nghịch đảo của z, ký hi u là z−1
=
1
.
z
Như v y C là m®t trường . Tương áng C → C, z → z, là m®t tự đȁng
cau liên hợp. Đong nhat a ∈ R với a + 0i ∈ C và coi R như là m®t trường
con của C hay R ⊂ C.
Chú ý, nghịch đảo của z 0 là z−1
=
z
|z|
và
z′
= z′z−1
=
z′
z
.
z |z|
Định nghĩa 1.2. Cho so phúc z /= 0. Giả sủ M là điem trong m¾t phȁng
phúc bieu dien so phúc z. So đo (rađian) của mői góc lượng giác tia đau
Ox và tia cuoi OM được goi là m®t Argument của z và được ký hi u là
Arg(z). Góc α=x
^
OM, − π ≤ α ≤ π, được goi là argument của z và được
ký hi u bới argz. Argument của so phúc 0 là không đ nh nghĩa.
Chú ý, neu α là m®t argument của z thì moi argument của z đeu có
dạng α+k2.π với k ∈ Z. Với z /= 0 , ký hi u α+k.2π là argument của z
.
Ký hi u r = zz. Khi đó so phác z = a + bi có a = rcosα, b = r sin α.V
y khi z =
/ 0 thì có the bieu dien z = r (cos α + i sin α) và bieu dien này
được goi là dạng lượng giác của z.
Ví dn 1.1. Với a + bi =(x + iy)n
có a2
+ b2
= x2
+ y2 n
.
Bài giải. Tà a + bi = x + iy
n
suy ra a bi = x
a2
+ b2
= x2
+ y2 n
.
iy
n
. Như v y
2
9
Viết đề tài giá sinh viên – ZALO:0973.287.149-TEAMLUANVAN.COM
(2) z1z2 = r1r2 [cos (α1 + α2) + i sin (α1 + α2)] .
M nh đe 1.4. [Moivre] Neu z = r (cosα + i sin α) thì với mői so nguyên
dương n có zn
=rn
[cos (na) + i sin (na) ].
H quả 1.1. Căn b¾c n của m®t so phúc z = r(cosa + i sin a) /= 0 là n
1
giá tr khác nhau zk=rn (cos
α + 2kπ
+ i sin
n
α + 2kπ
) với k = 1,2,...,n.
n
Tích vô hướng và tích l ch của hai so phác z1, z2, ký hi u < z1, z2 > và
[z1, z2], được định nghĩa tương áng qua các công thác sau đây:
<z1, z2 >=
1
2
(z1z2 + z1z2) , [z1,z2] =
1
2i
(z1z2 − z1z2) .
M nh đe 1.5. Neu z1 = r1 (cos α1 + i sin α1) , z2 = r2 (cos α2 + i sin α2]
với r1, r2 ≥ 0 thì: . .
(1) |z z | = |z ||z
| , .z1
. =
|z1|
và |z + z | ≤ |z
| + |z | .
1 2 1
2 .z2
.
|z2|
1 2 1 2
(3)
z1
z2
=
r1
[cos (α
r2
1
— α2) + i sin (α1 — α2)] khi r2 > 0.
(4) <z1, z2 >= |z1| |z2| cos (α1 − α2) và < z1, z2 >=< z2, z1>.
(5) < az1 + bz3, z2 >= a < z1, z2 > +b < z3, z2 > với moi so phúc z1, z2, z3
10
Viết đề tài giá sinh viên – ZALO:0973.287.149-TEAMLUANVAN.COM
và moi a, b ∈ R.
(6) [z1, z2] = |z1| |z2| sin (α2 − α1) và [z1, z2] = −[z2, z1].
(7) Với z1 = cosα1 + i sin α2, z2 = cosα2 + i sin α2 ta có bieu dien
11
Viết đề tài giá sinh viên – ZALO:0973.287.149-TEAMLUANVAN.COM
z − z
= 2i sin
α1 − α2
cos
α1 + α2
+ i sin
α1 + α2
12
Viết đề tài giá sinh viên – ZALO:0973.287.149-TEAMLUANVAN.COM
13
Viết đề tài giá sinh viên – ZALO:0973.287.149-TEAMLUANVAN.COM
14
Viết đề tài giá sinh viên – ZALO:0973.287.149-TEAMLUANVAN.COM
| = 2 .sin
α1 − α2 . .
15
Viết đề tài giá sinh viên – ZALO:0973.287.149-TEAMLUANVAN.COM
1 2 . 2 .
M nh đe 1.6. Với hai so phúc z1 và z2 ta luôn có các h thúc liên h sau:
16
Viết đề tài giá sinh viên – ZALO:0973.287.149-TEAMLUANVAN.COM
z1 = z2 ⇔ |z1| = |z2| , arg z1 = arg z2 + 2kπ, k ∈ Z.
17
Viết đề tài giá sinh viên – ZALO:0973.287.149-TEAMLUANVAN.COM
arg( z1z2 ) = arg(z1) + arg(z2) + 2kπ, k ∈ Z.
Arg (z1z2) = Arg (z1) + Arg (z2) .
Arg
z1
= Arg (z
z2
1) − Arg (z2) .
Người ta chỉ ra rang, đa thác b c dương thu®c C[x] đeu có nghi m trong
C. Đó chính là n®i dung Định lý cơ bản của đại so mà người đau tiên
cháng minh là nhà toán hoc Gauss (1977 − 1985).
Định nghĩa 1.3. Trường K được goi là m®t trường đóng đại so neu moi
đa thúc b¾c dương thu®c K[x] đeu có nghi m trong K.
Như v y trong K[x] moi đa thác b c dương đeu phân tích được thành
tích các nhân tả tuyen tính khi K là m®t trường đóng đại so.
Định lj 1.4. [D’Alembert-Gauss, định lý cơ bản của đại so] Moi đa
thúc b¾c dương thu®c C[x] đeu có ít nhat m®t nghi m thu®c C.
Ví dn 1.2. Giả sủ hai đa thúc f, g ∈ R [x]  {0} thóa mãn
f (x2
+ x + 1) = f (x)g(x). Chúng minh rang b¾c của f là so nguyên
dương chȁn.
Bài giải. Giả sả đa thác f có b c lẻ. Khi đó f (x) = 0 có ít nhat m®t
nghi m thực α. Tà h thác f(x2
+ x + 1) = f(x)g(x) suy ra α2
+ α + 1
cũng là nghi m của f(x) = 0. Theo quy lu t này, dãy so a0 = α và
2
r+1 + ar + 1 đeu là nghi m của f(x) = 0. Đieu đó cháng tỏ, f(x) = 0
có vô hạn nghi m, mâu thuan . Do v y, tat cả các nghi m của f (x) = 0
đeu là so phác, không là so thực. Tà đó suy ra b c của f là m®t so
nguyên dương chȁn.
Ví dn 1.3. Chúng minh rang, tat cả các nghi m của đa thúc dạng
f(x) = xn
+ 2nxn−1
+ 2n2
xn−2
+ ... không the đeu là nhũng so thực.
Bài giải. Giả sả tat cả các nghi m x1, x2, ..., xn đeu là nhǎng so thực.
) + 2kπ, k ∈ Z.
2
) − arg (z
1
= arg (z
z
1
z
2
arg
a
18
Viết đề tài giá sinh viên – ZALO:0973.287.149-TEAMLUANVAN.COM
Σ n
n Σ n
n−1
n
i
Σ
i
.
Ví dn 1.5. Giả sủ n so phúc z1, ..., zn thóa mãn |z1|+
. · · · +.|zn| = 2017.
. k. 4
4 4
. k.
4
Σ
xi
!
Theo định lý Vi-ét ta có:
n
xi = −2n,
i=1
1≤
Σ
i<j≤n
xixj = 2n2
.
V y 2n2
=
Σ
xixj = 1
"
Σ
n
2 n
xi − xi
2
#
≤ n − 1 Σ
n 2
1≤i<j≤n
2 i=1 i=1 2n i=1
và suy ra 2n2
< 2n(n − 1), mâu thuan. Như v y đieu giả sả là sai, suy
ra đieu phải cháng minh.
Ví dn 1.4. Tìm tat cả các đa thúc f(x) = a0xn
+a1xn−1
+a2xn−2
+...+an
với các ai ∈ {−1; 1} với moi i sao cho tat cả các nghi m đeu là so thực.
Bài giải. Giả sả tat cả các nghi m x1, x2, ..., xn đeu là nhǎng so thực.
Theo định lý Vi-ét ta có:
Σ
i=1
xi
2
=
n
i=1
2
xi − 2
1≤
Σ
i<j≤n
xixj = a2
— 2an−2 ≤ 3,
Y
i=1
x2
= 1.
V y
n
i=1
x2
≥ n. Tà đây suy ra n ≤ 3 và x1x2x3 = ±1. Tà đó xác định
được các đa thác x ± 1, x2
± x − 1, x3
− x ± (x2
− 1).
Chúng minh, ton tại m®t t¾p con I ⊂ {1, 2, ..., n} đe .
Σ
z
2017
≥ .
Bài giải. Giả sả zk = ak + ibk, ak, bk ∈ R, với k = 1, 2, ..., n. Khi đó
n n n
2017 =
Σ
|zk| ≤
Σ
|ak| +
Σ
|bk| =
Σ
ak −
Σ
ak +
Σ
bj −
Σ
bj
với các t p con I, J ⊂ {1, 2, ..., n} và ak, bj ≥ 0. Tà bieu dien này ta suy
ra ho c
Σ
a
2017
≥ ho c
Σ
b
≥
2017
, chȁng hạn
Σ
a
2017
≥ .
Như v y .
Σ
z . ≥
Σ
a
2017
≥ .
j∈J
k∈I
k∈I
k=1 k=1 k=1 k∈I k∈
/I j∈J j∈
/J
k
4
j
k∈I
k
k∈I
k∈I
k
19
Viết đề tài giá sinh viên – ZALO:0973.287.149-TEAMLUANVAN.COM
k
1 2 n
2 2 2 2 2 2
Ví dn 1.6. Giả sủ đa thúc f(x) = xn
+a1xn−1
+a2xn−2
+· · ·+an ∈ C[x]
với n nghi m α1, α2, ..., αn và đa thúc g(x) = xn
+b1xn−1
+b2xn−2
+· · ·+bn
với n nghi m α2
, α2
, ..., α2
. Chúng minh rang, neu a1 + a3 + a5 + · · · và
a2 + a4 + a6 + · · · đeu là nhũng so thực thì g(1) cũng là m®t so thực.
Bài giải. Ta có:
n n n
g x2
=
Y
x2
− α2
=
Y
(x − αk)
Y
(x + αk) = (−1)n
f (x) f (−x) .
k=1
Bieu dien
k=1 k=1
g(1) − 1 = (−1)n
f(1)f(−1) − 1 = (−1)n
(1 + b − a)(1 + b + a),
ở đó a = a1 + a3 + a5 + · · · và b = a2 + a4 + a6 + · · · đeu là nhǎng so
thực. Do v y, g(1) cũng là m®t so thực.
Ví dn 1.7. Tính lim cos2x+ · · · + cosnx .
1 1 1
n→∞
Bài giải. Đ t tőng
1 +
2
cosx+
22 2n
1 1 1
T = 1 +
2
cosx+
22
cos2x+ · · · +
2n
cosnx
và tőng
1 1 1
iS = i
2
sinx+i
22
sin2x+ · · · +i
2n
sinnx.
Ta có the bien đői f = T + iS :
f = 1 +
cos x
+ i
sin x
+
cos x
+ i
sin x 2
+ · · · +
cos x
+ i
sin x n
1 −
cos x
−
cos(n + 1)x
+
cos(n − 1)x
= 2 2n+1
5
4
− cos x
2n+2
+ i . . .
V y lim
1 1
cos2x+ · · · +
1
cosnx =
4 − 2cosx
.
n→∞ 1 +
2
cosx+
22 2n 5 − 4cosx
20
Viết đề tài giá sinh viên – ZALO:0973.287.149-TEAMLUANVAN.COM
−
Ví dn 1.8. Cho f(x) = x3
+ 15x2
+ 75x + 120. Hãy giải phương trình
f(f(. . . (f(f(x)) . . .)) = x và tính tőng tat cả các nghi m.
`
n
˛
la
¸
nf
x
Bài giải. Ta có f(x) = (x +5)3
− 5. Bang quy nạp theo n ta nh n được
f(f(...(f(f(x))...)) = (x + 5)3n
− 5.
`
n
˛
la
¸
nf
x
Giải ra x0 = −5 và nghi m của phương trình (x + 5)3n−1
= 1.
Tóm lại x0 = −5 và xk
k2π
= −5+cos
3n − 1
+i
sin k2π
với k = 1, 2, ..., 3n
1.
3n − 1
Tőng tat cả các nghi m của phương trình T = −5.3n
.
1.2 Đa thfíc chia đư ng tròn
1.2.1 Đa thfíc chia đư ng tròn
Định nghĩa 1.4. Cho n là m®t so nguyên dương và α là m®t căn b¾c
n của đơn v . Khi đó so nguyên dương nhó nhat k sao cho αk
= 1 được
goi là cap của α và kí hi u là ord(α). Ví dụ: Các căn b c 4 của đơn vị
là 1, −1, i, −i. Cap của 1 là 1, cap của -1 là 2, cap của i là 4 , cap của -i
là 4.
Định nghĩa 1.5. Cho n là so nguyên dương và α là m®t căn b¾c n
của đơn v . Khi đó α được goi là căn nguyên thủy b¾c n của đơn v neu
ord(α) = n. Ví dụ: Các căn b c 3 của đơn vị là:
1 i
√
3 1 i
√
3
α0 = 1, α1 = −
2
+
2
, α2 = −
2
−
2
.
Có ord(α0) = 1, ord(α1) = 3, ord(α2) = 3. Nên các căn nguyên thủy b c
ba của đơn vị là α1, α2.
Định nghĩa 1.6. Ta biet rang căn b¾c n của đơn v là αk = cos
với k = 1, 2, ..., n và chúng l¾p thành m®t nhóm xyclic cap n.
k.2π
+ i sin
n
k.2π
n
V y ta phải giải phương trình (x + 5)3n
= x + 5.
21
Viết đề tài giá sinh viên – ZALO:0973.287.149-TEAMLUANVAN.COM
n
Q
Y
3
2 2
− −
2
−
2
Đa thúc
φn (x) =
k=1,
Y
(k,n)=1
(x − αk).
được goi là đa thúc chia đường tròn thú n.
Ví dụ: Các căn nguyên thủy b c 3 của đơn vị là
1 i
√
3 1 i
√
3
α1 = −
2
+
2
, α2 = −
2
−
2
.
Do đó đa thác chia đường tròn thá ba là:
φ (x) =
"
x − −
1
+
i
√
3
!# "
x
1 i
√
3
!#
= x2
+ x + 1.
Các căn nguyên thủy b c 4 của đơn vị là α1 = i và α3 = −i. Do đó đa
thác chia đường tròn thá tư là:
φ4(x) = (x − i)(x + i) = x2
+ 1.
Định lj 1.5. Cho n là so nguyên dương. Khi đó
a) xn
− 1 = φd (x)
d|n
b) các h so của đa thúc chia đường tròn đeu là các so nguyên, túc là
φn (x) ∈ Z [x].
Chfíng minh. a) Với moi so tự nhiên k ∈ 1, 2, ..., n, (k, n) > 1, có ước
d của n đe αk là căn nguyên thủy b c d của đơn vị. V y φd(x) chia het
cho (x − αk). Ngược lại, moi căn nguyên thủy b c d của đơn vị, ở đó d
là m®t ước của n, thì nó cũng là m®t căn b c n của đơn vị.
Do v y, ta có
xn
− 1 = φd (x).
dn
b) Đa thác φ1(x) = x − 1 có h so nguyên. Giả sả đa thác φd(x) có các
h so đeu là so nguyên với moi ước dương d < n của n . Vì phép chia
22
Viết đề tài giá sinh viên – ZALO:0973.287.149-TEAMLUANVAN.COM
d|n
Σ
Q Q
1
Q
xn
− 1
φn (x) =
φ
d
d|n,d<n
(x)
thực hi n trong Z[x] nên các h so của đa thác
φn(x) đeu là các so nguyên.
Định nghĩa 1.7. Hàm Euler ϕ : N+
→ N được đ nh nghĩa như sau:
ϕ(1) = 1. Cho n < 1. Khi đó ϕ(n) là so các so tự nhiên nhó hơn n và
nguyên to cùng nhau với n.
Ví dụ: ϕ(2) = 1, ϕ(3) = 2, ϕ(4) = 2, ϕ(5) = 5.
H quả 1.2. Với moi so nguyên dương n ta luôn có n = ϕ (d).
d|n
Chfíng minh. Do xn
− 1 =
Q
φn (x) theo định lý 1.5 và so sánh b c
hai ve ta nh n được n = deg(xn
− 1) = deg(
Q
φd (x)) =
Σ
ϕ (d) .
Định lj 1.6. Với so nguyên dương lé n > 1, ta có φ2n(x) = φn(−x).
Chfíng minh. Vì n là so nguyên dương lẻ nên ϕ(2n) = ϕ(n). Giả sả
n = ps1
...psr
là phân tích tiêu chuȁn của n. Do n là so lẻ nên các so pi
1 r
đeu là so lẻ.
Vì ϕ(n) = n 1 −
1
... 1 −
1
nên ϕ(n) là m®t so chȁn.
p1
Ta thay ngay −αk = −cos
pr
k.2π
n
− i sin
k.2π
n
với k ∈ 1, 2, ..., n, (k,n)=1,
là tat cả các căn nguyên thủy b c 2n của đơn vị. Do v y
φ2n(x) =
n
k=1,(k,n)=1
(x + αk) = (−1)
ϕ(n)
n
k=1,(k,n)=1
−x − αk
= φn (−x)
V n dụng ket quả của định lý trên ta cháng minh công thác tường minh
của φn(x).
Định nghĩa 1.8. Hàm Mobius µ : Z+
→ {−1, 0, 1} được đ nh nghĩa
như sau: Đ¾t µ(1) = 1. Cho n > 1. Neu d2
không là ước của n với moi
so tự nhiên d > 1 thì ta đ¾t µ(n) = (−1)k
, trong đó k là so các ước
nguyên to của n. Neu có so tự nhiên d > 1 sao cho d2
là ước của n thì
ta đ¾t µ(n) = 0.
d|n
d|n
23
Viết đề tài giá sinh viên – ZALO:0973.287.149-TEAMLUANVAN.COM
Σ
Σ
n
d
Q
t
t
t
QQ
Q µ( )
d
(x) = x
− 1 .
d
d
d
d
n
t
Tà định nghĩa ta có µ(6) = (−1)2
= 1, mu(9) = 0, µ(12) = 0. Hien
nhiên µ là hàm nhân, tác là µ(mn) = µ(m)µ(n) với moi so nguyên
dương m, n nguyên to cùng nhau.
Sau đây giới thi u tính chat của hàm Mobius.
M nh đe 1.7. Cho n là so nguyên dương. Khi đó
a) Neu n = 1 thì µ (d) = 1.
dn
b) Neu n ≥ 2 thì µ (d) = 0.
dn
Định lj 1.7. Ta luôn có φn (x) =
Q
xd
− 1 với moi n ∈ N∗
.
µ(d
)
d|n
Chfíng minh. Chỉ can cháng minh các đa thác φn (x) =
Q
xd
− 1
µ(n
)
d|n
thỏa mãn công thác xn
− 1 = φd (x). Ta bien đői tích các đa thác
d|n
Y
φd (x) =
Y Y
xt
− 1 .
µ(d
)
d|n d|n t|d
So mũ của nhân tả xt
− 1 bang tőng
Σ
µ d
=
Σ
µ(d′) =
(
1 neu n = t
theo ket quả ve hàm Mobius.
d|n,t|d
d′|n
0 neu n > t
Do v y (xt
− 1)µ(d
) = xn
− 1. Và ta được φ
d|n t|d
n
d
d|n
H quả 1.3. Với moi so nguyên dương n ta có ϕ (n) =
Σ
dµ n
.
d|n
Chfíng minh. Do deg(φn(x)) = ϕn nên khi lay đạo hàm hai ve của
φ (x) =
Q
xd
− 1
µ(n
) ta có ϕ (n) =
Σ
dµ n
.
d|n d|n
H quả 1.4. Ta có nN (n) =
Σ
µ (d)p
n
, ớ đó N(n) là so nhân tủ bat
d|n
khả quy π(x) b¾c n với h tủ cao nhat bang 1 của đa thúc xpn
− x.
n
24
Viết đề tài giá sinh viên – ZALO:0973.287.149-TEAMLUANVAN.COM
Y
Y Y
−
Y µ
(
)
d
φn(xp
)
φn(xp
) neu (p, n) = p
pn
d
φn(xp
)
d
xd
Định lj 1.8. Với so nguyên to p ta có:
φn
neu (p, n) = 1.
(x)
Chfíng minh. Xét trường hợp (p, n) = p. Neu α là m®t căn nguyên
thủy b c pn của đơn vị thì ω = αp
là m®t căn nguyên thủy b c n của
đơn vị. Goi γ1w, ...γpw là nghi m của phương trình xp
= ω. Khi đó
p
xp
− ω = (x − γiω).
i=1
Như v y
φpn (x) =
pn
k=1,(k,pn)=1
(x αp
) =
ω=ap
(xp
− ω) = φn (xp
).
Xét trường hợp (p, n) = 1. Theo định lý 1.7 ta có bieu dien sau:
φpn (x) =
d
Y
|pn
µ(pn
)
— 1
Y
d|n
−
pn
d
d|n
xpd
µ(n
)
— 1 .
pn
φ (x) =
Q
xd
− 1
−µ(n
)Q
xpd
− 1
µ(n
)
Do đó, trong trường hợp này ta có φpn (x) =
H quả 1.5. Với so nguyên n > 1 có
.
φn (x)
φn(1) =
p neu n = ps
1 neu n /= ps
.
Chfíng minh. Neu n = ps
thì φps (1) = φps (1p
) = φps−1 (1).
Do v y φps (1) = ... = φp (1) = p.
Neu n ps
, chȁng hạn n = mps
với (m, p) = 1, m > 1, thì
φn (1)
φn (1) = φpsm (1) = φps−1m (1). V y φpsm (1) = ... = φpm (1) = = 1.
φn (1)
d|n
d|n
d
d
φpn(x) =
Vì µ = −µ
n
d nên .
xd
= 1
25
Viết đề tài giá sinh viên – ZALO:0973.287.149-TEAMLUANVAN.COM
k
i
.c
(k)
. |i = 0, 1, ..., m − 1, (k, n) = 1, 1 ≤ k ≤ n
0 1 m−1
Như v y các c
(k)
∈ Z đeu chia het cho p. Neu p > c ≥ .c
(k)
. và các c
(k)
Đ t c = max i .
H quả 1.6. Ta có (a − 1)ϕ(n)
< φn (a) < (a + 1)ϕ(n)
với moi so nguyên
a, n > 1. Đ¾c bi t, neu so nguyên to p|n và a ≥ 3 ta còn có φn(a) > p.
Chfíng minh. Với moi so nguyên a > 1 ta có a − 1 < |a − α| < a + 1
cho moi α ∈ C, |α| = 1. Do v y a − 1 < |a − αk| < a + 1 cho moi αk thỏa
mãn αn
= 1 và (k, n) = 1, 1 ≤ k ≤ n.
Suy ra (a − 1)ϕ(n)
< φn(a) < (a + 1)ϕ(n)
với moi so nguyên a, n > 1.
Khi p|n thì ϕ(n) ≥ (p − 1). Khi đó φn (a) > 2ϕ(n)
≥ 2p−1
> p vì p ≥ 2.
Định lj 1.9. Đa thúc φn(x) bat khả quy với moi n ∈ N∗
.
Chfíng minh. Goi α là m®t căn nguyên thủy của 1 và f (x) với b c m
là đa thác toi tieu monic của α trong Q[x]. Vì φn(α) = 0 nên φn(x)
chia het cho f (x). Theo bő đe Gauss, đa thác f (x) ∈ Z[x]. Neu ta
chỉ ra φn(x) = f (x) thì φn(x) là m®t đa thác bat khả quy. Vi c chỉ ra
φn(x) = f (x) tương đương với vi c cháng minh f (αk
) = 0 khi (k, n) = 1
và k ∈ 1, 2, ..., n. Moi phan tả của Q[α] = Q[x]/(f (x)) có bieu dien duy
nhat a0 + a1α + · · · + am−1αm−1
. Giả sả đa thác h(x) ∈ Z[x]. Bieu dien
h(x) = q(x)f(x) + r(x) với q(x), r(x) ∈ Z[x] và deg r(x) ≤ (m − 1).
Khi đó h(α) có dạng h(α) = r(α) = b0 + b1α + · · · + bm−1αm−1
, (1).
Đ c bi t f αk
= c(k)
+ c(k)
α + · · · + c(k)
αm−1
với tat cả các c(k)
∈ Z.
,. .0 1
m−1 , i
Với so nguyên to p, (p,n)=1, ton tại so nguyên dương k thỏa mãn
(k, n) = 1, p ≡ k(modn). Khi đó αp
= αk
. Vì f(x)p
≡ f(xp
)(modp) nên
f(xp
) = f(x)p
+ pG(x) với G(x) ∈ Z[x]. Với x = α ta có f(αp
) = pG(α).
Bieu dien G(α) trong dạng (1) ta có:
pG (α) = f (αp
) = f αk
= c(k)
+ c(k)
α + · · · + c(k) αm−1
.
chia het cho p thì tat cả các c(k)
= 0. Ta đã chỉ ra f(αp
) = f(αk
) = 0.
Tóm lại neu so nguyên to p thỏa mãn (p,n)=1 và p > c thì f(αp
) = 0.
i
i
i
26
Viết đề tài giá sinh viên – ZALO:0973.287.149-TEAMLUANVAN.COM
k Q
n kq+r k q r r
Q
Hoàn toàn tương tự ta cũng chỉ ra được f(αk
) = 0 với (k,n)=1.
Tà đó suy ra φn(x) = f(x).
1.2.2 V n dnng
Bo đe 1.2. Cho so nguyên dương r với ước nguyên dương thực sự của
n và so nguyên m. Neu so nguyên to p là m®t ước so chung của φn(m)
và φr(m) thì p|n.
Chfíng minh. Vì xn
− 1 = φd(x) và r|n, r < n nên xn
− 1 chia het
d|n
cho φr(x)φn(x) theo định lý 1.5. Vì φn(m) ≡ 0(modp) nên
φn(x) ≡ (x − m)φ(x)(modp). Tương tự φr(x) ≡ (x − m) ψ (x) (modp).
Vì các đa thác φd(x) là bat khả quy theo định lý 1.9 nên (φr(x), φn(x)) = 1.
Tà đây suy ra xn
− 1 ≡ 0(modp) có nghi m m b®i k ≥ 2. V y p|n.
Bo đe 1.3. Cho so nguyên dương n và so nguyên m. Neu so nguyên to
p chia het φn(m) thì p ≡ 1(modn) ho¾c p|n.
Chfíng minh. Giả sả so nguyên to p là m®t ước của φn(m). Tà p|φn(m)
và φn(m)|mn
− 1 suy ra p|mn
− 1. V y (m, p) = 1. Theo Định lý nhỏ Fer-
mat ta có mp−1
≡ 1(modp). Chon so nguyên dương nhỏ nhat k thỏa mãn
mk
≡ 1(modp). M t khác lại có mn
≡ 1(modp). Theo phép chia với dư,
bieu dien n = ks + r với 0 ≤ r < k.
Ta có 1 ≡ m ≡ m ≡ m m ≡ m (modp). Tà cách chon so nguyên
dương k suy ra r = 0 và k|n. Tương tự ta cũng chỉ ra k|p − 1. V y neu
k = n thì n|p − 1 hay p ≡ 1(modn). Neu k < n thì tà mk
≡ 1(modp) suy
ra 0 ≡ m − 1 ≡ φd (m) (modp). Vì p là so nguyên to nên ta có d|k
d|k
đe p|φd(m). Tà d|k và k|n suy ra d|n và p|n theo bő đe 1.2.
Đa thúc chia đường tròn có nhieu úng dựng, m®t trong nhũng úng dựng
phő bien của nó là chúng minh đ nh lý Dirichlet.
Định lj 1.10. [Dirichlet] Với so nguyên dương n có vô so so nguyên to
p thóa mãn p ≡ 1(modn).
27
Viết đề tài giá sinh viên – ZALO:0973.287.149-TEAMLUANVAN.COM
Y
} }
Q Q
Q
Y
i=1
Chfíng minh. Với n = 1 ket quả đúng.
Xét n > 1. Giả sả chỉ có hǎu hạn các so nguyên to p1, p2, ..., ps thỏa mãn
Q
k
các ước nguyên to của n và đ t T = PR. Do n > 1 nên T > 1. Lay so
nguyên dương đủ lớn k sao cho φn(Tk
) > 1. Goi q là m®t ước nguyên to
của φn(Tk
). Vì q là ước của Tnk
− 1 nên q không là ước của T và suy
ra được ngay q không là ước của P và của R. Do v y q /≡ 1(modn) và q
không là ước của n (mâu thuan theo bő đe 1.3).
Ví dn 1.9. [Dự bị IMO 2002] Với các so nguyên to p1, p2, ..., pn > 3, so
2p1p2...pn+
+ 1 có ít nhat 2
2n−1
ước.
Bài giải. Theo định lý 1.5, với x = 2 ta nh n được h thác sau:
(2p1p2...pn
+ 1) (2p1p2...pn
− 1) = 22p1p2...pn
− 1 =
d|2p1p2...pn
φd (2).
Vì 2k
− 1 và 2k
+ 1 chỉ có ước là so lẻ và (2k
− 1, 2k
+ 1) = (2k
+ 1, 2) = 1
nên 2k
− 1 và 2k
+ 1 chỉ có m®t ước dương chung bang 1. Bây giờ ta sě chỉ
ra 2p1p2...pn +1
có ít nhat 2n−1
ước đôi m®t nguyên to cùng nhau và l p 22n−1
tích tà các ước đó ta sě nh n được ít nhat 22n−1
ước của so 2p1p2...pn
+ 1.
Đ t A = d|d là ước của p1p2...pn , B = 2d|d là ước của p1p2...pn .
Hien nhiên A ∩ B = ∅, Card(A) = Card(B) = 2n
. Tà bieu dien dưới đây
d|2p1
Q
p2...pn
φd (2) =
d|p1p2...pn
φd (2)
d′|p1p2...pn
φ2d′ (2)
ta suy ra
= (2p1p2...pn − 1)
d′|p1p2...pn
φ2d′ (2)
(2p1p2...pn
+ 1) =
d′|p1p2...pn
φ2d′ (2).
Goi A là t p tat cả các ước d của p1p2...pn sao cho d là tích m®t so chȁn
các nhân tả tà t p p1p2...pn và goi B là t p tat cả các ước d của p1p2...pn
pi ≡ 1(modn) với i = 1, 2, ..., s. Ký hi u P = pi và R là tích tat cả
28
Viết đề tài giá sinh viên – ZALO:0973.287.149-TEAMLUANVAN.COM
a
Q
n ∈ N∗
|an
− 1. p
,
k=1,(k,n)=1
sao cho d là tích m®t so lẻ các nhân tả tà t p p1p2...pn. khi đó A và B
có cùng lực lượng và moi t p có 2n−1
phan tả. Giả sả a, b ∈ A, a /= b v
à
φa(2), φb(2) không nguyên to cùng nhau. Khi đó là lũy thàa nguyên
b
của m®t so nguyên to. Vì a, b đeu là tích của m®t so chȁn các thàa so
a
nguyên to phân bi t nên
b
không the là lũy thàa nguyên của m®t so
nguyên to. V y φa(2) và φb(2) phải là nhǎng so nguyên to cùng nhau khi
a, b ∈ A, a b . Vì A có 2n−1
phan tả nên 2p1p2...pn
+ 1 =
d′|p1p2...pn
φ2d′ (2)
có không ít hơn 22n−1
ước.
1.3 Định lj Zsigmondy
1.3.1 Định lj Zsigmondy
Bo đe 1.4. Với so nguyên to p và so nguyên a > 1, (a, p) = 1, luôn có
so nguyên dương nhó nhat n đe an
− 1 chia het cho p.
Chfíng minh. Do p là so nguyên to và so nguyên a > 1 với (a, p) = 1
nên ap−1
− 1 chia het cho p theo Định lý nhỏ Fermat. Như v y, có so
nguyên dương n đe an
− 1 chia
,het cho p . Chon n là so nguyên dương nhỏ
Tà bő đe này ta định nghĩa ước nguyên thủy của so nguyên dạng an
± 1.
Định nghĩa 1.9. Cho hai so nguyên dương n và a > 1. So nguyên to p
được goi là m®t ước nguyên thủy của an
− 1 ho¾c (an
+ 1) neu n là so
nguyên dương nhó nhat đe an
≡ 1(modp), ho¾c an
≡ −1(modp).
Tà định nghĩa ta thay rang, neu p là m®t ước nguyên thủy của an
− 1 thì
Q
n
Định lj 1.11. [Zsigmondy] Neu so nguyên to p là m®t ước nguyên
thủy của an
− 1 thì ho¾c p|n ho¾c n|p − 1.
Chfíng minh. Giả sả so nguyên to p là m®t ước của φn(m). Tà p|φn(m)
và φn(m)|mn
− 1 suy ra p|mn
− 1. V y (m, p) = 1. Theo Định lý nhỏ
nhat thu®c . Tính duy nhat của n là hien nhiên.
p chỉ có the là m®t ước của so φn(a).Trong đó φn (a) = (a − αk).
29
Viết đề tài giá sinh viên – ZALO:0973.287.149-TEAMLUANVAN.COM
Q
k
2
Fermat ta có mp−1
≡ 1(modp). Chon so nguyên dương nhỏ nhat k thỏa
mãn mk
≡ 1(modp). M t khác, lại có mn
≡ 1(modp). Theo phép chia
có dư, bieu dien n = ks + r với 0 ≤ r < k.
Ta có 1 ≡ mn
≡ mkq+r
≡ (mk
)q
mr
≡ mr
(modp). Tà cách chon so nguyên
dương k suy ra r = 0 và k|n. Tương tự, ta cũng chỉ ra k|p − 1. V y, neu
k = n thì n|p − 1 hay p ≡ 1(modp). Neu k < n thì tà mk
≡ 1(modp)
suy ra 0 ≡ m − 1 ≡ φd (m) (modp). Vì p là so nguyên to nên có d|k
d|k
đe p|φd(m). Tà d|k và theo bő đe 1.2 ta suy ra d|n và p|n.
Định lj 1.12. [Định lj Zsigmondy] Cho hai so nguyên dương a và
b với (a,b)=1. Giả sủ so nguyên n ≥ 2. Khi đó:
(1) an
− bn
có ít nhat m®t ước nguyên to không chia het ak
− bk
với moi
so nguyên dương k<n, ngoài hai trường hợp: 26
−1 ho¾c n=2, a+b = 2s
.
(2) Neu a,b,n với a>b và n ≥ 2, là nhũng so nguyên dương thì an
+ bn
có ít nhat m®t ước nguyên to không chia het ak
+ bk
với moi so nguyên
dương k<n, ngoài trường hợp: 23
+ 1 = 9 chia het cho 3 = 21
+ 1.
H quả 1.7. Giả sủ so nguyên to p thóa mãn p|an
+ bn
nhưng p / |ak
+ bk
với moi k = 1, 2, ..., n − 1. Khi đó p / |aj
+ bj
với j = n + 1, n + 2, ..., 2n.
Chfíng minh. Đ t j = n + x với x = 1, 2, ..., n − 1. Ta sě chỉ ra p / |ab.
Th t v y, neu p|ab thì p|a ho c p|b, chȁng hạn p|a. Khi đó p|an
. Vì
p|an
và p|an
+ bn
nên p|bn
. Do p là so nguyên to nên p|b. Tà đây suy ra
(a, b) = 1 và p|a, p|b, mâu thuan. Như v y p / |ab.
Chú ý rang, p| (an
+ bn
) (ax
+ bx
) = aj
+ bj
+ ax
bx
(an−x
+ bn−x
) .
Vì p / |ab, p / |(an−x
+ bn−x
) nên p / |(aj
+ bj
) với j = n + 1, n + 2, ..., 2n −
1
.Xét j = 2n, ta có p|(an
+ bn
)2
= a2n
+ 2an
bn
+ b2n
. Neu p|a2n
+ b2n
thì
p|2an
bn
. Vì p / |ab nên p = 2. Do (a, b) = 1 và 2 = p|an
+ bn
nên a, b cùng
là hai so lẻ. Khi đó 2 = p|ak
+ bk
với moi k = 1, 2, ..., n − 1, mâu thuan
với giả thiet. V y p / |a2n
+ b2n
.
H quả 1.8. Giả sủ so nguyên p thóa mãn p|an
+ bn
nhưng p / |ak
+ bk
n
với moi k = 1, 2, ..., n − 1. Khi đó p / |aj
− bj
với moi 1 ≤ j < .
30
Viết đề tài giá sinh viên – ZALO:0973.287.149-TEAMLUANVAN.COM
|
2
Chfíng minh. Xuat phát tà p (an
+ bn
) an−2j
+ bn−2j
ta suy ra ket
quả p|a2n−2j
+ b2n−2j
+ an−2j
a2j
+ b2j
, (1). M t khác ta lại có
p / |(an−j+bn−j)2
=a2n−2j + b2n−2j + 2an−jbn−j, (2).
Lay (1) − (2) ta có p / |an−2j
bn−2j
(a2j
+b2j
− 2aj
bj
).
n
Do v y p / |an−2j
bn−2j
(aj
− bj
)2
và suy ra p / |aj
− bj
với 1 ≤ j < .
1.3.2 V n dnng Định lj Zsigmondy
Trong phan này giới thi u vi c v¾n dựng đ nh lý Zsigmondy đe giải các
bài toán trong các đe thi hoc sinh giói và các bài toán liên quan đen so
nguyên to.
Ví dn 1.10. [Japanese MO 2011] Xác đ nh tat cả các b® năm so nguyên
dương a, n, p, q, r thóa mãn an
− 1 = (ap
− 1)(aq
− 1)(ar
− 1).
Bài giải. Hien nhiên n > p, q, r. Neu a > 2, n > 2 thì an
− 1 có m®t ước
nguyên to không chia het ap
− 1, aq
− 1, ar
− 1 theo định lý Zsigmondy.
V y a ≤ 2 ho c n ≤ 2. Kiem tra a = 1 . Khi đó moi m,n,p,r đủ thỏa mãn.
Kiem tra a = 2. Khi đó 2n
− 1 = (2p
− 1) (2q
− 1) (2r
− 1). Tà đây suy ra
{p, q, r} = {n, 1, 1}. Kiem tra n = 2, n = 1, a > 1 không thỏa mãn.
Ví dn 1.11. [IMO Shortlist 2000] Xác đ nh tat cả các b® ba so nguyên
dương a, m, n thóa mãn (am
+ 1)|(a + 1)n
.
Bài giải. Kiem tra (a, m, n) = (2, 3, n) với n ≥ 2 thỏa mãn đe bài. Neu
a > 1, m ≥ 2 và (a, m) =
/ (2, 3) thì theo Định lý Zsigmondy am
+1
có ước nguyên to không chia het a + 1. Do v y, am
+ 1 không chia het
(a + 1)n
và bài toán không có lời giải. Neu a = 1 ho c m = 1 thì bài
toán là tam thường.
Ví dn 1.12. [Polish MO 2010] Giả sủ p, q là hai so nguyên to thóa mãn
q > p > 2. Chúng minh rang 2pq
− 1 có ít nhat ba nhân tủ nguyên to
khác nhau.
31
Viết đề tài giá sinh viên – ZALO:0973.287.149-TEAMLUANVAN.COM
Bài giải. Hien nhiên 2pq
− 1 chia het cho 2p
− 1 và 2q
− 1. Theo định lý
Zsigmondy 2pq
− 1 có m®t ước nguyên to p1 không chia het 2p
− 1, 2q
− 1.
Hơn nǎa, 2q
− 1 có m®t ước nguyên to p2 không chia het 2p
− 1. Cuoi
cùng, 2p
− 1 có nhân tả nguyên to p3. Do v y 2pq
− 1 có ít nhat ba nhân
tả nguyên to phân bi t p1, p2, p3.
Ví dn 1.13. Tìm tat cả b® bon so nguyên dương x, r, p, n đe p là so
nguyên to và xr
− 1 = p
n
với n, r > 1.
Bài giải. Neu xp
− 1 có m®t ước nguyên to q không chia het x − 1,
(theo Định lý Zsigmondy luôn có q), thì xr
− 1 có hai ước nguyên to
phân bi t. Tà đó suy ra bieu dien xr
− 1 = pn
là không the xảy ra.
Xét các trường hợp đ c bi t:
Trường hợp x = 2; r = 6: Khi đó 26
− 1 = pn
hay 32
.7 = pn
. Đieu này
không the xảy ra.
Trường hợp x + 1 = 2s
, r = 2 : khi đó x2
− 1 = pn
hay (x − 1)(x + 1) = pn
.
Ta có 2s
(2s
− 2) = pn
. Tà đây suy ra s = 2. V y x = 3, r = 2, p = 2, n = 3.
Ví dn 1.14. [Czech - Slovak MO 1996] Xác đ nh tat cả các nghi m
nguyên dương x, y, p, trong đó p là so nguyên to, thóa mãn phương trình:
px
− yp
= 1.
Bài giải. Ta có px
= yp
+ 1. Kiem tra y = 1; khi đó px
= 1p
+ 1 = 2.
V y p = 2, x = 1. Kiem tra y = 2; p = 3: Khi đó 3x
= 23
+ 1 = 9 = 32
.
V y x = 2.
Xét trường hợp y > 1: Theo Định lý Zsigmondy, so yp
+ 1 có ước nguyên
to không chia het y + 1. Vì yp
+ 1, p /= 2, có hai ước nguyên to khác nhau
nên px
= yp
+ 1 không the xảy ra.
Xét trường hợp p = 2: Khi đó y2
+ 1 = 2x
. Như v y y là so lẻ. Đ t
y = 2k + 1. Ta có 2k2
+ 2k + 1 = 2x−1
. Tà đây suy ra x = 1, k = 0 và
nh n được x = 1, y = 1, p = 2.
32
Viết đề tài giá sinh viên – ZALO:0973.287.149-TEAMLUANVAN.COM
Ví dn 1.15. [Romania TST 1994] Chúng minh, dãy so (an = 3n
− 2n
)
không chúa ba so hạng l¾p thành cap so nhân.
Bài giải. Giả sả dãy so (an) có cháa ba so hạng với chỉ so x, y, z và
x < y < z, tương áng l p thành m®t cap so nhân. Khi đó ta có h thác
(3y
− 2y
)2
= (3x
− 2x
) (3z
− 2z
) (∗). Theo Định lý Zsigmondy, so 3z
− 2z
có ước nguyên to p không chia het cho 3y
− 2y
và (*) không the xảy ra.
Như v y đieu giả sả là sai.
Ví dn 1.16. [Italy TST 2003] Xác đ nh tat cả các b® ba so nguyên dương
(a, b, p) thóa mãn 2a
+ pb
= 19a
.
Bài giải. Giả sả có b® ba so nguyên dương (a, b, p) thỏa mãn 2a
+ pb
= 19a
.
De dàng suy ra 17 = 19 − 2 là m®t ước nguyên to của 19a
− 2a
= pb
. Tà
17|pb
suy ra p = 17. Khi đó ta có h thác 19a
− 2a
= 17b
. Với a = 1 ta
có b = 1 . Với a ≥ 2, theo định lý Zsigmondy so 19a
− 2a
có ước nguyên
to p không chia het 17 = 19 − 2. Do v y ước nguyên to p không chia het
cho 17p
, vô lý. V y a không the khác 1.
Ví dn 1.17. Xác đ nh tat cả các so nguyên không âm m, n đe 3m
− 5n
là so chính phương.
Bài giải. Lay theo modulo 4 ta có (−1)m
− 1 ≡ x2
(mod4). Do v y m
phải là so chȁn. Đ t m = 2k với k ∈ N. Khi đó ta có h thác 32k
− 5n
= x2
hay (3k
− x)(3k
+ x) = 5n
. Đ t 3k
− x = 5r
và 3k
+ x = 5s
với s > r . Ta
cũng có 2.3k
= 3k
− x + 3k
+ x = 5r
+ 5s
= 5r
(1 + 5s − r
). Tà đây suy
ra r = 0. Khi đó 2.3k
= 1 + 5s
. Neu k = 0 thì m = 0, s = 0 và ta có
m = n = 0, x = 0. Neu k > 0 thì ta xét 2.3k
= 1 + 5s
. Với s ≥ 3, theo
Định lý Zsigmondy 2.3k
= 1 + 5s
có ước nguyên to p không chia het
1 + 5 = 6|2.3k
, vô lý. V y s = 1, k = 1 và suy ra m = 2, 32
− 51
= 4 = 22
.
Với s = 2 có 2.3k
= 1 + 52
= 26, vô nghi m. Tóm lại m = 2, n = 1 ho c
m = n = 0 và coi 0 cũng là m®t so chính phương.
33
Viết đề tài giá sinh viên – ZALO:0973.287.149-TEAMLUANVAN.COM
Ví dn 1.18. [Vi t Nam TST 2016] Tìm hai so nguyên dương a,n với
a > 2 sao cho mői ước nguyên to của an
− 1 cũng đeu là ước nguyên to
của a32016
− 1.
Bài giải. Có ước chung lớn nhat an
− 1, a32016
− 1 = a(n,32016
) − 1 ;
ước chung lớn nhat của xm
− 1 và xn
− 1 đúng bang x(m,n)
− 1 suy ra
ket lu n đúng cho moi so nguyên a > 2 khi n = 1.
Xét n > 1. Đ t n = 3s
m với (m, 3) = 1. Khi đó ta có:
an
− 1, a32016
− 1 = a3r
− 1, ở đó r = min {s, 2016}.
Xét trường hợp: s > 0. Neu m > 1 thì n > 3r
. Khi đó an
− 1 có m®t
ước nguyên to p không là ước của a3r
− 1 . Do v y, trường hợp này
không the xảy ra . Tà đó suy ra m = 1. Ta có an
− 1 = a3r
− 1. Đe moi
ước nguyên to của an
− 1 cũng là ước của a32016
− 1 thì r ≤ 2016. V y
(a, n) = (a, 3r
) với so nguyên a > 2 và r ≤ 2016.
nguyên to của an
− 1 cũng là ước của a32016
− 1 thì n = 1.
V y (a, n) = (a, 1) với a > 2.
Xét trường hợp n = 2 và a = 2s
− 1 với s > 1. Trường hợp này yêu cau
bài toán luôn thỏa mãn.
= a − 1. Đe moi ước
an
− 1, a32016
− 1
Xét trường hợp s = 0. Khi đó
34
Viết đề tài giá sinh viên – ZALO:0973.287.149-TEAMLUANVAN.COM
Σ
Chương 2
Tính chat so hoc của đa thfíc
Trong chương 2 này, dự kien trình bày m®t so tính chât so hoc của đa
thác. Các kien thác trong chương này sě được tham khảo tà tài li u [1]
và [2].
2.1 Tính chat đ c bi t của đa thfíc thu c Z[x]
2.1.1 Định lj Bézout
Định lj 2.1. Cho đa thúc f(x) ∈ Z[x]. Khi đó, với moi a, b ∈ Z ta luôn
có a − b|f(a) − f(b).
Chfíng minh. Giả sả f(x) = a0xn
+ a1xn−1
+ · · · + an ∈ Z[x]. Với
a, b ∈ Z ta có f(a) − f(b) =
n−1
k=0
ak(a
k
— bk
) chia het cho a − b. Chý ý
rang, khi a = b ta có a − b = 0 và f(a) − f(b) = 0. Do 0 = 0.0 nên
f(a) − f(b) cũng van chia het cho a − b = 0.
Giả sả đa thác f (x) = a0xn
+ a1xn−1
+ · · · + an ∈ Z[x]. Khi đó, có nhieu
ket quả thường được sả dụng. Trước tiên, ta công nh n định lý
Định lj 2.2. [Dirichlet] Cho đa thúc b¾c nhat f (x) = ax+b với a, b ∈ N∗
và (a, b) = 1. Khi đó, t¾p giá tr {f(n)|n ∈ N∗
} chúa nhieu vô hạn so
nguyên to.
Định lj 2.3. Cho đa thúc f(x) ∈ Z[x] với f(0) /= 0 và deg f(x) ≥ 1.
Khi đó, với mői so nguyên dương k luôn luôn có so nguyên to p > |f(0)|
35
Viết đề tài giá sinh viên – ZALO:0973.287.149-TEAMLUANVAN.COM
đe so f(p) có ít nhat k thùa so nguyên to phân bi t.
Chfíng minh. Với k = 1, ton tại so nguyên to p1 > |f(0)| đe |f(p1)| = 1
bởi vì neu khác đi thì f(p)2
= 1 cho moi so nguyên to p. Khi đó đa thác
f(x)2
− 1 có nhieu vô hạn nghi m. Đa thác này phải là đa thác 0 và ta
có f(x) ≡ 1 ho c f(x) ≡ −1, mâu thuan với đieu ki n deg f(x) ≥ 1.
Do f (p1) là so nguyên khác ±1 nên f (p1) có ít nhat m®t ước nguyên
to và ket lu n đúng cho k = 1. Giả sả so nguyên to pk thỏa mãn
pk > |f(0)| và f(pk) có ít nhat k thàa so nguyên to phân bi t. Chú
ý rang, (pk, f(pk) = 1 vì f(pk) ≡ f(0)(mod pk) theo Định lý 2.1 và vì
pk > |f(0)| là so nguyên to. Xét cap so c®ng {f(pk)2
n + pk|n ∈ N∗
}.
Theo Định lý Dirichlet, Định lý 2.2, dãy này có cháa nhieu vô hạn so
nguyên to. Giả sả pk+1 = f (pk)2
m + pk là m®t so nguyên to. Khi đó
f(pk+1) − f(pk) chia het cho pk+1 − pk = f(pk)2
m theo Định lý 2.1.
Do v y f(pk+1) ≡ f(pk)(mod f(pk)2
). Tà đây suy ra sự ton tại của so
nguyên r đe f (pk+1) = f (pk)(1 + rf (pk)). Tích này chỉ ra rang, f (pk+1)
có ít nhat m®t ước nguyên to nhieu hơn so ước nguyên to của f(pk) hay
f (pk+1) có ít nhat k + 1 ước nguyên to. Tóm lại, ket lu n đã được cháng
minh.
Định lj 2.4. [Schur] Giả sủ đa thúc f(x) b¾c n ≥ 1 thu®c vành Z[x].
Khi đó có vô hạn so nguyên to chia het toi thieu m®t so hạng khác không
trong dãy (f(n)) với n ∈ N∗
.
Chfíng minh. Không hạn che ta có the giả thiet f (0) = 1. Vì f (x) = 1
chỉ có m®t so hǎu hạn nghi m nên ton tại so nguyên M đe f (n) /= 1
với moi so nguyên n > M. Hien nhiên f (n!) ≡ 1( mod n!). Khi lay so
nguyên dương n đủ lớn, ta có the xây dựng nhǎng so nguyên to tùy ý
chia het f(n!).
Neu f(0) = 0 thì ket quả là hien nhiên.
f(xf(0))
Neu f(0) /= 0 thì ta xét g(x) = và ta có g(0) = 1 và v n dụng
f(0)
l p lu n trên ta chỉ ra được ket quả nêu ra ở trên của Schur.
36
Viết đề tài giá sinh viên – ZALO:0973.287.149-TEAMLUANVAN.COM
M®t ket quả khá női tieng, Hensel’s Lemma, cũng được nêu ra (không
cháng minh) sau đây.
Định lj 2.5. [Hensel’s Lemma] Giả sủ a0, a1, . . . , an là nhũng so nguyên
và đa thúc f (x) = anxn
+ · · · + a1x + a0 với đạo hàm f ′(x). Giả sủ
so nguyên to p và so nguyên x1 thóa mãn f (x1) ≡ 0(modp), f ′(x1) /≡
0(modp). Khi đó, với mői so nguyên k có duy nhat m®t th¾ng dư xk(modpk
)
đe f(xk) ≡ 0(modpk
) và xk ≡ x1(modp).
2.1.2 V n dnng
Ví dn 2.1. [RMC 2010] Xác đ nh tat cả các đa thúc f (x) ∈ Z[x] thóa
mãn đieu ki n: Ton tại so nguyên dương k đe với moi so nguyên to p,
so f(p) có nhieu nhat k thùa so nguyên to phân bi t.
Bài giải. Giả sả đa thác f(x) ∈ Z[x] thỏa mãn đieu ki n đau bài, có
nghĩa: Có so nguyên k đe f(p) có nhieu nhat k thàa so nguyên to phân
bi t cho moi so nguyên to p. Giả sả f(0) =
/ 0. Với so nguyên m > k
có so nguyên to q > |f(0)| đe f(q) có ít nhat m > k ước so nguyên to
phân bi t theo Định lý 2.3, mâu thuan. Do v y, đieu giả sả là sai và ta
có f (0) = 0. Bieu dien f (x) = xr
g(x) với g(0) /= 0. Neu g(x) không l
à
hang so thì ta l p lại quá trình trên và suy ra mâu thuan. Tà đó suy ra
f(x) = axm
với m ∈ N và a ∈ N∗
.
Ví dn 2.2. Giả sủ đa thúc f (x) ∈ Z[x] nh¾n giá tr ±1 tại 3 điem nguyên
khác nhau. Chúng minh rang, f(x) không the có nghi m nguyên.
Bài giải. Giả sả có bon so nguyên phân bi t a, b, c, d đe f (a), f (b), f (c)
thu®c t p {−1, 1} và f (d) = 0. Khi đó a − d, b − d, c − d là ba so nguyên
phân bi t đôi m®t cùng là ước của 1, vô lý.
Ví dn 2.3. Cho dãy các so nguyên (an) với tính chat m − n|am − an với
moi m, n ∈ N và m n. Giả sủ có đa thúc p(x) đe |an| < p(n) với moi
n ∈ N. Chúng minh, có đa thúc q(x) thóa mãn an = q(n) với moi n.
37
Viết đề tài giá sinh viên – ZALO:0973.287.149-TEAMLUANVAN.COM
2 2 2 2
Bài giải. Đ t d = deg p(x). De dàng chỉ ra m®t đa thác q(x) với
deg q(x) ≤ d thỏa mãn q(k) = ak với moi k = 1, 2, . . . , d + 1. Ta sě
chỉ ra q(n) = an với moi n ∈ N∗
. Xét n > d + 1. Đa thác q(x) có the
không thu®c Z[x], nhưng chac chan thu®c Q[x]. Chon so nguyên M đe
g(x) = Mq(x) ∈ Z[x]. Tích
M(an − q(n)) = Man − Mq(n) = M(an − ak) − (g(n) − g(k))
chia het cho n − k theo giả thiet cho k = 1, 2, . . . , d + 1. Như v y, ho c
an = q(n) ho c b®i chung nhỏ nhat Ln của các so n−1, n−2, . . . , n−d−1
không vượt quá M (an − q(n)) hay Ln ≤ M (an − q(n)) < cnd
, trong đó
hang so c không phụ thu®c n.
Ví dn 2.4. Giả sủ đa thúc f(x) = x2018
+ x2017
+ 2
2019
= a + a1x +
a2x2
+ · · · + anxn
. Hãy tính tőng
T = a
a1 a2
— − + a
a4 a5
— − + a — · · · .
2π
Bài giải. Với α = cos
3
2π
+ i sin
3
/= 1 ta có α3
= 1, α2
+ α = −1.
Tính
1 = f(α) = a0 + a1α + a2α2
+ a3 + a4α + a5α2
+ a6 + · · ·
1 = f(α2
) = a0 + a1α2
+ a2α + a3 + a4α2
+ a5α + a6 + · · · .
V y 2 = 2a0 −a1 −a2 +2a3 −a4 −a5 +2a6 −· · · và suy ra tőng T = 1.
Ví dn 2.5. Giả sủ f (x) = x3
− 3x. Xây dựng dãy truy hoi qua vi c
đ¾t f1(x) = f (x) và fn+1(x) = f1(fn(x)) với n = 1, 2, 3, . . . Chúng minh
rang, phương trình đa thúc fn(x) = x có 3n
nghi m thực phân bi t.
Bài giải. Trước tiên ta chỉ ra fn(x) là đa thác với h so nguyên b c 3n
.
Vì f1(x) = x3
− 3x nên ket lu n đúng cho n = 1. Giả sả fn(x) là đa thác
với h so nguyên b c 3n
. Khi đó fn+1(x) = f1(fn(x)) = (fn(x))3
− 3fn(x)
là đa thác với h so nguyên b c 3n+1
và ket quả đã được cháng minh.
Xét x = x(t) = 2 cos t với 0 ≤ t ≤ π. Ta có −2 ≤ x ≤ 2. De dàng kiem
tra f (x) = 2 cos 3t và f2(x) = 2 cos 32
t. Qua quy nạp theo n, ta nh n
được ket quả fn(x) = 2 cos 3n
t.
0
0 3 6
38
Viết đề tài giá sinh viên – ZALO:0973.287.149-TEAMLUANVAN.COM
∈
−
Xét phương trình fn(x) = x với x = 2 cos t. Ta có cos 3n
t = cos t.
2kπ
Giải ra t =
3n + 1
2kπ
và t =
3n − 1
với k = 0, 1, 2, . . . , 3n
với chú ý
2.0.π
3n + 1
2.0.π
=
3n − 1
. Ta nh n được 3n
nghi m phân bi t.
Ví dn 2.6. Giả sủ đa thúc f(x) Z[x] thóa mãn f(f(. . . f(m) . . .)) = m
n lan f
với so nguyên m và so nguyên n ≥ 2. Chúng minh rang f(f(m)) = m.
Tù đó suy ra so nghi m nguyên của phương trình f(f(. . . f(x) . . .)) = x
2017 lan f
với f(x) = x2
+ 3x − 4 không lớn hơn 2.
Bài giải. Xét dãy (mk) với m0 = m và mk+1 = f(mk) với so nguyên
k ≥ 0. Giả sả mr = m0 = m. Theo Định lý 2.1 ta có dk = mk+1 −
mk|f(mk+1) − f(mk) = mk+2 − mk+1 = dk+1 cho moi so nguyên k.
Ket hợp với mr = m0 = m ta suy ra dr = d0 và nh n được |d0| = |d1| =
· · · = |dr|.
Giả sả d1 = d0 = d 0. Khi đó d2 = d. Tương tự d3 = d, . . .
V y mk = m0 + kd /= m0 cho moi k, mâu thuan. Tà đó suy ra d1 = −d0
và suy ra m2 = m0 = m.
Giả sả x = m là nghi m nguyên của phương trình f(f(. . . f(x) . . .)) = x.
2017 lan f
Khi đó m phải là nghi m nguyên của phương trình f(f(x)) = x. Ta có
x4
+ 6x3
+ 4x2
− 16x = 0. Phương trình có hai nghi m nguyên x = 0
và x = 4. V y, so nghi m nguyên của phương trình f(. . . f(x) . . .) = x
2017 lan f
với f(x) = x2
+ 3x − 4 không lớn hơn 2.
Ví dn 2.7. Cho so nguyên m và đa thúc f(x) = anxn
+ · · · + a1x + a0 ∈
R[x] thóa mãn đieu ki n f(x) là so nguyên chia het cho m khi x nh¾n
giá tr nguyên. Chúng minh rang, n!an chia het cho m.
1
Bài giải. Xét đa thác
1
f(x). Vì f(x) là so nguyên chia het cho m khi
m
x là so nguyên nên f(x) nh n giá trị nguyên khi x là so nguyên. Theo
m
39
Viết đề tài giá sinh viên – ZALO:0973.287.149-TEAMLUANVAN.COM
n 0
Định lý ??, có các so nguyên b0, b1, . . . , bn đe
1
f(x) viet được thành
m
1
f(x) = b0
m
x
+ b
x
n − 1
+ · · · + bn
x
.
H so cao nhat của đa thác này bang b0 + nb1 + · · · + n!bn. Do v y, n!an
chia het cho m.
Ví dn 2.8. Xác đ nh tat cả các so nguyên dương n đe sao tat cả các đa
thúc f (x) = (x − a1)(x − a2) . . . (x − an) + 1 là bat khả quy với moi n so
nguyên phân bi t đôi m®t a1, a2, . . . , an.
Bài giải. Với n = 1, đa thác f (x) = x − a + 1 luôn là bat khả quy. Với
n = 2, đa thác f (x) = (x − a)(x − a − 2) + 1 = (x − a − 1)2
là khả quy.
Với n = 4, đa thác f (x) = (x − a)(x − a − 1)(x − a − 2)(x − a − 3) + 1
hay f(x) = [(x − a − 1)(x − a − 2) − 1]2
là khả quy.
Xét n = 3 ho c n ≥ 5. Giả sả đa thác f(x) là khả quy trong Z[x], có
nghĩa: Có hai đa thác g(x), h(x) b c lớn hơn 0 và g(x), h(x) ∈ Z[x] đe
f(x) = g(x)h(x). Khi đó g(ai)h(ai) = 1 với moi i = 1, 2, . . . , n. Như v y,
g(ai) = h(ai) = ±1 với moi i = 1, 2, . . . , n. Do b c của g(x), h(x) đeu
lớn hơn 0 và bang nhau tại n điem phân bi t ai, trong đó
n > deg g(x), deg h(x), nên g(x) = h(x). Ta có f(x) = g(x)2
. Tà đây
suy ra n phải là so chȁn, n = 2m, và khi n là so lẻ thì f(x) là bat khả
quy. Viet
[g(x) + 1][g(x) − 1] = (x − a1)(x − a2) . . . (x − a2m), m ≥ 3.
Sap xep lại, neu can, ta có the viet
g(x) + 1 = (x − a1)(x − a3) . . . (x − a2m−1)
g(x) − 1 = (x − a2)(x − a4) . . . (x − a2m)
với a1 > a3 > · · · > a2m−1 và a2 > a4 > · · · > a2m. Trà hai ve, ta có
2 = (x − a1)(x − a3) . . . (x − a2m−1) − (x − a2)(x − a4) . . . (x − a2m). V y
2 = (a2k − a1)(a2k − a3) . . . (a2k − a2m−1).
1
40
Viết đề tài giá sinh viên – ZALO:0973.287.149-TEAMLUANVAN.COM
Σ
Σ
n
n
k
n
Σ
= c(n, k)
k
k
Ta có the coi a2 là so lớn nhat. Vì 2 = 1.2 ho c 2 = (−1)(−2) nên vi c
bieu dien 2 = (a2 − a1)(a2 − a3) . . . (a2 − a2m−1) bang tích m ≥ 3 thàa
so khác nhau là không the xảy ra. Đieu này chỉ ra đieu giả sả là sai.
Đa thác f(x) là bat khả quy. Tóm lại, với n = 1, n = 3, n ≥ 5, đa thác
f (x) = (x − a1)(x − a2) . . . (x − an) + 1 là bat khả quy với moi n so
nguyên phân bi t đôi m®t a1, a2, . . . , an.
Ví dn 2.9. Giả sủ xn
n > 2017.
n
=
k=0
n
c(n, k)
Σ
n
x
. Tính
x
c(n, 2017) + c(n, 2018)
khi
c(n + 1, 2018)
xn+1
=
n
k=0
Σ
c(n, k) x
x =
x
Σ
k=0
c(n, k)
Σ
x (x + 1 − 1)
x
=
k=0
c(n, k)(k + 1)
x + 1
k + 1
—
Σ
k=0
c(n, k)
x
. Do v y
xn+1
=
Σ
k=0
c(n, k)(k + 1)
x
k + 1
+
x i
−
Σ
k=0
c(n, k)
x
=
k=0
c(n, k − 1)k + (k + 1 − 1)c(n, k)
x
= k
k=0
c(n, k − 1) + c(n, k) x
.
Tà đây suy ra c(n + 1, k) = k c(n, k − 1) + c(n, k) . Ta nh n được
c(n, k − 1) + c(n, k)
c(n + 1, k) 1
= . Khi so nguyên n > 2017 ta nh n được ket
k
quả
c(n, 2017) + c(n, 2018)
=
c(n + 1, 2018)
1
.
2018
Ví dn 2.10. Cho so tự nhiên n > 1. Chúng minh rang, đa thúc thuan
nhat hai bien f(y, z) = yn
+ 5yn−1
z + 3zn
là bat khả quy trong Z[y, z].
k
n
n
k
k
n
k
k
k=0
k
k=0
k
n
k
k=0
n n
h
Bài giải. Với bieu dien x = c(n, k) ta có the bien đői tőng
(x + 1) − c(n, k)
Σ
Σ
41
Viết đề tài giá sinh viên – ZALO:0973.287.149-TEAMLUANVAN.COM
−
αi (αi +5) = = g( 5) . Tà g( 5)h( 5) = k(
i=1
Bài giải. Bieu dien f(y, z) = zn
xn
+ 5xn−1
+ 3 với x =
y
. Ta chỉ
z
can cháng minh k(x) = xn
+ 5xn−1
+ 3 là bat khả quy trong Z[x]. Với
n = 2, đa thác k(x) = x2
+ 5x + 3 không có nghi m trong Z. Đa thác
này là bat khả quy. Với n > 2, giả sả đa thác k(x) = g(x)h(x), trong
đó g(x), h(x) ∈ Z[x] và deg g(x), deg h(x) > 1 do k(x) không có nghi m
trong Z. Do g(0)h(0) = k(0) = 3 nên ho c |g(0)| = 1 ho c |h(0)| = 1,
chȁng hạn |g(0)| = 1. Bieu dien đa thác
g(x) = xs
+ a1xs−1
+ · · · + as, s > 1, |g(0)| = 1.
Goi α1, . . . , αs là nghi m của g(x) = 0, trong đó các nghi m có the là
Q
s
0 ta suy ra k(αi) = 0 với moi i = 1, 2, . . . , s. De dàng chỉ ra 3s
=
Q
s
|
n−1
|
Q
s
| | | − | − − −
nên |g(−5)| = 1 ho c |g(−5)| = 3. Do s > 1 nên |g(−5)| = 3s
là sai. V y
đieu giả sả k(x) khả quy là sai. Tà đó suy ra k(x) bat khả quy.
Ví dn 2.11. Cho so tự nhiên n > 1. Chúng minh, đa thúc thuan nhat
hai bien f(y, z) = y5
− yz4
+ nz5
là bat khả quy trong Z[y, z] khi 5 / |n.
Bài giải. Bieu dien f(y, z) = z5
x5
x+n với x =
y
. Ta chỉ can cháng
z
minh k(x) = x5
− x + n là bat khả quy trong Z[x] khi 5 / |n.
Giả sả k(x) = (x − m)h(x). Khi đó k(m) = 0 hay m5
− m = −n. Vì
m ∈ N∗
và 5 là so nguyên to nên m5
− m ≡ 0( mod 5) theo Định lý
nhỏ Fermat. Tà đây suy ra 5|n, mâu thuan với giả thiet.
Giả sả k(x) = (x2
−bx−c)h(x). Khi đó x5
−x+n chia het cho x2
−bx−c.
Thực hi n phép chia x5
− x + n cho x2
− bx − c ta nh n được thương
x3
+ bx2
+(b2
+ c)x + b3
+ 2bc và đa thác dư (b4
+ 3b2
c + c2
− 1)x +(b3
c +
2bc2
+ n). Do đa thác dư phải bang 0 nên
b4
+ 3b2
c + c2
− 1 = 0
b3
c + 2bc2
+ n = 0.
i=1
thực hay phác. Ta có |αi| = |g(0)| = 1. Tà k(αi) = g(αi)h(αi) =
i=1
(αi +5) 5) = 3
42
Viết đề tài giá sinh viên – ZALO:0973.287.149-TEAMLUANVAN.COM
− −
— −
z
−
— −
Σ t −
z |z|2
1
Suy ra b(b4
+ 3b2
c + c2
− 1) − 3(b3
c + 2bc2
+ n) = 0 hay b5
− b − 5bc2
= 3n.
Do b, c nguyên và b5
− b chia het cho 5 nên n˙:5, vô lý. V y, đieu giả sả
k(x) khả quy là sai. Tà đó suy ra k(x) bat khả quy.
Ví dn 2.12. Chúng minh rang, đa thúc f (x, y) = xn
− xyn−1
− yn
là bat
khả quy trong Q[x, y] cho moi so nguyên dương n.
Bài giải. Bieu dien f(x, y) = yn
tn
t 1] với t =
x
. Do v y, ta chỉ
y
can cháng minh g(t) = tn
t 1 là đa thác bat khả quy trong Q[t].
Nh n xét: Neu z là nghi m của tn
− t − 1 thì 2Re z −
1
>
1
− 1.
Th t v y, viet z = reiu
với r ≥ 0. Ta thay ngay r = 0 là không thỏa
mãn. Neu r = 1 thì cos nu = cos u + 1 và sin nu = sin u. Tà đó suy ra
1 = cos2
nu + sin2
nu = (1 + cos u)2
+ sin2
u.
Tà đây suy ra mâu thuan. Bat đȁng thác can xét sě trở thành (1 +
2r cos u)(r2
− 1) > 0. Do r2n
= |z|2n
= |z + 1|2
= 1 + 2r cos u + r2
nên
r2n
− r2
= 1 + 2r cos u, và ta nh n được (1 + 2r cos u)(r2
− 1) = (r2n
−
r2
)(r2
− 1) > 0. Ta có 2Re z − = 2r cos u. và 2Re z − >
1 r2
1 1
r2
1
1 tương đương (1 + 2r cos u)(r2
1) > 0, đúng.
|z|2
Giả sả F(t) = tn
− t − 1 = g(t)h(t) trong Q[t]. Theo Bő đe Gauss, ta
có the xét g(t), h(t) ∈ Z[t]. Hien nhiên tn
− t − 1 = 0 không có nghi m
trong Q. Ta có F ′(t) = ntn−1
− 1. Neu t là nghi m của tn
− t − 1 = 0
thì tn
= t + 1. Giả sả t cũng là nghi m của F ′(t) = ntn−1
− 1 = 0.
Khi đó n(t + 1) = t và suy ra t ∈ Q, vô lý. Do v y, tat cả các nghi m
của F (t) = 0 là khác nhau đôi m®t. Giả sả F(t) có n nghi m phác
phân bi t t1, . . . , tn và giả sả t1, . . . , ts là nghi m của g(t) = 0. Ta có
|t1| . . . |ts| = |g(0)| = 1. Viet g(t) = ts
+ a1ts−1
+ · · · + as−1t ± 1. Ta có
t1 +· · ·+ts = −a1 và
1 1
t
+· · ·+
t
= ±as−1. V y
s
j=1
1
j
t1
∈ Z. Theo
z
s
43
Viết đề tài giá sinh viên – ZALO:0973.287.149-TEAMLUANVAN.COM
‚
.Y
,
Σ
− −
Σ t −
tj −
t
— u = −1. V y tj −
t
Σ
Σ
j
nh n xét trên, qua bat đȁng thác Cauchy ta có the đánh giá tőng
2 Re
j=1
1
tj −
t
>
j=1
1
|tj|2
s
— s ≥ s s
j=1
1
|tj|2
— s = s − s = 0.
Vì các tj phân bi t nên dau = không xảy ra. Vì g(t) là đa thác monic
Σ
s 1
với h so nguyên nên
Σ
n
Re
j=1
tj −
t
1
≥ 1. Tương tự, l p lu n cho đa
Hien nhiên, với n = 2 đa thác t2
− t − 1 = 0 là bat khả quy. Xét n ≥ 3.
Vì t1 , . . . , tn là nghi m của tn
t 1 nên tj
j=1
1
= 0 và
tj
là nghi m của
n−1 n 1 Σ
n 1
thuan với ket quả
n
Re
j=1
1
j
tj
≥ 1 + 1 = 2. Tà đây suy ra, đieu giả
sả tn
− t − 1 khả quy là sai. V y, đa thác tn
− t − 1 là bat khả quy.
Ví dn 2.13. Cho so tự nhiên n > 1. Chúng minh rang, đa thúc thuan
nhat hai bien f(y, z) = (y2
+ z2
)n
+ pz2n
là bat khả quy trong Z[y, z] khi
p = 4k + 3 là so nguyên to với so nguyên không âm k.
Bài giải. Bieu dien f(y, z) = z2n
(x2
+ 1)n
+ p với x =
y
. Ta chỉ can
z
cháng minh k(x) = (x2
+1)n
+p là bat khả quy trong Z[x] khi p = 4k+3
là so nguyên to.
Giả sả k(x) = g(x)h(x) với đa thác g(x), h(x) ∈ Z[x] và deg g(x) ≥ 1,
deg h(x) ≥ 1. Ký hi u g(x), h(x) là đa thác g(x), h(x) xét trong Zp[x].
Khi đó g(x).h(x) = (x2
+ 1)n
. Đa thác x2
+ 1 là bat khả quy do −1
không là th ng dư b c hai trong Zp. V y, có so nguyên dương s đe
g(x) = (x2
+1)s
và h(x) = (x2
+1)n−s
, trong đó 1 ≤ s ≤ n−1. Tà đây suy
ra, có đa thác với h so nguyên g1(x) và h1(x) đe g(x) = (x2
+1)s
+pg1(x)
và h(x) = (x2
+ 1)n−s
+ ph1(x). Nhân đa thác ta có
(x2
+ 1)n
+ p = g(x)h(x) = [(x2
+ 1)s
+ pg1(x)][(x2
+ 1)n−s
+ ph1(x)]
j
j=1
j
t
j=1
j
s
s
thác h(t). Như v y
j=1
Re ≥ 1 + 1 = 2.
1 + u nên = 1. Đieu này mâu
j
Σ
44
Viết đề tài giá sinh viên – ZALO:0973.287.149-TEAMLUANVAN.COM
2
2
— −
hay (x2
+1)s
h1(x)+(x2
+1)n−s
g1(x)+pg1(x)h1(x) = 1. Lay theo modulo
p ta thay ngay đa thác x2
+ 1 là ước của 1 trong Zp[x]. Tà đieu mâu
thuan này suy ra rang, giả sả k(x) khả quy là sai. Tà đó suy ra k(x)
bat khả quy.
Định nghĩa 2.1. Hàm Z(n) = min
,
m|m ∈ N∗
, n|
m(m + 1),
được goi
là hàm pseudo Smarandache.
Ví dụ Z(1) = 1, Z(2) = 3, Z(3) = 2, Z(4) = 7, Z(5) = 4, Z(6) = 3,
Z(7) = 6, Z(8) = 15, Z(9) = 8, Z(10) = 4, Z(11) = 10, Z(12) = 8,
Z(13) = 12, Z(14) = 7, Z(15) = 5, Z(16) = 31, Z(17) = 16, Z(18) = 8,
Z(19) = 18, Z(20) = 15.
Bo đe 2.1. Với moi so nguyên lé n ≥ 3 ta có Z(n) ≤ n − 1 và với moi
so nguyên to p > 2 có Z(p) = p − 1.
Chfíng minh. Vì
(n − 1)n
2 chia het cho n nên Z(n) ≤ n − 1 với moi
so nguyên lẻ n ≥ 3. Vì p > 2 và p là so nguyên to nên p là so lẻ.
V y 1 + 2 + · · · + p − 1 =
p(p − 1)
là so nguyên dương chia het cho p.
V y Z(p) ≤ p 1. Kiem tra, với moi so nguyên dương k < p 1 có
k(k + 1)
1 + 2 + · · · + p − 1 = đeu không chia het cho so nguyên to p
2
vì k, k + 1 đeu không chia het cho p. Do v y Z(p) = p − 1 cho moi so
nguyên to p > 2.
Định lj 2.6. Với so dương M đủ lớn luôn có nhieu vô hạn so nguyên
dương n đe
|Z(n + 1) − Z(n)| > M và
Z(n + 1)
> M.
Z(n)
Chfíng minh. Cho so dương bat kỳ M đủ lớn, ta chon m ∈ N∗
đe
2m
> M. Vì (22m+1
, 2m
+ 1) = 1 nên có nhieu vô hạn so nguyên to dạng
22m+1
k + 2m
+ 1 theo Định lý 2.1. Như v y, có so nguyên dương k0 đe
p = 22m+1
k0 + 2m
+ 1 là m®t so nguyên to. Với so nguyên to p ta có
Z(p) = p − 1 = 22m+1
k0 + 2m
theo Bő đe 2.2. Bien đői
45
Viết đề tài giá sinh viên – ZALO:0973.287.149-TEAMLUANVAN.COM
n
n
n n!
Σ
2m+1k0
m+1 m+1
Z(p − 1) = Z 2m
(2m+1
k0 + 1) và
i
Σ
=1
i =
2 k0(2 k0 + 1)
.
2
2m+1k0
Hien nhiên 2m
(2m+1
k0 + 1) chia het 2m
k0(2m+1
k0 + 1) = i.
i=1
Tà đó suy ra Z(p − 1) ≤ 2m+1
k0. Với các đánh giá này ta nh n được ket
quả
Z(p)
Z(p − 1)
22m+1k0 + 2m
2m+1k0
> 2m
> M
|Z(p) − Z(p − 1)| ≥ Z(p) − Z(p − 1) = 22m+1
k0 + 2m
− 2m+1
k0
= 2m+1
k0(2m
− 1) + 2m
> 2m
> M.
Z(n + 1)
Đieu này chỉ ra |Z(n + 1) − Z(n)| và không bị ch n trên.
Z(n)
2.2 Đa thfíc Hilbert và bieu dien Mahler
Xét đa thúc
X
=
X(X − 1)(X − 2) · · · (X − n + 1)
,
được goi là các đa thúc Hilbert thú n. Các đa thúc này có các h so hũu
ty (không nhat thiet nguyên) và nó nh¾n các giá tr nguyên tại moi so
nguyên. Chú ý
x(x − 1) · · · (x − n + 1) = n!
x
neu x ≥ 1,
x(x − 1) · · · (x − n + 1) = (−1)n −x + n − 1
neu x < 0.
Các đa thác đó đóng m®t vai trò cơ bản trong lý thuyet các đa thác
nh n giá trị nguyên do ket quả sau đây của Polya:
Định lj 2.7. Giả sủ f là m®t đa thúc b¾c n với các h so thực. Khi
đó f (X) nh¾n giá tr nguyên neu và chí neu ton tại các so nguyên
a0, a1, a2, . . . , an sao cho
≥
46
Viết đề tài giá sinh viên – ZALO:0973.287.149-TEAMLUANVAN.COM
n
k
n
k
k
k
d
k
1
1 2 n
1 2 n
k
f(X) = a + a
X
+ a
X
+ · · · + a
X
.
Chfíng minh. Giả sả rang f(Z) ⊂ Z. Các đa thác X
, X
, · · · , X
có b c 0, 1, . . . , n, do đó chúng tạo thành m®t cơ sở của R-không gian
vectơ của các đa thác với h so lớn nhat là n. Do đó ton tại duy nhat
các so thực a0, a1, a2, . . . , an sao cho
f(X) =
Σ
k=0
ak ·
X
.
Xét toán tả ∆f(X) = f(X + 1) − f(X) và quan sát rang
∆
X
=
X
.
n n − 1
Tác đ®ng toán tả ∆ liên tiep vào quan h
f(X) =
Σ
k=0
ak ·
X
.
ta suy ra aj = ∆j
f(0) với moi j. Bang quy nạp toán hoc ta có
∆k
f(X) =
Σ
j=0
(−1)k−j
k
f(X + j).
Do đó, neu f(0), f(1), . . . , f(n) là các so nguyên, thì a0, a1, a2, . . . , an
cũng là các so nguyên.
Phép cháng minh của định lý trên chỉ ra rang
ak =
Σ
j=0
(−1)k−j
k
f(j).
Ta goi aj là các h so Mahler của f. Tà Định lý 2.7 ta có m nh đe sau.
M nh đe 2.1. Giả sủ d = deg f. Tù Đ nh lý 2.7 ta có the viet
f(X)
=
n
Σ
k=0
a
X
j
j
0 2 n
47
Viết đề tài giá sinh viên – ZALO:0973.287.149-TEAMLUANVAN.COM
·
·
D
i
d i
i i−1
i
phải cháng minh.
với các so nguyên ai nào đó. Xem xét các h so đau trong đȁng thúc này
cho thay d! là b®i của n. M¾t khác, neu d! là m®t b®i của n, thì ta có the
lay f (X) = X(X + 1) · · · (X + d − 1). Do đó, d là so nguyên nhó nhat
sao cho d! là m®t b®i của n.
2.3 V n dnng giải bài toán thi hoc sinh giỏi
Ví dn 2.14. [MOSP 2001] Giả sủ f là m®t đa thúc sao cho f(n) ∈ Z
với moi n ∈ Z. Chúng minh rang với bat kỳ các so nguyên m, n thì so
lcm[1, 2, . . . , deg(f)]
f(m) − f(n)
m − n
là nguyên.
Bài giải. Co định các so nguyên phân bi t m và n, giả sả d = m − n
và g(X) = f (n + X). Khi đó g có các h so hǎu t , gải các so nguyên
đen các so nguyên và deg(f) = deg(g). Do đó ta cháng minh rang
lcm[1, 2, . . . , deg(g)]
g(d) − g(0)
d
là nguyên. Giả sả D = deg g, do đó, sả dụng Định lý 2.8, ton tại các so
nguyên a0, a1, . . . , aD sao cho:
g(X) =
Σ
i=0
a
X
.
Nó cháng tỏ raơng với bat kỳ 1 ≤ i ≤ D ta có
lcm[1, 2, . . . , D] ·
1 d
∈ Z.
Nhưng ve trái bang
lcm[1, 2, . . . , D] d−1
, là m®t so nguyên. Suy ra đieu
Ví dn 2.15. [Holden Lee] Giả sủ f là m®t đa thúc b¾c d sao cho f (Z) ⊂
Z và với f (m)− f (n) là m®t b®i của m− n với moi 0 ≤ m, n ≤ d. Chúng
minh rang f(m) − f(n) là m®t b®i của m − n với moi so nguyên m, n
mà m /= n.
48
Viết đề tài giá sinh viên – ZALO:0973.287.149-TEAMLUANVAN.COM
d
k
d
k
i
k k k
d
k
k k
k
k
k
Σ
k
Bài giải. Do f(Z) ⊂ Z, Định lý 2.7 chỉ ra sự ton tại của các so nguyên
a0, a1, . . . , ad sao cho
f(X) =
Σ
k=0
a
X
.
Yêu cau bài toán được suy ra tà đây, Ví dụ 2.14 và ket quả tőng quát
dưới đây.
Bo đe 2.2. Giả sủ ai ∈ Z và
f(X) =
Σ
k=0
a
X
và Lk = lcm(1, 2, . . . , k)
(với quy ước L0 = 1). Khi đó các phát bieu sau đây là tương đương:
(a) m − n chia het f(m) − f(n) với moi 0 ≤ m /= n ≤ d = deg f.
(b) Lk chia het ak với moi 0 ≤ k ≤ d.
(c) m − n chia het f(m) − f(n) với moi m /= n ∈ Z.
Chfíng minh. Giả sả (a) xảy ra. Ta sě cháng minh bang quy nạp toán
hoc theo i rang Li chia het ai. Đieu này rõ ràng với i = 0, do đó giả sả
rang a0, a1, . . . , ai−1 là các b®i của L0, L1, . . . , Li−1 và co định 0 ≤ j < i.
Khi đó j − i chia het
f(i) − f(j) =
Σ
k=0
a
i
−
j
= ai + a
k
0≤k<i
i
−
j
.
Tà ví dụ 2.14 và giả thiet quy nạp, moi so trong các so ak
i
− j
với 0 ≤ k < i là m®t b®i của i − j. Suy ra i − j chia het ai và do đó
j < i với bat kỳ, suy ra Li chia het ai. Do v y (a) kéo theo (b). Ví dụ
2.14 chỉ ra rang (b) kéo theo (c) và do đó rat hien nhiên rang (c) kéo
theo (a). Phép cháng minh được ket thúc.
Nh n xét 2.1. Đa thúc
f(X) =
Σ
k=0
a
X
k
49
Viết đề tài giá sinh viên – ZALO:0973.287.149-TEAMLUANVAN.COM
n
i
≤ ≤
i i
thóa mãn
f(a) − f(b)
a − b
∈ Z với moi a /= b ∈ Z neu và chí neu
lcm(1, 2, . . . , k) | ak với moi k .
Bài toán sau đây gợi ý sả dụng Định lý n®i suy Lagrange, nhưng có m®t
so khó khăn trong vi c l p lu n, do các đa thác xuat hi n trong công
thác Lagrange không có h so nguyên. Bài toán là khá phác tạp và sả
dụng lại các đa thác Hilbert.
Ví dn 2.16. [Chinese TST 2004] (a) Chúng minh rang với moi so
nguyên dương n ton tại m®t đa thúc f ∈ Z[X] sao cho moi so f (1) <
f(2) < . . . < f(n) là lũy thùa của 2.
(b) Giả sủ a > 1 là m®t so nguyên và n nguyên dương. Chúng minh rang
ton tại m®t đa thúc f b¾c n có h so nguyên, sao cho f(0), f(1), . . . , f(n)
là các c¾p phân bi t nguyên dương, tat cả có dạng 2ak
+ 3 với m®t so
nguyên k nào đó.
Bài giải. (a) Ta sě chon f có dạng
f(X) = A.
Σ
i=0
X
Bi
với các so nguyên A, B phù hợp, sě được chon sau. Tà công thác nhị
thác, với bat kỳ 0 i n ta có f (i) = A(1 + B)i
. Ta can f có h so
nguyên nhưng X
không có các h so nguyên. Tuy nhiên, n! X
có h
so nguyên với moi 1 ≤ i ≤ n. Do A(1 + B)i
không phải là m®t lũy thàa
của 2 nên ta không the lay cho B m®t b®i của n!. Ta có the t n dụng
sự xuat hi n của A: chon A là 2v2(n!)
và B là m®t b®i lẻ của ước lẻ lớn
nhat của n!. Khi đó các chú ý trên cháng tỏ rang f có h so nguyên.
Cuoi cùng, ta muon 1 + B là m®t lũy thàa của 2. Do đó ta có the chon
(chȁng hạn) 1 + B = 2ϕ(d)
, trong đó d là ước lẻ lớn nhat của n!. Với
nhǎng lựa chon đó, f (1) < f (2) < . . . < f (n) là lũy thàa của 2, theo
yêu cau bài toán.
(b) Sả dụng ý tưởng như cháng minh (a). Ta viet n! = m · q, với moi
nhân tả nguyên to của m là m®t trong so a và trong đó gcd(q, a) = 1.
50
Viết đề tài giá sinh viên – ZALO:0973.287.149-TEAMLUANVAN.COM
n
Giả sả b = aϕ(q)
− 1, do đó q chia het b. Cuoi cùng, định nghĩa:
f(X) = 2am
Σ
i=0
X
bi
+ 3.
Nó có h so nguyên bởi vì i! | n! | am
· b với moi 0 ≤ i ≤ n. Hơn nǎa, với
1 ≤ k ≤ n ta có
P(i) = 2am
· (b + 1)i
+ 3 = 2am+ϕ(q)i
+ 3.
Nh n xét 2.2. Với moi so nguyên dương n có m®t đa thúc f ∈ Z[X]
sao cho với moi so f(1) < f(2) < . . . < f(n) là nguyên to.
Ta sě tìm m®t đa thác như v y dưới dạng:
f(X) = a1(X − 2)(X − 3) · · · (X − n) + a2(X − 1)(X − 3) · · · (X − n)+
+ . . . + an(X − 1) · · · (X − n + 1) + 1,
với các so nguyên ai thích hợp nào đó. Mà f(i) chỉ phụ thu®c vào ai, do
đó ta có the chon ai đe f(i) là nguyên to. Chú ý rang:
f(i) = (−1)n−i
(n − i)!(i − 1)!ai + 1.
bây giờ ta sě chon ai m®t cách quy nạp sao cho f(1) < . . . < f(n) là
các so nguyên to. Chúng tôi sě sả dụng Định lý 9.6 trong tài li u tham
khảo so [2], theo đó với moi n ton tại vô so so nguyên to p ≡ 1(modn),
các so nguyên to này có dạng 1 + kn. Do đó, ton tại m®t so nguyên a1
sao cho 1 + (−1)n−1
(n − 1)!a1 = f (1) là m®t so nguyên to. Co định a1
như v y và chon (m®t lan nǎa bởi ket quả đã trích dan) m®t so nguyên
a2 sao cho 1 + a2(n − 2)!(−1)n−2
= f (2) nguyên to lớn hơn f (1). Tiep
tục như v y, ta tìm a1, a2, . . . , an sao cho f (1) < f (2) < . . . < f (n) là
các so nguyên to và bài toán được giải quyet.
i
51
Viết đề tài giá sinh viên – ZALO:0973.287.149-TEAMLUANVAN.COM
Ket lu n
Trong lu n văn "Định lý Zsigmondy và tính chat so hoc của đa
thác" chúng tôi đã trình bày được nhǎng van đe sau:
1. Đa thác và so phác.
2. Đa thác chia đường tròn
3. Định lý Zsigmondy và v n dụng.
4. Tính chat đ c bi t của đa thác thu®c Z[x].
5. Đa thác Hilber bieu dien Mahler.
Lu n văn có the làm tài li u tham khảo cho giáo viên và hoc
sinh trong công tác boi dương hoc sinh giỏi
52
Viết đề tài giá sinh viên – ZALO:0973.287.149-TEAMLUANVAN.COM
Tài li u tham khảo
Tieng Vi t
[1] Đàm Văn Nhỉ, Văn Đác Chín, Tran Thị Hong Dung, Lê Xuân Dũng,
Đào Ngoc Dũng, Đ ng Xuân Sơn, Tran Trung Tình, Nguyen Anh
Tuan (2017), Đa thúc-Chuői lũy thùa và chuyên đe nâng cao, NXB
thông tin và truyen thông .
Tieng Anh
[2] T. Andreescu and G. Dospinescu (2010), Straight from the Book,
XYZ Press .
[3] Lawrence Sun (February 17, 2013), Cyclotomic Polynomials in
Olympiad Number Theory, lala-sun@hotmail.com.

More Related Content

Similar to Định lý zsigmondy và Tính chất số học của đa thức.docx

Hàm Đơn Đi U, Tựa Đơn Đi U Và M T So Ứng Dụng Của Phép Đơn Đi U Hóa Hàm So.docx
Hàm Đơn Đi U, Tựa Đơn Đi U Và M T So Ứng Dụng Của Phép Đơn Đi U Hóa Hàm So.docxHàm Đơn Đi U, Tựa Đơn Đi U Và M T So Ứng Dụng Của Phép Đơn Đi U Hóa Hàm So.docx
Hàm Đơn Đi U, Tựa Đơn Đi U Và M T So Ứng Dụng Của Phép Đơn Đi U Hóa Hàm So.docx
DV Viết Luận văn luanvanmaster.com ZALO 0973287149
 
Bài Toán Cực Trị Với Điều Kiện Ràng Buộc Bất Đẳng Thức, Hệ Bất Đẳng Thức.docx
Bài Toán Cực Trị Với Điều Kiện Ràng Buộc Bất Đẳng Thức, Hệ Bất Đẳng Thức.docxBài Toán Cực Trị Với Điều Kiện Ràng Buộc Bất Đẳng Thức, Hệ Bất Đẳng Thức.docx
Bài Toán Cực Trị Với Điều Kiện Ràng Buộc Bất Đẳng Thức, Hệ Bất Đẳng Thức.docx
DV Viết Luận văn luanvanmaster.com ZALO 0973287149
 
Hàm Đơn Đi›U, Tựa Đơn Đi›U Và Một Số Ứng Dụng Của Phép Đơn Đi›U Hóa Hàm Số.docx
Hàm Đơn Đi›U, Tựa Đơn Đi›U Và Một Số Ứng Dụng Của Phép Đơn Đi›U Hóa Hàm Số.docxHàm Đơn Đi›U, Tựa Đơn Đi›U Và Một Số Ứng Dụng Của Phép Đơn Đi›U Hóa Hàm Số.docx
Hàm Đơn Đi›U, Tựa Đơn Đi›U Và Một Số Ứng Dụng Của Phép Đơn Đi›U Hóa Hàm Số.docx
DV Viết Luận văn luanvanmaster.com ZALO 0973287149
 
Một số dạng toán về đa thức qua các kỳ thi Olympic 6732069.pdf
Một số dạng toán về đa thức qua các kỳ thi Olympic 6732069.pdfMột số dạng toán về đa thức qua các kỳ thi Olympic 6732069.pdf
Một số dạng toán về đa thức qua các kỳ thi Olympic 6732069.pdf
TieuNgocLy
 
M T So Dạng Toán Cực Tr± Trong L P Hàm Mũ Và Hàm Hyperbolic.docx
M T So Dạng Toán Cực Tr± Trong L P Hàm Mũ Và Hàm Hyperbolic.docxM T So Dạng Toán Cực Tr± Trong L P Hàm Mũ Và Hàm Hyperbolic.docx
M T So Dạng Toán Cực Tr± Trong L P Hàm Mũ Và Hàm Hyperbolic.docx
DV Viết Luận văn luanvanmaster.com ZALO 0973287149
 
Ve H Phương Trình Phi Tuyen Và Ứng Dụng.docx
Ve H Phương Trình Phi Tuyen Và Ứng Dụng.docxVe H Phương Trình Phi Tuyen Và Ứng Dụng.docx
Ve H Phương Trình Phi Tuyen Và Ứng Dụng.docx
DV Viết Luận văn luanvanmaster.com ZALO 0973287149
 
Bat Đang Thức V I Hàm Loi B Ph N Và Ứng Dụng.docx
Bat Đang Thức V I Hàm Loi B Ph N Và Ứng Dụng.docxBat Đang Thức V I Hàm Loi B Ph N Và Ứng Dụng.docx
Bat Đang Thức V I Hàm Loi B Ph N Và Ứng Dụng.docx
DV Viết Luận văn luanvanmaster.com ZALO 0973287149
 
Một số phương pháp giải các đề thi olympic Về phương trình diophant.docx
Một số phương pháp giải các đề thi olympic Về phương trình diophant.docxMột số phương pháp giải các đề thi olympic Về phương trình diophant.docx
Một số phương pháp giải các đề thi olympic Về phương trình diophant.docx
DV Viết Luận văn luanvanmaster.com ZALO 0973287149
 
Tich phan %28 nguyen duy khoi%29
Tich phan %28 nguyen duy khoi%29Tich phan %28 nguyen duy khoi%29
Tich phan %28 nguyen duy khoi%29trongphuckhtn
 
Tich phan (nguyen duy khoi)
Tich phan (nguyen duy khoi)Tich phan (nguyen duy khoi)
Tich phan (nguyen duy khoi)roggerbob
 
Tổng quát về tích phân
Tổng quát về tích phân Tổng quát về tích phân
Tổng quát về tích phân
Hoàng Hải Huy
 
Một Số Dạng Toán Cực Trị Trong Lîp Hàm Mũ Và Hàm Hyperbolic.docx
Một Số Dạng Toán Cực Trị Trong Lîp Hàm Mũ Và Hàm Hyperbolic.docxMột Số Dạng Toán Cực Trị Trong Lîp Hàm Mũ Và Hàm Hyperbolic.docx
Một Số Dạng Toán Cực Trị Trong Lîp Hàm Mũ Và Hàm Hyperbolic.docx
DV Viết Luận văn luanvanmaster.com ZALO 0973287149
 
Luận văn: Điều kiện cực trị và chính quy của nhân tử Lagrange, 9đ
Luận văn: Điều kiện cực trị và chính quy của nhân tử Lagrange, 9đLuận văn: Điều kiện cực trị và chính quy của nhân tử Lagrange, 9đ
Luận văn: Điều kiện cực trị và chính quy của nhân tử Lagrange, 9đ
Dịch Vụ Viết Bài Trọn Gói ZALO 0917193864
 
Toán Tử Sai Phân Và Ứng Dụng Vào Giải Toán Sơ Cấp.docx
Toán Tử Sai Phân Và Ứng Dụng Vào Giải Toán Sơ Cấp.docxToán Tử Sai Phân Và Ứng Dụng Vào Giải Toán Sơ Cấp.docx
Toán Tử Sai Phân Và Ứng Dụng Vào Giải Toán Sơ Cấp.docx
DV Viết Luận văn luanvanmaster.com ZALO 0973287149
 
108 bai toan chon loc lop 7
108 bai toan chon loc lop 7108 bai toan chon loc lop 7
108 bai toan chon loc lop 7
leroben
 
Luận văn: Định lý bézout và chiều ngược lại, HAY, 9đ
Luận văn: Định lý bézout và chiều ngược lại, HAY, 9đLuận văn: Định lý bézout và chiều ngược lại, HAY, 9đ
Luận văn: Định lý bézout và chiều ngược lại, HAY, 9đ
Dịch vụ viết bài trọn gói ZALO 0917193864
 
Luận văn: Bất đẳng thức trong lớp hàm siêu việt, HAY, 9đ
Luận văn: Bất đẳng thức trong lớp hàm siêu việt, HAY, 9đLuận văn: Bất đẳng thức trong lớp hàm siêu việt, HAY, 9đ
Luận văn: Bất đẳng thức trong lớp hàm siêu việt, HAY, 9đ
Dịch Vụ Viết Bài Trọn Gói ZALO 0917193864
 
Về Tổng Gauss Và Một Số Ứng Dụng.docx
Về Tổng Gauss Và Một Số Ứng Dụng.docxVề Tổng Gauss Và Một Số Ứng Dụng.docx
Về Tổng Gauss Và Một Số Ứng Dụng.docx
DV Viết Luận văn luanvanmaster.com ZALO 0973287149
 
Ôn tập phương trình nghiệm nguyên trong toán THCS ôn thi vào lớp 10
Ôn tập phương trình nghiệm nguyên trong toán THCS ôn thi vào lớp 10Ôn tập phương trình nghiệm nguyên trong toán THCS ôn thi vào lớp 10
Ôn tập phương trình nghiệm nguyên trong toán THCS ôn thi vào lớp 10
Gia sư môn Toán tại nhà Hà Nội Chất Lượng Cao
 
Phương trình diophantine dạng X2 − dy2 = ±4.docx
Phương trình diophantine dạng X2 − dy2 = ±4.docxPhương trình diophantine dạng X2 − dy2 = ±4.docx
Phương trình diophantine dạng X2 − dy2 = ±4.docx
DV Viết Luận văn luanvanmaster.com ZALO 0973287149
 

Similar to Định lý zsigmondy và Tính chất số học của đa thức.docx (20)

Hàm Đơn Đi U, Tựa Đơn Đi U Và M T So Ứng Dụng Của Phép Đơn Đi U Hóa Hàm So.docx
Hàm Đơn Đi U, Tựa Đơn Đi U Và M T So Ứng Dụng Của Phép Đơn Đi U Hóa Hàm So.docxHàm Đơn Đi U, Tựa Đơn Đi U Và M T So Ứng Dụng Của Phép Đơn Đi U Hóa Hàm So.docx
Hàm Đơn Đi U, Tựa Đơn Đi U Và M T So Ứng Dụng Của Phép Đơn Đi U Hóa Hàm So.docx
 
Bài Toán Cực Trị Với Điều Kiện Ràng Buộc Bất Đẳng Thức, Hệ Bất Đẳng Thức.docx
Bài Toán Cực Trị Với Điều Kiện Ràng Buộc Bất Đẳng Thức, Hệ Bất Đẳng Thức.docxBài Toán Cực Trị Với Điều Kiện Ràng Buộc Bất Đẳng Thức, Hệ Bất Đẳng Thức.docx
Bài Toán Cực Trị Với Điều Kiện Ràng Buộc Bất Đẳng Thức, Hệ Bất Đẳng Thức.docx
 
Hàm Đơn Đi›U, Tựa Đơn Đi›U Và Một Số Ứng Dụng Của Phép Đơn Đi›U Hóa Hàm Số.docx
Hàm Đơn Đi›U, Tựa Đơn Đi›U Và Một Số Ứng Dụng Của Phép Đơn Đi›U Hóa Hàm Số.docxHàm Đơn Đi›U, Tựa Đơn Đi›U Và Một Số Ứng Dụng Của Phép Đơn Đi›U Hóa Hàm Số.docx
Hàm Đơn Đi›U, Tựa Đơn Đi›U Và Một Số Ứng Dụng Của Phép Đơn Đi›U Hóa Hàm Số.docx
 
Một số dạng toán về đa thức qua các kỳ thi Olympic 6732069.pdf
Một số dạng toán về đa thức qua các kỳ thi Olympic 6732069.pdfMột số dạng toán về đa thức qua các kỳ thi Olympic 6732069.pdf
Một số dạng toán về đa thức qua các kỳ thi Olympic 6732069.pdf
 
M T So Dạng Toán Cực Tr± Trong L P Hàm Mũ Và Hàm Hyperbolic.docx
M T So Dạng Toán Cực Tr± Trong L P Hàm Mũ Và Hàm Hyperbolic.docxM T So Dạng Toán Cực Tr± Trong L P Hàm Mũ Và Hàm Hyperbolic.docx
M T So Dạng Toán Cực Tr± Trong L P Hàm Mũ Và Hàm Hyperbolic.docx
 
Ve H Phương Trình Phi Tuyen Và Ứng Dụng.docx
Ve H Phương Trình Phi Tuyen Và Ứng Dụng.docxVe H Phương Trình Phi Tuyen Và Ứng Dụng.docx
Ve H Phương Trình Phi Tuyen Và Ứng Dụng.docx
 
Bat Đang Thức V I Hàm Loi B Ph N Và Ứng Dụng.docx
Bat Đang Thức V I Hàm Loi B Ph N Và Ứng Dụng.docxBat Đang Thức V I Hàm Loi B Ph N Và Ứng Dụng.docx
Bat Đang Thức V I Hàm Loi B Ph N Và Ứng Dụng.docx
 
Một số phương pháp giải các đề thi olympic Về phương trình diophant.docx
Một số phương pháp giải các đề thi olympic Về phương trình diophant.docxMột số phương pháp giải các đề thi olympic Về phương trình diophant.docx
Một số phương pháp giải các đề thi olympic Về phương trình diophant.docx
 
Tich phan %28 nguyen duy khoi%29
Tich phan %28 nguyen duy khoi%29Tich phan %28 nguyen duy khoi%29
Tich phan %28 nguyen duy khoi%29
 
Tich phan (nguyen duy khoi)
Tich phan (nguyen duy khoi)Tich phan (nguyen duy khoi)
Tich phan (nguyen duy khoi)
 
Tổng quát về tích phân
Tổng quát về tích phân Tổng quát về tích phân
Tổng quát về tích phân
 
Một Số Dạng Toán Cực Trị Trong Lîp Hàm Mũ Và Hàm Hyperbolic.docx
Một Số Dạng Toán Cực Trị Trong Lîp Hàm Mũ Và Hàm Hyperbolic.docxMột Số Dạng Toán Cực Trị Trong Lîp Hàm Mũ Và Hàm Hyperbolic.docx
Một Số Dạng Toán Cực Trị Trong Lîp Hàm Mũ Và Hàm Hyperbolic.docx
 
Luận văn: Điều kiện cực trị và chính quy của nhân tử Lagrange, 9đ
Luận văn: Điều kiện cực trị và chính quy của nhân tử Lagrange, 9đLuận văn: Điều kiện cực trị và chính quy của nhân tử Lagrange, 9đ
Luận văn: Điều kiện cực trị và chính quy của nhân tử Lagrange, 9đ
 
Toán Tử Sai Phân Và Ứng Dụng Vào Giải Toán Sơ Cấp.docx
Toán Tử Sai Phân Và Ứng Dụng Vào Giải Toán Sơ Cấp.docxToán Tử Sai Phân Và Ứng Dụng Vào Giải Toán Sơ Cấp.docx
Toán Tử Sai Phân Và Ứng Dụng Vào Giải Toán Sơ Cấp.docx
 
108 bai toan chon loc lop 7
108 bai toan chon loc lop 7108 bai toan chon loc lop 7
108 bai toan chon loc lop 7
 
Luận văn: Định lý bézout và chiều ngược lại, HAY, 9đ
Luận văn: Định lý bézout và chiều ngược lại, HAY, 9đLuận văn: Định lý bézout và chiều ngược lại, HAY, 9đ
Luận văn: Định lý bézout và chiều ngược lại, HAY, 9đ
 
Luận văn: Bất đẳng thức trong lớp hàm siêu việt, HAY, 9đ
Luận văn: Bất đẳng thức trong lớp hàm siêu việt, HAY, 9đLuận văn: Bất đẳng thức trong lớp hàm siêu việt, HAY, 9đ
Luận văn: Bất đẳng thức trong lớp hàm siêu việt, HAY, 9đ
 
Về Tổng Gauss Và Một Số Ứng Dụng.docx
Về Tổng Gauss Và Một Số Ứng Dụng.docxVề Tổng Gauss Và Một Số Ứng Dụng.docx
Về Tổng Gauss Và Một Số Ứng Dụng.docx
 
Ôn tập phương trình nghiệm nguyên trong toán THCS ôn thi vào lớp 10
Ôn tập phương trình nghiệm nguyên trong toán THCS ôn thi vào lớp 10Ôn tập phương trình nghiệm nguyên trong toán THCS ôn thi vào lớp 10
Ôn tập phương trình nghiệm nguyên trong toán THCS ôn thi vào lớp 10
 
Phương trình diophantine dạng X2 − dy2 = ±4.docx
Phương trình diophantine dạng X2 − dy2 = ±4.docxPhương trình diophantine dạng X2 − dy2 = ±4.docx
Phương trình diophantine dạng X2 − dy2 = ±4.docx
 

More from DV Viết Luận văn luanvanmaster.com ZALO 0973287149

Ảnh Hưởng Của Marketing Quan Hệ Đến Lòng Trung Thành Của Khách Hàng.Tình Huốn...
Ảnh Hưởng Của Marketing Quan Hệ Đến Lòng Trung Thành Của Khách Hàng.Tình Huốn...Ảnh Hưởng Của Marketing Quan Hệ Đến Lòng Trung Thành Của Khách Hàng.Tình Huốn...
Ảnh Hưởng Của Marketing Quan Hệ Đến Lòng Trung Thành Của Khách Hàng.Tình Huốn...
DV Viết Luận văn luanvanmaster.com ZALO 0973287149
 
Phát triển nguồn nhân lực tại Uỷ ban nhân dân huyện Trà Bồng, tỉnh Quảng Ngãi...
Phát triển nguồn nhân lực tại Uỷ ban nhân dân huyện Trà Bồng, tỉnh Quảng Ngãi...Phát triển nguồn nhân lực tại Uỷ ban nhân dân huyện Trà Bồng, tỉnh Quảng Ngãi...
Phát triển nguồn nhân lực tại Uỷ ban nhân dân huyện Trà Bồng, tỉnh Quảng Ngãi...
DV Viết Luận văn luanvanmaster.com ZALO 0973287149
 
Báo cáo tốt Nghiệp tài chính hợp nhất tại tổng công ty Indochina gol...
Báo cáo tốt Nghiệp  tài chính hợp nhất tại tổng công ty Indochina gol...Báo cáo tốt Nghiệp  tài chính hợp nhất tại tổng công ty Indochina gol...
Báo cáo tốt Nghiệp tài chính hợp nhất tại tổng công ty Indochina gol...
DV Viết Luận văn luanvanmaster.com ZALO 0973287149
 
Tạo động lực thúc đẩy nhân viên làm việc tại ngân hàng TMCP Ngoại Thương Việt...
Tạo động lực thúc đẩy nhân viên làm việc tại ngân hàng TMCP Ngoại Thương Việt...Tạo động lực thúc đẩy nhân viên làm việc tại ngân hàng TMCP Ngoại Thương Việt...
Tạo động lực thúc đẩy nhân viên làm việc tại ngân hàng TMCP Ngoại Thương Việt...
DV Viết Luận văn luanvanmaster.com ZALO 0973287149
 
Phát triển công nghiệp trên địa bàn Thành phố Tam Kỳ, Tỉnh Quảng Na...
Phát triển công nghiệp trên địa bàn Thành phố Tam Kỳ, Tỉnh Quảng Na...Phát triển công nghiệp trên địa bàn Thành phố Tam Kỳ, Tỉnh Quảng Na...
Phát triển công nghiệp trên địa bàn Thành phố Tam Kỳ, Tỉnh Quảng Na...
DV Viết Luận văn luanvanmaster.com ZALO 0973287149
 
Giải pháp phát triển cho vay xuất nhập khẩu tại ngân hàng NN&PTNN ch...
Giải pháp phát triển cho vay xuất nhập khẩu tại ngân hàng NN&PTNN ch...Giải pháp phát triển cho vay xuất nhập khẩu tại ngân hàng NN&PTNN ch...
Giải pháp phát triển cho vay xuất nhập khẩu tại ngân hàng NN&PTNN ch...
DV Viết Luận văn luanvanmaster.com ZALO 0973287149
 
Hoàn thiện công tác lập báo cáo tài chính hợp nhất tại tổng công ...
Hoàn thiện công tác lập báo cáo tài chính hợp nhất tại tổng công ...Hoàn thiện công tác lập báo cáo tài chính hợp nhất tại tổng công ...
Hoàn thiện công tác lập báo cáo tài chính hợp nhất tại tổng công ...
DV Viết Luận văn luanvanmaster.com ZALO 0973287149
 
Luận Văn Thạc Sĩ Quản trị thành tích nhân viên tại Cục Hải quan TP Đà Nẵng.doc
Luận Văn Thạc Sĩ  Quản trị thành tích nhân viên tại Cục Hải quan TP Đà Nẵng.docLuận Văn Thạc Sĩ  Quản trị thành tích nhân viên tại Cục Hải quan TP Đà Nẵng.doc
Luận Văn Thạc Sĩ Quản trị thành tích nhân viên tại Cục Hải quan TP Đà Nẵng.doc
DV Viết Luận văn luanvanmaster.com ZALO 0973287149
 
Hoàn thiện công tác quản lý thuế thu nhập cá nhân tại cục thuế Tỉ...
Hoàn thiện công tác quản lý thuế thu nhập cá nhân tại cục thuế Tỉ...Hoàn thiện công tác quản lý thuế thu nhập cá nhân tại cục thuế Tỉ...
Hoàn thiện công tác quản lý thuế thu nhập cá nhân tại cục thuế Tỉ...
DV Viết Luận văn luanvanmaster.com ZALO 0973287149
 
Đề Tài Phát triển bền vững nông nghiệp Huyện Ba Tơ, Tỉnh Quảng Ngãi....
Đề Tài Phát triển bền vững nông nghiệp Huyện Ba Tơ, Tỉnh Quảng Ngãi....Đề Tài Phát triển bền vững nông nghiệp Huyện Ba Tơ, Tỉnh Quảng Ngãi....
Đề Tài Phát triển bền vững nông nghiệp Huyện Ba Tơ, Tỉnh Quảng Ngãi....
DV Viết Luận văn luanvanmaster.com ZALO 0973287149
 
Hoàn thiện công tác bảo trợ xã hội trên địa bàn huyện Phong Điền, tỉnh Thừa T...
Hoàn thiện công tác bảo trợ xã hội trên địa bàn huyện Phong Điền, tỉnh Thừa T...Hoàn thiện công tác bảo trợ xã hội trên địa bàn huyện Phong Điền, tỉnh Thừa T...
Hoàn thiện công tác bảo trợ xã hội trên địa bàn huyện Phong Điền, tỉnh Thừa T...
DV Viết Luận văn luanvanmaster.com ZALO 0973287149
 
Đề Tài Luận VănPhát triển sản phẩm du lịch tại thành phố Đà Nẵng.doc
Đề Tài Luận VănPhát triển sản phẩm du lịch tại thành phố Đà Nẵng.docĐề Tài Luận VănPhát triển sản phẩm du lịch tại thành phố Đà Nẵng.doc
Đề Tài Luận VănPhát triển sản phẩm du lịch tại thành phố Đà Nẵng.doc
DV Viết Luận văn luanvanmaster.com ZALO 0973287149
 
Đào tạo nghề cho lao động thuộc diện thu hồi đất trên địa bàn Thàn...
Đào tạo nghề cho lao động thuộc diện thu hồi đất trên địa bàn Thàn...Đào tạo nghề cho lao động thuộc diện thu hồi đất trên địa bàn Thàn...
Đào tạo nghề cho lao động thuộc diện thu hồi đất trên địa bàn Thàn...
DV Viết Luận văn luanvanmaster.com ZALO 0973287149
 
Tóm Tắt Luận Văn Thạc Sĩ Quản Trị Kinh Doanh Xây dựng chính sách Marketing tạ...
Tóm Tắt Luận Văn Thạc Sĩ Quản Trị Kinh Doanh Xây dựng chính sách Marketing tạ...Tóm Tắt Luận Văn Thạc Sĩ Quản Trị Kinh Doanh Xây dựng chính sách Marketing tạ...
Tóm Tắt Luận Văn Thạc Sĩ Quản Trị Kinh Doanh Xây dựng chính sách Marketing tạ...
DV Viết Luận văn luanvanmaster.com ZALO 0973287149
 
Đề Tài Nghiên cứu rủi ro cảm nhận đối với mua hàng thời trang trực tuyến.docx
Đề Tài Nghiên cứu rủi ro cảm nhận đối với mua hàng thời trang trực tuyến.docxĐề Tài Nghiên cứu rủi ro cảm nhận đối với mua hàng thời trang trực tuyến.docx
Đề Tài Nghiên cứu rủi ro cảm nhận đối với mua hàng thời trang trực tuyến.docx
DV Viết Luận văn luanvanmaster.com ZALO 0973287149
 
Giải pháp nâng cao động lực thúc đẩy người lao động tại công ty khai...
Giải pháp nâng cao động lực thúc đẩy người lao động tại công ty khai...Giải pháp nâng cao động lực thúc đẩy người lao động tại công ty khai...
Giải pháp nâng cao động lực thúc đẩy người lao động tại công ty khai...
DV Viết Luận văn luanvanmaster.com ZALO 0973287149
 
Giải pháp phát triển dịch vụ ngân hàng điện tử tại ngân hàng đầu ...
Giải pháp phát triển dịch vụ ngân hàng điện tử tại ngân hàng đầu ...Giải pháp phát triển dịch vụ ngân hàng điện tử tại ngân hàng đầu ...
Giải pháp phát triển dịch vụ ngân hàng điện tử tại ngân hàng đầu ...
DV Viết Luận văn luanvanmaster.com ZALO 0973287149
 
Giải pháp phát triển dịch vụ ngân hàng điện tử tại ngân hàng đầu ...
Giải pháp phát triển dịch vụ ngân hàng điện tử tại ngân hàng đầu ...Giải pháp phát triển dịch vụ ngân hàng điện tử tại ngân hàng đầu ...
Giải pháp phát triển dịch vụ ngân hàng điện tử tại ngân hàng đầu ...
DV Viết Luận văn luanvanmaster.com ZALO 0973287149
 
Quản trị quan hệ khách hàng tại Chi nhánh Viettel Đà Nẵng – Tập đoàn Viễn thô...
Quản trị quan hệ khách hàng tại Chi nhánh Viettel Đà Nẵng – Tập đoàn Viễn thô...Quản trị quan hệ khách hàng tại Chi nhánh Viettel Đà Nẵng – Tập đoàn Viễn thô...
Quản trị quan hệ khách hàng tại Chi nhánh Viettel Đà Nẵng – Tập đoàn Viễn thô...
DV Viết Luận văn luanvanmaster.com ZALO 0973287149
 
Đề Tài Đánh giá thành tích đội ngũ giảng viên trường Đại Học Phạm ...
Đề Tài Đánh giá thành tích đội ngũ giảng viên trường Đại Học Phạm ...Đề Tài Đánh giá thành tích đội ngũ giảng viên trường Đại Học Phạm ...
Đề Tài Đánh giá thành tích đội ngũ giảng viên trường Đại Học Phạm ...
DV Viết Luận văn luanvanmaster.com ZALO 0973287149
 

More from DV Viết Luận văn luanvanmaster.com ZALO 0973287149 (20)

Ảnh Hưởng Của Marketing Quan Hệ Đến Lòng Trung Thành Của Khách Hàng.Tình Huốn...
Ảnh Hưởng Của Marketing Quan Hệ Đến Lòng Trung Thành Của Khách Hàng.Tình Huốn...Ảnh Hưởng Của Marketing Quan Hệ Đến Lòng Trung Thành Của Khách Hàng.Tình Huốn...
Ảnh Hưởng Của Marketing Quan Hệ Đến Lòng Trung Thành Của Khách Hàng.Tình Huốn...
 
Phát triển nguồn nhân lực tại Uỷ ban nhân dân huyện Trà Bồng, tỉnh Quảng Ngãi...
Phát triển nguồn nhân lực tại Uỷ ban nhân dân huyện Trà Bồng, tỉnh Quảng Ngãi...Phát triển nguồn nhân lực tại Uỷ ban nhân dân huyện Trà Bồng, tỉnh Quảng Ngãi...
Phát triển nguồn nhân lực tại Uỷ ban nhân dân huyện Trà Bồng, tỉnh Quảng Ngãi...
 
Báo cáo tốt Nghiệp tài chính hợp nhất tại tổng công ty Indochina gol...
Báo cáo tốt Nghiệp  tài chính hợp nhất tại tổng công ty Indochina gol...Báo cáo tốt Nghiệp  tài chính hợp nhất tại tổng công ty Indochina gol...
Báo cáo tốt Nghiệp tài chính hợp nhất tại tổng công ty Indochina gol...
 
Tạo động lực thúc đẩy nhân viên làm việc tại ngân hàng TMCP Ngoại Thương Việt...
Tạo động lực thúc đẩy nhân viên làm việc tại ngân hàng TMCP Ngoại Thương Việt...Tạo động lực thúc đẩy nhân viên làm việc tại ngân hàng TMCP Ngoại Thương Việt...
Tạo động lực thúc đẩy nhân viên làm việc tại ngân hàng TMCP Ngoại Thương Việt...
 
Phát triển công nghiệp trên địa bàn Thành phố Tam Kỳ, Tỉnh Quảng Na...
Phát triển công nghiệp trên địa bàn Thành phố Tam Kỳ, Tỉnh Quảng Na...Phát triển công nghiệp trên địa bàn Thành phố Tam Kỳ, Tỉnh Quảng Na...
Phát triển công nghiệp trên địa bàn Thành phố Tam Kỳ, Tỉnh Quảng Na...
 
Giải pháp phát triển cho vay xuất nhập khẩu tại ngân hàng NN&PTNN ch...
Giải pháp phát triển cho vay xuất nhập khẩu tại ngân hàng NN&PTNN ch...Giải pháp phát triển cho vay xuất nhập khẩu tại ngân hàng NN&PTNN ch...
Giải pháp phát triển cho vay xuất nhập khẩu tại ngân hàng NN&PTNN ch...
 
Hoàn thiện công tác lập báo cáo tài chính hợp nhất tại tổng công ...
Hoàn thiện công tác lập báo cáo tài chính hợp nhất tại tổng công ...Hoàn thiện công tác lập báo cáo tài chính hợp nhất tại tổng công ...
Hoàn thiện công tác lập báo cáo tài chính hợp nhất tại tổng công ...
 
Luận Văn Thạc Sĩ Quản trị thành tích nhân viên tại Cục Hải quan TP Đà Nẵng.doc
Luận Văn Thạc Sĩ  Quản trị thành tích nhân viên tại Cục Hải quan TP Đà Nẵng.docLuận Văn Thạc Sĩ  Quản trị thành tích nhân viên tại Cục Hải quan TP Đà Nẵng.doc
Luận Văn Thạc Sĩ Quản trị thành tích nhân viên tại Cục Hải quan TP Đà Nẵng.doc
 
Hoàn thiện công tác quản lý thuế thu nhập cá nhân tại cục thuế Tỉ...
Hoàn thiện công tác quản lý thuế thu nhập cá nhân tại cục thuế Tỉ...Hoàn thiện công tác quản lý thuế thu nhập cá nhân tại cục thuế Tỉ...
Hoàn thiện công tác quản lý thuế thu nhập cá nhân tại cục thuế Tỉ...
 
Đề Tài Phát triển bền vững nông nghiệp Huyện Ba Tơ, Tỉnh Quảng Ngãi....
Đề Tài Phát triển bền vững nông nghiệp Huyện Ba Tơ, Tỉnh Quảng Ngãi....Đề Tài Phát triển bền vững nông nghiệp Huyện Ba Tơ, Tỉnh Quảng Ngãi....
Đề Tài Phát triển bền vững nông nghiệp Huyện Ba Tơ, Tỉnh Quảng Ngãi....
 
Hoàn thiện công tác bảo trợ xã hội trên địa bàn huyện Phong Điền, tỉnh Thừa T...
Hoàn thiện công tác bảo trợ xã hội trên địa bàn huyện Phong Điền, tỉnh Thừa T...Hoàn thiện công tác bảo trợ xã hội trên địa bàn huyện Phong Điền, tỉnh Thừa T...
Hoàn thiện công tác bảo trợ xã hội trên địa bàn huyện Phong Điền, tỉnh Thừa T...
 
Đề Tài Luận VănPhát triển sản phẩm du lịch tại thành phố Đà Nẵng.doc
Đề Tài Luận VănPhát triển sản phẩm du lịch tại thành phố Đà Nẵng.docĐề Tài Luận VănPhát triển sản phẩm du lịch tại thành phố Đà Nẵng.doc
Đề Tài Luận VănPhát triển sản phẩm du lịch tại thành phố Đà Nẵng.doc
 
Đào tạo nghề cho lao động thuộc diện thu hồi đất trên địa bàn Thàn...
Đào tạo nghề cho lao động thuộc diện thu hồi đất trên địa bàn Thàn...Đào tạo nghề cho lao động thuộc diện thu hồi đất trên địa bàn Thàn...
Đào tạo nghề cho lao động thuộc diện thu hồi đất trên địa bàn Thàn...
 
Tóm Tắt Luận Văn Thạc Sĩ Quản Trị Kinh Doanh Xây dựng chính sách Marketing tạ...
Tóm Tắt Luận Văn Thạc Sĩ Quản Trị Kinh Doanh Xây dựng chính sách Marketing tạ...Tóm Tắt Luận Văn Thạc Sĩ Quản Trị Kinh Doanh Xây dựng chính sách Marketing tạ...
Tóm Tắt Luận Văn Thạc Sĩ Quản Trị Kinh Doanh Xây dựng chính sách Marketing tạ...
 
Đề Tài Nghiên cứu rủi ro cảm nhận đối với mua hàng thời trang trực tuyến.docx
Đề Tài Nghiên cứu rủi ro cảm nhận đối với mua hàng thời trang trực tuyến.docxĐề Tài Nghiên cứu rủi ro cảm nhận đối với mua hàng thời trang trực tuyến.docx
Đề Tài Nghiên cứu rủi ro cảm nhận đối với mua hàng thời trang trực tuyến.docx
 
Giải pháp nâng cao động lực thúc đẩy người lao động tại công ty khai...
Giải pháp nâng cao động lực thúc đẩy người lao động tại công ty khai...Giải pháp nâng cao động lực thúc đẩy người lao động tại công ty khai...
Giải pháp nâng cao động lực thúc đẩy người lao động tại công ty khai...
 
Giải pháp phát triển dịch vụ ngân hàng điện tử tại ngân hàng đầu ...
Giải pháp phát triển dịch vụ ngân hàng điện tử tại ngân hàng đầu ...Giải pháp phát triển dịch vụ ngân hàng điện tử tại ngân hàng đầu ...
Giải pháp phát triển dịch vụ ngân hàng điện tử tại ngân hàng đầu ...
 
Giải pháp phát triển dịch vụ ngân hàng điện tử tại ngân hàng đầu ...
Giải pháp phát triển dịch vụ ngân hàng điện tử tại ngân hàng đầu ...Giải pháp phát triển dịch vụ ngân hàng điện tử tại ngân hàng đầu ...
Giải pháp phát triển dịch vụ ngân hàng điện tử tại ngân hàng đầu ...
 
Quản trị quan hệ khách hàng tại Chi nhánh Viettel Đà Nẵng – Tập đoàn Viễn thô...
Quản trị quan hệ khách hàng tại Chi nhánh Viettel Đà Nẵng – Tập đoàn Viễn thô...Quản trị quan hệ khách hàng tại Chi nhánh Viettel Đà Nẵng – Tập đoàn Viễn thô...
Quản trị quan hệ khách hàng tại Chi nhánh Viettel Đà Nẵng – Tập đoàn Viễn thô...
 
Đề Tài Đánh giá thành tích đội ngũ giảng viên trường Đại Học Phạm ...
Đề Tài Đánh giá thành tích đội ngũ giảng viên trường Đại Học Phạm ...Đề Tài Đánh giá thành tích đội ngũ giảng viên trường Đại Học Phạm ...
Đề Tài Đánh giá thành tích đội ngũ giảng viên trường Đại Học Phạm ...
 

Recently uploaded

CHUYÊN ĐỀ DẠY THÊM HÓA HỌC LỚP 10 - SÁCH MỚI - FORM BÀI TẬP 2025 (DÙNG CHUNG ...
CHUYÊN ĐỀ DẠY THÊM HÓA HỌC LỚP 10 - SÁCH MỚI - FORM BÀI TẬP 2025 (DÙNG CHUNG ...CHUYÊN ĐỀ DẠY THÊM HÓA HỌC LỚP 10 - SÁCH MỚI - FORM BÀI TẬP 2025 (DÙNG CHUNG ...
CHUYÊN ĐỀ DẠY THÊM HÓA HỌC LỚP 10 - SÁCH MỚI - FORM BÀI TẬP 2025 (DÙNG CHUNG ...
Nguyen Thanh Tu Collection
 
FSSC 22000 version 6_Seminar_FINAL end.pptx
FSSC 22000 version 6_Seminar_FINAL end.pptxFSSC 22000 version 6_Seminar_FINAL end.pptx
FSSC 22000 version 6_Seminar_FINAL end.pptx
deviv80273
 
Quan Tri Doi Moi Sang Tao_ Innovation Management
Quan Tri Doi Moi Sang Tao_ Innovation ManagementQuan Tri Doi Moi Sang Tao_ Innovation Management
Quan Tri Doi Moi Sang Tao_ Innovation Management
ChuPhan32
 
THONG BAO nop ho so xet tuyen TS6 24-25.pdf
THONG BAO nop ho so xet tuyen TS6 24-25.pdfTHONG BAO nop ho so xet tuyen TS6 24-25.pdf
THONG BAO nop ho so xet tuyen TS6 24-25.pdf
QucHHunhnh
 
Văn 7. Truyện ngụ ngôn Rùa và thỏ+ Viết PT nhân vật.docx
Văn 7. Truyện ngụ ngôn Rùa và thỏ+ Viết PT nhân vật.docxVăn 7. Truyện ngụ ngôn Rùa và thỏ+ Viết PT nhân vật.docx
Văn 7. Truyện ngụ ngôn Rùa và thỏ+ Viết PT nhân vật.docx
metamngoc123
 
BÀI TẬP BỔ TRỢ TIẾNG ANH I-LEARN SMART WORLD 9 CẢ NĂM CÓ TEST THEO UNIT NĂM H...
BÀI TẬP BỔ TRỢ TIẾNG ANH I-LEARN SMART WORLD 9 CẢ NĂM CÓ TEST THEO UNIT NĂM H...BÀI TẬP BỔ TRỢ TIẾNG ANH I-LEARN SMART WORLD 9 CẢ NĂM CÓ TEST THEO UNIT NĂM H...
BÀI TẬP BỔ TRỢ TIẾNG ANH I-LEARN SMART WORLD 9 CẢ NĂM CÓ TEST THEO UNIT NĂM H...
Nguyen Thanh Tu Collection
 
Ngân hàng điện tử số ptit - giảng viên cô Hà
Ngân hàng điện tử số ptit - giảng viên cô HàNgân hàng điện tử số ptit - giảng viên cô Hà
Ngân hàng điện tử số ptit - giảng viên cô Hà
onLongV
 
Halloween vocabulary for kids in primary school
Halloween vocabulary for kids in primary schoolHalloween vocabulary for kids in primary school
Halloween vocabulary for kids in primary school
AnhPhm265031
 
Chương 3 Linh kien ban dan và KD dien tu - Copy.ppt
Chương 3 Linh kien ban dan và KD dien tu - Copy.pptChương 3 Linh kien ban dan và KD dien tu - Copy.ppt
Chương 3 Linh kien ban dan và KD dien tu - Copy.ppt
PhiTrnHngRui
 
insulin cho benh nhan nam vien co tang duong huyet
insulin cho benh nhan nam vien co tang duong huyetinsulin cho benh nhan nam vien co tang duong huyet
insulin cho benh nhan nam vien co tang duong huyet
lmhong80
 
Sinh-12-Chuyên-2022-2023.dành cho ôn thi hsg
Sinh-12-Chuyên-2022-2023.dành cho ôn thi hsgSinh-12-Chuyên-2022-2023.dành cho ôn thi hsg
Sinh-12-Chuyên-2022-2023.dành cho ôn thi hsg
vivan030207
 
Biểu tượng trăng và bầu trời trong tác phẩm của Nguyễn Quang Thiều
Biểu tượng trăng và bầu trời trong tác phẩm của Nguyễn Quang ThiềuBiểu tượng trăng và bầu trời trong tác phẩm của Nguyễn Quang Thiều
Biểu tượng trăng và bầu trời trong tác phẩm của Nguyễn Quang Thiều
lamluanvan.net Viết thuê luận văn
 
Giải phẫu tim sau đại học- LÊ QUANG TUYỀN
Giải phẫu tim sau đại học- LÊ QUANG TUYỀNGiải phẫu tim sau đại học- LÊ QUANG TUYỀN
Giải phẫu tim sau đại học- LÊ QUANG TUYỀN
linh miu
 
Smartbiz_He thong MES nganh may mac_2024june
Smartbiz_He thong MES nganh may mac_2024juneSmartbiz_He thong MES nganh may mac_2024june
Smartbiz_He thong MES nganh may mac_2024june
SmartBiz
 
40 câu hỏi - đáp Bộ luật dân sự năm 2015 (1).doc
40 câu hỏi - đáp Bộ  luật dân sự năm  2015 (1).doc40 câu hỏi - đáp Bộ  luật dân sự năm  2015 (1).doc
40 câu hỏi - đáp Bộ luật dân sự năm 2015 (1).doc
NguynDimQunh33
 
PLĐC-chương 1 (1).ppt của trường ĐH Ngoại thương
PLĐC-chương 1 (1).ppt của trường  ĐH Ngoại thươngPLĐC-chương 1 (1).ppt của trường  ĐH Ngoại thương
PLĐC-chương 1 (1).ppt của trường ĐH Ngoại thương
hieutrinhvan27052005
 
100 DẪN CHỨNG NGHỊ LUẬN XÃ HỘiI HAY.docx
100 DẪN CHỨNG NGHỊ LUẬN XÃ HỘiI HAY.docx100 DẪN CHỨNG NGHỊ LUẬN XÃ HỘiI HAY.docx
100 DẪN CHỨNG NGHỊ LUẬN XÃ HỘiI HAY.docx
khanhthy3000
 
LỊCH SỬ 12 - CHUYÊN ĐỀ 10 - TRẮC NGHIỆM.pptx
LỊCH SỬ 12 - CHUYÊN ĐỀ 10 - TRẮC NGHIỆM.pptxLỊCH SỬ 12 - CHUYÊN ĐỀ 10 - TRẮC NGHIỆM.pptx
LỊCH SỬ 12 - CHUYÊN ĐỀ 10 - TRẮC NGHIỆM.pptx
12D241NguynPhmMaiTra
 

Recently uploaded (18)

CHUYÊN ĐỀ DẠY THÊM HÓA HỌC LỚP 10 - SÁCH MỚI - FORM BÀI TẬP 2025 (DÙNG CHUNG ...
CHUYÊN ĐỀ DẠY THÊM HÓA HỌC LỚP 10 - SÁCH MỚI - FORM BÀI TẬP 2025 (DÙNG CHUNG ...CHUYÊN ĐỀ DẠY THÊM HÓA HỌC LỚP 10 - SÁCH MỚI - FORM BÀI TẬP 2025 (DÙNG CHUNG ...
CHUYÊN ĐỀ DẠY THÊM HÓA HỌC LỚP 10 - SÁCH MỚI - FORM BÀI TẬP 2025 (DÙNG CHUNG ...
 
FSSC 22000 version 6_Seminar_FINAL end.pptx
FSSC 22000 version 6_Seminar_FINAL end.pptxFSSC 22000 version 6_Seminar_FINAL end.pptx
FSSC 22000 version 6_Seminar_FINAL end.pptx
 
Quan Tri Doi Moi Sang Tao_ Innovation Management
Quan Tri Doi Moi Sang Tao_ Innovation ManagementQuan Tri Doi Moi Sang Tao_ Innovation Management
Quan Tri Doi Moi Sang Tao_ Innovation Management
 
THONG BAO nop ho so xet tuyen TS6 24-25.pdf
THONG BAO nop ho so xet tuyen TS6 24-25.pdfTHONG BAO nop ho so xet tuyen TS6 24-25.pdf
THONG BAO nop ho so xet tuyen TS6 24-25.pdf
 
Văn 7. Truyện ngụ ngôn Rùa và thỏ+ Viết PT nhân vật.docx
Văn 7. Truyện ngụ ngôn Rùa và thỏ+ Viết PT nhân vật.docxVăn 7. Truyện ngụ ngôn Rùa và thỏ+ Viết PT nhân vật.docx
Văn 7. Truyện ngụ ngôn Rùa và thỏ+ Viết PT nhân vật.docx
 
BÀI TẬP BỔ TRỢ TIẾNG ANH I-LEARN SMART WORLD 9 CẢ NĂM CÓ TEST THEO UNIT NĂM H...
BÀI TẬP BỔ TRỢ TIẾNG ANH I-LEARN SMART WORLD 9 CẢ NĂM CÓ TEST THEO UNIT NĂM H...BÀI TẬP BỔ TRỢ TIẾNG ANH I-LEARN SMART WORLD 9 CẢ NĂM CÓ TEST THEO UNIT NĂM H...
BÀI TẬP BỔ TRỢ TIẾNG ANH I-LEARN SMART WORLD 9 CẢ NĂM CÓ TEST THEO UNIT NĂM H...
 
Ngân hàng điện tử số ptit - giảng viên cô Hà
Ngân hàng điện tử số ptit - giảng viên cô HàNgân hàng điện tử số ptit - giảng viên cô Hà
Ngân hàng điện tử số ptit - giảng viên cô Hà
 
Halloween vocabulary for kids in primary school
Halloween vocabulary for kids in primary schoolHalloween vocabulary for kids in primary school
Halloween vocabulary for kids in primary school
 
Chương 3 Linh kien ban dan và KD dien tu - Copy.ppt
Chương 3 Linh kien ban dan và KD dien tu - Copy.pptChương 3 Linh kien ban dan và KD dien tu - Copy.ppt
Chương 3 Linh kien ban dan và KD dien tu - Copy.ppt
 
insulin cho benh nhan nam vien co tang duong huyet
insulin cho benh nhan nam vien co tang duong huyetinsulin cho benh nhan nam vien co tang duong huyet
insulin cho benh nhan nam vien co tang duong huyet
 
Sinh-12-Chuyên-2022-2023.dành cho ôn thi hsg
Sinh-12-Chuyên-2022-2023.dành cho ôn thi hsgSinh-12-Chuyên-2022-2023.dành cho ôn thi hsg
Sinh-12-Chuyên-2022-2023.dành cho ôn thi hsg
 
Biểu tượng trăng và bầu trời trong tác phẩm của Nguyễn Quang Thiều
Biểu tượng trăng và bầu trời trong tác phẩm của Nguyễn Quang ThiềuBiểu tượng trăng và bầu trời trong tác phẩm của Nguyễn Quang Thiều
Biểu tượng trăng và bầu trời trong tác phẩm của Nguyễn Quang Thiều
 
Giải phẫu tim sau đại học- LÊ QUANG TUYỀN
Giải phẫu tim sau đại học- LÊ QUANG TUYỀNGiải phẫu tim sau đại học- LÊ QUANG TUYỀN
Giải phẫu tim sau đại học- LÊ QUANG TUYỀN
 
Smartbiz_He thong MES nganh may mac_2024june
Smartbiz_He thong MES nganh may mac_2024juneSmartbiz_He thong MES nganh may mac_2024june
Smartbiz_He thong MES nganh may mac_2024june
 
40 câu hỏi - đáp Bộ luật dân sự năm 2015 (1).doc
40 câu hỏi - đáp Bộ  luật dân sự năm  2015 (1).doc40 câu hỏi - đáp Bộ  luật dân sự năm  2015 (1).doc
40 câu hỏi - đáp Bộ luật dân sự năm 2015 (1).doc
 
PLĐC-chương 1 (1).ppt của trường ĐH Ngoại thương
PLĐC-chương 1 (1).ppt của trường  ĐH Ngoại thươngPLĐC-chương 1 (1).ppt của trường  ĐH Ngoại thương
PLĐC-chương 1 (1).ppt của trường ĐH Ngoại thương
 
100 DẪN CHỨNG NGHỊ LUẬN XÃ HỘiI HAY.docx
100 DẪN CHỨNG NGHỊ LUẬN XÃ HỘiI HAY.docx100 DẪN CHỨNG NGHỊ LUẬN XÃ HỘiI HAY.docx
100 DẪN CHỨNG NGHỊ LUẬN XÃ HỘiI HAY.docx
 
LỊCH SỬ 12 - CHUYÊN ĐỀ 10 - TRẮC NGHIỆM.pptx
LỊCH SỬ 12 - CHUYÊN ĐỀ 10 - TRẮC NGHIỆM.pptxLỊCH SỬ 12 - CHUYÊN ĐỀ 10 - TRẮC NGHIỆM.pptx
LỊCH SỬ 12 - CHUYÊN ĐỀ 10 - TRẮC NGHIỆM.pptx
 

Định lý zsigmondy và Tính chất số học của đa thức.docx

  • 1. ĐẠI HỌC THÁI NGUYÊN TRƢỜNG ĐẠI HỌC KHOA HỌC ------------------------------- vTải tài liệu tại sividoc.com Viết đề tài giá sinh viên – ZALO:0973.287.149-TEAMLUANVAN.COM ĐỖ LAN HƢƠNG ĐỊNH LÝ ZSIGMONDY VÀ TÍNH CHẤT SỐ HỌC CỦA ĐA THỨC LUẬN VĂN THẠC SĨ TOÁN HỌC THÁI NGUYÊN - 2018
  • 2. ĐẠI HỌC THÁI NGUYÊN TRƢỜNG ĐẠI HỌC KHOA HỌC ------------------------------- vTải tài liệu tại sividoc.com Viết đề tài giá sinh viên – ZALO:0973.287.149-TEAMLUANVAN.COM ĐỖ LAN HƢƠNG ĐỊNH LÝ ZSIGMONDY VÀ TÍNH CHẤT SỐ HỌC CỦA ĐA THỨC Chuyên ngành: Phƣơng pháp Toán sơ cấp Mã số 84 60 113 LUẬN VĂN THẠC SĨ TOÁN HỌC NGƯỜI HƯỚNG DẪN KHOA HỌC PGS.TS. ĐÀM VĂN NHỈ THÁI NGUYÊN - 2018
  • 3. 1 Viết đề tài giá sinh viên – ZALO:0973.287.149-TEAMLUANVAN.COM Mnc lnc 1 Định lj Zsigmondy 4 1.1 Đa thác và so phác . . . . . . . . . . . . . . . . . . . . . 4 1.1.1 Khái ni m đa thác, phép toán . . . . . . . . . . . 4 1.1.2 Thu t toán Euclid . . . . . . . . . . . . . . . . . 5 1.1.3 Xây dựng trường so phác C . . . . . . . . . . . . 6 1.2 Đa thác chia đường tròn....................................................... 13 1.2.1 Đa thác chia đường tròn........................................... 13 1.2.2 V n dụng..............................................................19 1.3 Định lý Zsigmondy................................................................. 21 1.3.1 Định lý Zsigmondy ..................................................... 21 1.3.2 V n dụng Định lý Zsigmondy ...............................23 2 Tính chat so hoc của đa thfíc 27 2.1 Tính chat đ c bi t của đa thác thu®c Z[x]..........................27 2.1.1 Định lý Bézout.........................................................27 2.1.2 V n dụng..............................................................29 2.2 Đa thác Hilbert và bieu dien Mahler................................... 38 2.3 V n dụng giải bài toán thi hoc sinh giỏi.............................40 Ket lu n 44 Tài li u tham khảo 45
  • 4. 2 Viết đề tài giá sinh viên – ZALO:0973.287.149-TEAMLUANVAN.COM L i cảm ơn Lu n văn này được thực hi n tại Trường Đại hoc Khoa hoc - Đại hoc Thái Nguyên và hoàn thành với sự hướng dan của PGS.TS. Đàm Văn Nhỉ. Tác giả xin được bày tỏ lòng biet ơn chân thành và sâu sac tới người hướng dan khoa hoc của mình, người đã đ t van đe nghiên cáu, dành nhieu thời gian hướng dan và t n tình giải đáp nhǎng thac mac của tác giả trong suot quá trình làm lu n văn. Tác giả xin trân trong cảm ơn Ban Giám hi u Trường Đại hoc Khoa hoc - Đại hoc Thái Nguyên, Ban Chủ nhi m Khoa Toán - Tin, cùng các giảng viên đã tham gia giảng dạy, đã tạo moi đieu ki n tot nhat đe tác giả hoc t p và nghiên cáu. Tác giả muon gải nhǎng lời cảm ơn tot đep nhat tới t p the Lớp B, cao hoc Toán khóa 10 (2016 - 2018) đã đ®ng viên và giúp đơ tác giả rat nhieu trong suot quá trình hoc t p. Nhân dịp này, tác giả cũng xin chân thành cảm ơn Sở Giáo dục và Đào tạo Hải Phòng, Ban Giám hi u và các đong nghi p ở Trường THPT Lý Thường Ki t, Huy n Thủy Nguyên, Thành pho Hải Phòng đã tạo đieu ki n cho tác giả hoàn thành tot nhi m vụ hoc t p và công tác của mình. Cuoi cùng, tác giả muon dành nhǎng lời cảm ơn đ c bi t nhat đen bo me và đại gia đình đã luôn đ®ng viên và chia sẻ nhǎng khó khăn đe tác giả hoàn thành tot lu n văn này.
  • 5. 3 Viết đề tài giá sinh viên – ZALO:0973.287.149-TEAMLUANVAN.COM L i nói đau Đa thác có vị trí rat quan trong trong Toán hoc vì nó không nhǎng là m®t đoi tượng nghiên cáu trong tâm của Đại so mà còn là m®t công cụ đac lực của Giải tích trong lý thuyet xap xỉ, lý thuyet bieu dien, lý thuyet n®i suy,... Ngoài ra, đa thác còn được sả dụng nhieu trong tính toán và áng dụng. Trong các kì thi hoc sinh giỏi toán quoc gia và Olympic toán quoc te thì các bài toán ve đa thác cũng thường được đe c p đen và được xem như nhǎng bài toán khó của b c phő thông. Đã có nhieu đe tài viet ve đa thác nhưng trong lu n văn của mình tôi muon t p trung xét vi c v n dụng đa thác trong so hoc. Mục đích của lu n văn này là giới thi u Định lý Zsigmondy - m®t định lý rat mạnh trong xả lý các bài toán khó ve so nguyên to và giới thi u tính chat đ c bi t của đa thác thu®c Z[x]. Lu n văn gom phan mở đau, ket lu n và hai chương. Chương 1. Định lj Zsigmondy. Chương này gom ba mục chính: Mục 1.1 trình bày ve m®t so tính chat cơ bản ve đa thác và so phác. Mục 1.2 trình bày ve đa thác chia đường tròn. Mục 1.3 trình bày ve Định lý Zsigmondy và v n dụng Định lý Zsigmondy trong giải m®t so bài toán thi hoc sinh giỏi. Chương 2. Tính chat so hoc của đa thfíc. Chương này được chia thành ba mục chính: Mục 2.1 trình bày ve tính chat đ c bi t của đa thác thu®c Z[x]. Mục 2.2 trình bày ve đa thác Hilbert và bieu dien Mahler. Mục 2.3 trình bày ve cách v n dụng đa thác Hilbert.
  • 6. 4 Viết đề tài giá sinh viên – ZALO:0973.287.149-TEAMLUANVAN.COM Σ } Σ Σ Σ Chương 1 Định lj Zsigmondy Trước khi giới thi u ve định lý Zsigmondy, phan đau của chương này lu n văn trình bày các kien thác cơ sở ve đa thác, trường so phác và đa thác chia đường tròn. Các kien thác trong chương này được tham khảo tà tài li u [1] và [3]. 1.1 Đa thfíc và so phfíc 1.1.1 Khái ni m đa thfíc, phép toán Mục này t p trung nghiên cáu vành các đa thác m®t bien trên m®t trường. Trường K có the là trường Q, R, C. Ký hi u t p đa thác trên K n K[x] = {a0 + a1x + · · · + anxn |ai ∈ K, n ∈ N} = aixi | ai ∈ K . i=0 Moi phan tả thu®c K[x] được viet là f(x) ho c đơn giản f. Phan tả n f = i=0 aix i với quy ước x0 = 1, được goi là m®t đa thúc của bien x với các h tả thu®c K. Khi an 0 và n là so tự nhiên thì n được goi là b c của đa thác f và được ký hi u n = deg f ; an được goi là h tủ cao nhat; a0 được goi là h tủ tự do hay so hạng tự do. Trường hợp f = a /= 0, a ∈ K , được goi là đa thác b¾c 0. Đ c bi t, khi f = 0 thì đa thác này được quy ước có b c −1 ho c −∞, tùy theo vi c sả dụng b c vào lĩnh vực nào. Đa thác dạng f = a0 + a1x + · · · + an−1xn−1 + xn được goi là đa thúc monic. Các phép toán trong K[x] : Với f = n i=0 aixi , g = m i=0 bix i ∈ K[x]
  • 7. 5 Viết đề tài giá sinh viên – ZALO:0973.287.149-TEAMLUANVAN.COM Σ Σ ta định nghĩa f = g khi và chỉ khi m = n ai = bi, i = 0, 1, . . . , n f + g = Σ (ai + bi)xi , fg = m Σ +n Σ i ai−jbj xi . i=0 i=0 j=0 M nh đe 1.1. Với các phép toán trên, K[x] l¾p thành m®t vành giao hoán có đơn v . M nh đe 1.2. Với hai đa thúc f, g ∈ K[x] ta có các ket quả ve b¾c: (1) deg(f + g) ≤ max{deg f, deg g}. (2) deg(fg) = deg f. deg g. Chfíng minh. (1) Giả sả f = n i=0 aix i và g = m i=0 bixi . Không hạn che có the coi m ≤ n. Neu m < n thì deg(f + g) = n ≤ max{n, m}. Neu m = n và an + bn = / 0 thì deg(f + g) = n = max{n, n}. Neu m = n và an + bn = 0 thì deg(f + g) < n = max{n, n}. Tóm lại, ta luôn có deg(f + g) ≤ max{deg f, deg g}. (2) Vì an, bm /= 0 nên an.bm 0. Do v y deg(fg) = m.n = deg f. deg g. 1.1.2 Thu t toán Euclid Cho hai đa thác f(x) và g(x) với b c n = deg f(x) và m = deg g(x). Giả thiet m > 0. Neu có đa thác h(x) đe f(x) = h(x)g(x) thì ta nói rang f (x) chia het cho g(x) với thương h(x). Neu không có đa thác h(x) nào đe f (x) = h(x)g(x) thì ta nói rang đa thác f (x) không chia het cho g(x). Ta có hai đa thác duy nhat h(x), r(x) đe f(x) = h(x)g(x) + r(x), deg r(x) < m. Đa thác r(x) được goi là đa thác dư trong phép chia đa thác f(x) cho đa thác g(x).
  • 8. 6 Viết đề tài giá sinh viên – ZALO:0973.287.149-TEAMLUANVAN.COM − Định lj 1.1. Với các đa thúc f(x), g(x) thu®c vành K[x] và g(x) /= 0 có hai đa thúc duy nhat q(x), r(x) sao cho f (x) = q(x)g(x) + r(x), trong đó deg r(x) < deg g(x). Chfíng minh. Sự ton tại: Giả sả f(x) = anxn +an−1xn−1 +· · ·+a2x2 + a1x + a0 và g(x) = bmxm + bm−1xm−1 + · · · + b1x + b0. Neu n < m thì chon q(x) = 0, r(x) = f(x). Neu n ≥ m thì ta xét hi u f1 (x) = f(x) an xn−m g(x). Khi đó n = bm 1 deg f1 (x) ≤ n−1. Neu n1 < m thì chon q(x) = an xn−m và r(x) = f bm 1(x). Neu n1 ≥ m ta tiep tục quá trình trên. Sau m®t so hǎu hạn bước, ta đạt được q(x) và r(x) thỏa mãn các yêu cau đ t ra. Tính duy nhat: Giả sả có các đa thác q1(x), q2(x), r1(x), r2(x) thỏa mãn q1(x)g(x)+r1(x) = f (x) = q2(x)g(x)+r2(x) với deg r1(x), deg r2(x) < m. Tà đây suy ra [q1(x) − q2(x)]g(x) = r1(x) − r2(x). Neu q1(x)−q2(x) /= 0 thì deg[q1(x)−q2(x)]g(x) ≥ m > deg[r1(x)−r2(x)], vô lý. Tà đó suy ra q1(x) = q2(x) và r1(x) = r2(x). Định nghĩa 1.1. Đa thác d(x) được goi là nhân tủ chung của hai đa thác f(x) và g(x) neu f(x) và g(x) cùng chia het cho đa thác d(x). Hai đa thác f (x) và g(x) được goi là nguyên to cùng nhau neu chúng chỉ có ước chung là các đa thác b c 0. Định lj 1.2. [Bézout] Hai đa thúc f(x) và g(x) nguyên to cùng nhau khi và chí khi có hai đa thúc p(x), q(x) đe p(x)f(x) + q(x)g(x) = 1. Định lj 1.3. Vành K[x] là m®t vành chính và nó là vành nhân tủ hóa. 1.1.3 Xây dfing trư ng so phfíc C Xét tích T = RxR = {(a, b) |a, b ∈ R}. Với kí hi u i ∈ / R ta đong nhat c p (a, b) với a + bi và tích Carte T = RxR được coi như t p
  • 9. 7 Viết đề tài giá sinh viên – ZALO:0973.287.149-TEAMLUANVAN.COM √ } T = {(a + bi) |a, b ∈ R}. Định nghĩa các phép toán trong T: a + bi = c + di khi và chỉ khi a = c, b = d (a + bi) + (c + di) = (a + c) + (b + d)i (a + bi).(c + di) = (ac − bd) + (ad + bc)i a = a + 0i, i = 0 + bi, bi = ib. Đe đơn giản, ta quy ước viet (a + bi)(c + di) thay cho (a + bi).(c + di) Tà định nghĩa, ta có : (1) Với i = 0 + 1i ∈ T có i2 = (0 + 1i)(0 + 1i) = −1 + 0i = −1 (2) (a + bi)(1 + 0i) = a + bi = (1 + 0i)(a + bi). Ký hi u C là t p T cùng với các phép toán đã nêu ra ở trên. Ta có: Bo đe 1.1. Ánh xạ φ : R → C, a ›→ (a, 0), là m®t đơn ánh và nó thóa mãn φ(a + a′) = φ(a) + φ(a′), φ(aa′) = φ(a)φ(a′) với moi a, a′ ∈ R. Đong nhat (a, 0) ∈ C với a ∈ R. Khi đó ta có the viet (a, b) = (a, 0) + (b, 0)(0, 1) = a + bi với i2 = (−1, 0) = −1. Do đó i hay a ho c a+ bi là bình đȁng trong C. Như v y C = (a + bi) |a, b ∈ R, i2 = −1 và trong C ta có các ket quả: a + bi = c + di khi và chỉ khi a = c, b = d a + bi + c + di = a + c + (b + d)i (a + bi)(c + di) = ac − bd + (ad + bc)i. Moi phan tả z = a + bi ∈ C được goi là m®t so phác với phan thực a, ký hi u Re(z), và phan ảo b, ký hi u Im(z); còn i được goi là đơn vị ảo. So phác a − bi được goi là so phác liên hợp của của z = a + bi và được ký hi u là z = a + bi. Ta có zz = (a + bi) (a − bi) = a2 + b2 , z1z2 = z1 z2 và goi |z| = zz là mô-đun của z. So đoi của z′ = c + di là −z′ = −c − di và hi u z − z′ = (a + bi) − (c + di) = a − c + (b − d)i. Xét m t phȁng toa đ® (Oxy). Moi so phác z = a + bi ta cho tương áng với điem M (a;b). Tương áng này là m®t song ánh: C → R × R, z = a + bi → M (a; b) .
  • 10. 8 Viết đề tài giá sinh viên – ZALO:0973.287.149-TEAMLUANVAN.COM a 2 √ − − Khi đong nhat C với (Oxy) qua vi c đong nhat z với M, m t phȁng toa đ® bieu dien so phác như the goi là m¾t phȁng phúc hay m t phȁng Gauss, ghi công C. F. Gauss-người đau tiên đưa ra bieu dien. M nh đe 1.3. C là trường chúa trường R như m®t trường con. Chfíng minh. De dàng kiem tra C là m®t vành giao hoán với đơn vị 1. Giả sả z = a + bi = / 0. Khi đó a2 + b2 > 0. Giả sả z′ =x + yi ∈ C thỏa mãn zz’=1 hay ( ax − by = 1 . Giải h ta được x = a2 + b2 b bx + ay = 0 y = − a2 + b2 V y z′ = a a2 + b2 b — a2 + b2 i là nghịch đảo của z, ký hi u là z−1 = 1 . z Như v y C là m®t trường . Tương áng C → C, z → z, là m®t tự đȁng cau liên hợp. Đong nhat a ∈ R với a + 0i ∈ C và coi R như là m®t trường con của C hay R ⊂ C. Chú ý, nghịch đảo của z 0 là z−1 = z |z| và z′ = z′z−1 = z′ z . z |z| Định nghĩa 1.2. Cho so phúc z /= 0. Giả sủ M là điem trong m¾t phȁng phúc bieu dien so phúc z. So đo (rađian) của mői góc lượng giác tia đau Ox và tia cuoi OM được goi là m®t Argument của z và được ký hi u là Arg(z). Góc α=x ^ OM, − π ≤ α ≤ π, được goi là argument của z và được ký hi u bới argz. Argument của so phúc 0 là không đ nh nghĩa. Chú ý, neu α là m®t argument của z thì moi argument của z đeu có dạng α+k2.π với k ∈ Z. Với z /= 0 , ký hi u α+k.2π là argument của z . Ký hi u r = zz. Khi đó so phác z = a + bi có a = rcosα, b = r sin α.V y khi z = / 0 thì có the bieu dien z = r (cos α + i sin α) và bieu dien này được goi là dạng lượng giác của z. Ví dn 1.1. Với a + bi =(x + iy)n có a2 + b2 = x2 + y2 n . Bài giải. Tà a + bi = x + iy n suy ra a bi = x a2 + b2 = x2 + y2 n . iy n . Như v y 2
  • 11. 9 Viết đề tài giá sinh viên – ZALO:0973.287.149-TEAMLUANVAN.COM (2) z1z2 = r1r2 [cos (α1 + α2) + i sin (α1 + α2)] . M nh đe 1.4. [Moivre] Neu z = r (cosα + i sin α) thì với mői so nguyên dương n có zn =rn [cos (na) + i sin (na) ]. H quả 1.1. Căn b¾c n của m®t so phúc z = r(cosa + i sin a) /= 0 là n 1 giá tr khác nhau zk=rn (cos α + 2kπ + i sin n α + 2kπ ) với k = 1,2,...,n. n Tích vô hướng và tích l ch của hai so phác z1, z2, ký hi u < z1, z2 > và [z1, z2], được định nghĩa tương áng qua các công thác sau đây: <z1, z2 >= 1 2 (z1z2 + z1z2) , [z1,z2] = 1 2i (z1z2 − z1z2) . M nh đe 1.5. Neu z1 = r1 (cos α1 + i sin α1) , z2 = r2 (cos α2 + i sin α2] với r1, r2 ≥ 0 thì: . . (1) |z z | = |z ||z | , .z1 . = |z1| và |z + z | ≤ |z | + |z | . 1 2 1 2 .z2 . |z2| 1 2 1 2 (3) z1 z2 = r1 [cos (α r2 1 — α2) + i sin (α1 — α2)] khi r2 > 0. (4) <z1, z2 >= |z1| |z2| cos (α1 − α2) và < z1, z2 >=< z2, z1>. (5) < az1 + bz3, z2 >= a < z1, z2 > +b < z3, z2 > với moi so phúc z1, z2, z3
  • 12. 10 Viết đề tài giá sinh viên – ZALO:0973.287.149-TEAMLUANVAN.COM và moi a, b ∈ R. (6) [z1, z2] = |z1| |z2| sin (α2 − α1) và [z1, z2] = −[z2, z1]. (7) Với z1 = cosα1 + i sin α2, z2 = cosα2 + i sin α2 ta có bieu dien
  • 13. 11 Viết đề tài giá sinh viên – ZALO:0973.287.149-TEAMLUANVAN.COM z − z = 2i sin α1 − α2 cos α1 + α2 + i sin α1 + α2
  • 14. 12 Viết đề tài giá sinh viên – ZALO:0973.287.149-TEAMLUANVAN.COM
  • 15. 13 Viết đề tài giá sinh viên – ZALO:0973.287.149-TEAMLUANVAN.COM
  • 16. 14 Viết đề tài giá sinh viên – ZALO:0973.287.149-TEAMLUANVAN.COM | = 2 .sin α1 − α2 . .
  • 17. 15 Viết đề tài giá sinh viên – ZALO:0973.287.149-TEAMLUANVAN.COM 1 2 . 2 . M nh đe 1.6. Với hai so phúc z1 và z2 ta luôn có các h thúc liên h sau:
  • 18. 16 Viết đề tài giá sinh viên – ZALO:0973.287.149-TEAMLUANVAN.COM z1 = z2 ⇔ |z1| = |z2| , arg z1 = arg z2 + 2kπ, k ∈ Z.
  • 19. 17 Viết đề tài giá sinh viên – ZALO:0973.287.149-TEAMLUANVAN.COM arg( z1z2 ) = arg(z1) + arg(z2) + 2kπ, k ∈ Z. Arg (z1z2) = Arg (z1) + Arg (z2) . Arg z1 = Arg (z z2 1) − Arg (z2) . Người ta chỉ ra rang, đa thác b c dương thu®c C[x] đeu có nghi m trong C. Đó chính là n®i dung Định lý cơ bản của đại so mà người đau tiên cháng minh là nhà toán hoc Gauss (1977 − 1985). Định nghĩa 1.3. Trường K được goi là m®t trường đóng đại so neu moi đa thúc b¾c dương thu®c K[x] đeu có nghi m trong K. Như v y trong K[x] moi đa thác b c dương đeu phân tích được thành tích các nhân tả tuyen tính khi K là m®t trường đóng đại so. Định lj 1.4. [D’Alembert-Gauss, định lý cơ bản của đại so] Moi đa thúc b¾c dương thu®c C[x] đeu có ít nhat m®t nghi m thu®c C. Ví dn 1.2. Giả sủ hai đa thúc f, g ∈ R [x] {0} thóa mãn f (x2 + x + 1) = f (x)g(x). Chúng minh rang b¾c của f là so nguyên dương chȁn. Bài giải. Giả sả đa thác f có b c lẻ. Khi đó f (x) = 0 có ít nhat m®t nghi m thực α. Tà h thác f(x2 + x + 1) = f(x)g(x) suy ra α2 + α + 1 cũng là nghi m của f(x) = 0. Theo quy lu t này, dãy so a0 = α và 2 r+1 + ar + 1 đeu là nghi m của f(x) = 0. Đieu đó cháng tỏ, f(x) = 0 có vô hạn nghi m, mâu thuan . Do v y, tat cả các nghi m của f (x) = 0 đeu là so phác, không là so thực. Tà đó suy ra b c của f là m®t so nguyên dương chȁn. Ví dn 1.3. Chúng minh rang, tat cả các nghi m của đa thúc dạng f(x) = xn + 2nxn−1 + 2n2 xn−2 + ... không the đeu là nhũng so thực. Bài giải. Giả sả tat cả các nghi m x1, x2, ..., xn đeu là nhǎng so thực. ) + 2kπ, k ∈ Z. 2 ) − arg (z 1 = arg (z z 1 z 2 arg a
  • 20. 18 Viết đề tài giá sinh viên – ZALO:0973.287.149-TEAMLUANVAN.COM Σ n n Σ n n−1 n i Σ i . Ví dn 1.5. Giả sủ n so phúc z1, ..., zn thóa mãn |z1|+ . · · · +.|zn| = 2017. . k. 4 4 4 . k. 4 Σ xi ! Theo định lý Vi-ét ta có: n xi = −2n, i=1 1≤ Σ i<j≤n xixj = 2n2 . V y 2n2 = Σ xixj = 1 " Σ n 2 n xi − xi 2 # ≤ n − 1 Σ n 2 1≤i<j≤n 2 i=1 i=1 2n i=1 và suy ra 2n2 < 2n(n − 1), mâu thuan. Như v y đieu giả sả là sai, suy ra đieu phải cháng minh. Ví dn 1.4. Tìm tat cả các đa thúc f(x) = a0xn +a1xn−1 +a2xn−2 +...+an với các ai ∈ {−1; 1} với moi i sao cho tat cả các nghi m đeu là so thực. Bài giải. Giả sả tat cả các nghi m x1, x2, ..., xn đeu là nhǎng so thực. Theo định lý Vi-ét ta có: Σ i=1 xi 2 = n i=1 2 xi − 2 1≤ Σ i<j≤n xixj = a2 — 2an−2 ≤ 3, Y i=1 x2 = 1. V y n i=1 x2 ≥ n. Tà đây suy ra n ≤ 3 và x1x2x3 = ±1. Tà đó xác định được các đa thác x ± 1, x2 ± x − 1, x3 − x ± (x2 − 1). Chúng minh, ton tại m®t t¾p con I ⊂ {1, 2, ..., n} đe . Σ z 2017 ≥ . Bài giải. Giả sả zk = ak + ibk, ak, bk ∈ R, với k = 1, 2, ..., n. Khi đó n n n 2017 = Σ |zk| ≤ Σ |ak| + Σ |bk| = Σ ak − Σ ak + Σ bj − Σ bj với các t p con I, J ⊂ {1, 2, ..., n} và ak, bj ≥ 0. Tà bieu dien này ta suy ra ho c Σ a 2017 ≥ ho c Σ b ≥ 2017 , chȁng hạn Σ a 2017 ≥ . Như v y . Σ z . ≥ Σ a 2017 ≥ . j∈J k∈I k∈I k=1 k=1 k=1 k∈I k∈ /I j∈J j∈ /J k 4 j k∈I k k∈I k∈I k
  • 21. 19 Viết đề tài giá sinh viên – ZALO:0973.287.149-TEAMLUANVAN.COM k 1 2 n 2 2 2 2 2 2 Ví dn 1.6. Giả sủ đa thúc f(x) = xn +a1xn−1 +a2xn−2 +· · ·+an ∈ C[x] với n nghi m α1, α2, ..., αn và đa thúc g(x) = xn +b1xn−1 +b2xn−2 +· · ·+bn với n nghi m α2 , α2 , ..., α2 . Chúng minh rang, neu a1 + a3 + a5 + · · · và a2 + a4 + a6 + · · · đeu là nhũng so thực thì g(1) cũng là m®t so thực. Bài giải. Ta có: n n n g x2 = Y x2 − α2 = Y (x − αk) Y (x + αk) = (−1)n f (x) f (−x) . k=1 Bieu dien k=1 k=1 g(1) − 1 = (−1)n f(1)f(−1) − 1 = (−1)n (1 + b − a)(1 + b + a), ở đó a = a1 + a3 + a5 + · · · và b = a2 + a4 + a6 + · · · đeu là nhǎng so thực. Do v y, g(1) cũng là m®t so thực. Ví dn 1.7. Tính lim cos2x+ · · · + cosnx . 1 1 1 n→∞ Bài giải. Đ t tőng 1 + 2 cosx+ 22 2n 1 1 1 T = 1 + 2 cosx+ 22 cos2x+ · · · + 2n cosnx và tőng 1 1 1 iS = i 2 sinx+i 22 sin2x+ · · · +i 2n sinnx. Ta có the bien đői f = T + iS : f = 1 + cos x + i sin x + cos x + i sin x 2 + · · · + cos x + i sin x n 1 − cos x − cos(n + 1)x + cos(n − 1)x = 2 2n+1 5 4 − cos x 2n+2 + i . . . V y lim 1 1 cos2x+ · · · + 1 cosnx = 4 − 2cosx . n→∞ 1 + 2 cosx+ 22 2n 5 − 4cosx
  • 22. 20 Viết đề tài giá sinh viên – ZALO:0973.287.149-TEAMLUANVAN.COM − Ví dn 1.8. Cho f(x) = x3 + 15x2 + 75x + 120. Hãy giải phương trình f(f(. . . (f(f(x)) . . .)) = x và tính tőng tat cả các nghi m. ` n ˛ la ¸ nf x Bài giải. Ta có f(x) = (x +5)3 − 5. Bang quy nạp theo n ta nh n được f(f(...(f(f(x))...)) = (x + 5)3n − 5. ` n ˛ la ¸ nf x Giải ra x0 = −5 và nghi m của phương trình (x + 5)3n−1 = 1. Tóm lại x0 = −5 và xk k2π = −5+cos 3n − 1 +i sin k2π với k = 1, 2, ..., 3n 1. 3n − 1 Tőng tat cả các nghi m của phương trình T = −5.3n . 1.2 Đa thfíc chia đư ng tròn 1.2.1 Đa thfíc chia đư ng tròn Định nghĩa 1.4. Cho n là m®t so nguyên dương và α là m®t căn b¾c n của đơn v . Khi đó so nguyên dương nhó nhat k sao cho αk = 1 được goi là cap của α và kí hi u là ord(α). Ví dụ: Các căn b c 4 của đơn vị là 1, −1, i, −i. Cap của 1 là 1, cap của -1 là 2, cap của i là 4 , cap của -i là 4. Định nghĩa 1.5. Cho n là so nguyên dương và α là m®t căn b¾c n của đơn v . Khi đó α được goi là căn nguyên thủy b¾c n của đơn v neu ord(α) = n. Ví dụ: Các căn b c 3 của đơn vị là: 1 i √ 3 1 i √ 3 α0 = 1, α1 = − 2 + 2 , α2 = − 2 − 2 . Có ord(α0) = 1, ord(α1) = 3, ord(α2) = 3. Nên các căn nguyên thủy b c ba của đơn vị là α1, α2. Định nghĩa 1.6. Ta biet rang căn b¾c n của đơn v là αk = cos với k = 1, 2, ..., n và chúng l¾p thành m®t nhóm xyclic cap n. k.2π + i sin n k.2π n V y ta phải giải phương trình (x + 5)3n = x + 5.
  • 23. 21 Viết đề tài giá sinh viên – ZALO:0973.287.149-TEAMLUANVAN.COM n Q Y 3 2 2 − − 2 − 2 Đa thúc φn (x) = k=1, Y (k,n)=1 (x − αk). được goi là đa thúc chia đường tròn thú n. Ví dụ: Các căn nguyên thủy b c 3 của đơn vị là 1 i √ 3 1 i √ 3 α1 = − 2 + 2 , α2 = − 2 − 2 . Do đó đa thác chia đường tròn thá ba là: φ (x) = " x − − 1 + i √ 3 !# " x 1 i √ 3 !# = x2 + x + 1. Các căn nguyên thủy b c 4 của đơn vị là α1 = i và α3 = −i. Do đó đa thác chia đường tròn thá tư là: φ4(x) = (x − i)(x + i) = x2 + 1. Định lj 1.5. Cho n là so nguyên dương. Khi đó a) xn − 1 = φd (x) d|n b) các h so của đa thúc chia đường tròn đeu là các so nguyên, túc là φn (x) ∈ Z [x]. Chfíng minh. a) Với moi so tự nhiên k ∈ 1, 2, ..., n, (k, n) > 1, có ước d của n đe αk là căn nguyên thủy b c d của đơn vị. V y φd(x) chia het cho (x − αk). Ngược lại, moi căn nguyên thủy b c d của đơn vị, ở đó d là m®t ước của n, thì nó cũng là m®t căn b c n của đơn vị. Do v y, ta có xn − 1 = φd (x). dn b) Đa thác φ1(x) = x − 1 có h so nguyên. Giả sả đa thác φd(x) có các h so đeu là so nguyên với moi ước dương d < n của n . Vì phép chia
  • 24. 22 Viết đề tài giá sinh viên – ZALO:0973.287.149-TEAMLUANVAN.COM d|n Σ Q Q 1 Q xn − 1 φn (x) = φ d d|n,d<n (x) thực hi n trong Z[x] nên các h so của đa thác φn(x) đeu là các so nguyên. Định nghĩa 1.7. Hàm Euler ϕ : N+ → N được đ nh nghĩa như sau: ϕ(1) = 1. Cho n < 1. Khi đó ϕ(n) là so các so tự nhiên nhó hơn n và nguyên to cùng nhau với n. Ví dụ: ϕ(2) = 1, ϕ(3) = 2, ϕ(4) = 2, ϕ(5) = 5. H quả 1.2. Với moi so nguyên dương n ta luôn có n = ϕ (d). d|n Chfíng minh. Do xn − 1 = Q φn (x) theo định lý 1.5 và so sánh b c hai ve ta nh n được n = deg(xn − 1) = deg( Q φd (x)) = Σ ϕ (d) . Định lj 1.6. Với so nguyên dương lé n > 1, ta có φ2n(x) = φn(−x). Chfíng minh. Vì n là so nguyên dương lẻ nên ϕ(2n) = ϕ(n). Giả sả n = ps1 ...psr là phân tích tiêu chuȁn của n. Do n là so lẻ nên các so pi 1 r đeu là so lẻ. Vì ϕ(n) = n 1 − 1 ... 1 − 1 nên ϕ(n) là m®t so chȁn. p1 Ta thay ngay −αk = −cos pr k.2π n − i sin k.2π n với k ∈ 1, 2, ..., n, (k,n)=1, là tat cả các căn nguyên thủy b c 2n của đơn vị. Do v y φ2n(x) = n k=1,(k,n)=1 (x + αk) = (−1) ϕ(n) n k=1,(k,n)=1 −x − αk = φn (−x) V n dụng ket quả của định lý trên ta cháng minh công thác tường minh của φn(x). Định nghĩa 1.8. Hàm Mobius µ : Z+ → {−1, 0, 1} được đ nh nghĩa như sau: Đ¾t µ(1) = 1. Cho n > 1. Neu d2 không là ước của n với moi so tự nhiên d > 1 thì ta đ¾t µ(n) = (−1)k , trong đó k là so các ước nguyên to của n. Neu có so tự nhiên d > 1 sao cho d2 là ước của n thì ta đ¾t µ(n) = 0. d|n d|n
  • 25. 23 Viết đề tài giá sinh viên – ZALO:0973.287.149-TEAMLUANVAN.COM Σ Σ n d Q t t t QQ Q µ( ) d (x) = x − 1 . d d d d n t Tà định nghĩa ta có µ(6) = (−1)2 = 1, mu(9) = 0, µ(12) = 0. Hien nhiên µ là hàm nhân, tác là µ(mn) = µ(m)µ(n) với moi so nguyên dương m, n nguyên to cùng nhau. Sau đây giới thi u tính chat của hàm Mobius. M nh đe 1.7. Cho n là so nguyên dương. Khi đó a) Neu n = 1 thì µ (d) = 1. dn b) Neu n ≥ 2 thì µ (d) = 0. dn Định lj 1.7. Ta luôn có φn (x) = Q xd − 1 với moi n ∈ N∗ . µ(d ) d|n Chfíng minh. Chỉ can cháng minh các đa thác φn (x) = Q xd − 1 µ(n ) d|n thỏa mãn công thác xn − 1 = φd (x). Ta bien đői tích các đa thác d|n Y φd (x) = Y Y xt − 1 . µ(d ) d|n d|n t|d So mũ của nhân tả xt − 1 bang tőng Σ µ d = Σ µ(d′) = ( 1 neu n = t theo ket quả ve hàm Mobius. d|n,t|d d′|n 0 neu n > t Do v y (xt − 1)µ(d ) = xn − 1. Và ta được φ d|n t|d n d d|n H quả 1.3. Với moi so nguyên dương n ta có ϕ (n) = Σ dµ n . d|n Chfíng minh. Do deg(φn(x)) = ϕn nên khi lay đạo hàm hai ve của φ (x) = Q xd − 1 µ(n ) ta có ϕ (n) = Σ dµ n . d|n d|n H quả 1.4. Ta có nN (n) = Σ µ (d)p n , ớ đó N(n) là so nhân tủ bat d|n khả quy π(x) b¾c n với h tủ cao nhat bang 1 của đa thúc xpn − x. n
  • 26. 24 Viết đề tài giá sinh viên – ZALO:0973.287.149-TEAMLUANVAN.COM Y Y Y − Y µ ( ) d φn(xp ) φn(xp ) neu (p, n) = p pn d φn(xp ) d xd Định lj 1.8. Với so nguyên to p ta có: φn neu (p, n) = 1. (x) Chfíng minh. Xét trường hợp (p, n) = p. Neu α là m®t căn nguyên thủy b c pn của đơn vị thì ω = αp là m®t căn nguyên thủy b c n của đơn vị. Goi γ1w, ...γpw là nghi m của phương trình xp = ω. Khi đó p xp − ω = (x − γiω). i=1 Như v y φpn (x) = pn k=1,(k,pn)=1 (x αp ) = ω=ap (xp − ω) = φn (xp ). Xét trường hợp (p, n) = 1. Theo định lý 1.7 ta có bieu dien sau: φpn (x) = d Y |pn µ(pn ) — 1 Y d|n − pn d d|n xpd µ(n ) — 1 . pn φ (x) = Q xd − 1 −µ(n )Q xpd − 1 µ(n ) Do đó, trong trường hợp này ta có φpn (x) = H quả 1.5. Với so nguyên n > 1 có . φn (x) φn(1) = p neu n = ps 1 neu n /= ps . Chfíng minh. Neu n = ps thì φps (1) = φps (1p ) = φps−1 (1). Do v y φps (1) = ... = φp (1) = p. Neu n ps , chȁng hạn n = mps với (m, p) = 1, m > 1, thì φn (1) φn (1) = φpsm (1) = φps−1m (1). V y φpsm (1) = ... = φpm (1) = = 1. φn (1) d|n d|n d d φpn(x) = Vì µ = −µ n d nên . xd = 1
  • 27. 25 Viết đề tài giá sinh viên – ZALO:0973.287.149-TEAMLUANVAN.COM k i .c (k) . |i = 0, 1, ..., m − 1, (k, n) = 1, 1 ≤ k ≤ n 0 1 m−1 Như v y các c (k) ∈ Z đeu chia het cho p. Neu p > c ≥ .c (k) . và các c (k) Đ t c = max i . H quả 1.6. Ta có (a − 1)ϕ(n) < φn (a) < (a + 1)ϕ(n) với moi so nguyên a, n > 1. Đ¾c bi t, neu so nguyên to p|n và a ≥ 3 ta còn có φn(a) > p. Chfíng minh. Với moi so nguyên a > 1 ta có a − 1 < |a − α| < a + 1 cho moi α ∈ C, |α| = 1. Do v y a − 1 < |a − αk| < a + 1 cho moi αk thỏa mãn αn = 1 và (k, n) = 1, 1 ≤ k ≤ n. Suy ra (a − 1)ϕ(n) < φn(a) < (a + 1)ϕ(n) với moi so nguyên a, n > 1. Khi p|n thì ϕ(n) ≥ (p − 1). Khi đó φn (a) > 2ϕ(n) ≥ 2p−1 > p vì p ≥ 2. Định lj 1.9. Đa thúc φn(x) bat khả quy với moi n ∈ N∗ . Chfíng minh. Goi α là m®t căn nguyên thủy của 1 và f (x) với b c m là đa thác toi tieu monic của α trong Q[x]. Vì φn(α) = 0 nên φn(x) chia het cho f (x). Theo bő đe Gauss, đa thác f (x) ∈ Z[x]. Neu ta chỉ ra φn(x) = f (x) thì φn(x) là m®t đa thác bat khả quy. Vi c chỉ ra φn(x) = f (x) tương đương với vi c cháng minh f (αk ) = 0 khi (k, n) = 1 và k ∈ 1, 2, ..., n. Moi phan tả của Q[α] = Q[x]/(f (x)) có bieu dien duy nhat a0 + a1α + · · · + am−1αm−1 . Giả sả đa thác h(x) ∈ Z[x]. Bieu dien h(x) = q(x)f(x) + r(x) với q(x), r(x) ∈ Z[x] và deg r(x) ≤ (m − 1). Khi đó h(α) có dạng h(α) = r(α) = b0 + b1α + · · · + bm−1αm−1 , (1). Đ c bi t f αk = c(k) + c(k) α + · · · + c(k) αm−1 với tat cả các c(k) ∈ Z. ,. .0 1 m−1 , i Với so nguyên to p, (p,n)=1, ton tại so nguyên dương k thỏa mãn (k, n) = 1, p ≡ k(modn). Khi đó αp = αk . Vì f(x)p ≡ f(xp )(modp) nên f(xp ) = f(x)p + pG(x) với G(x) ∈ Z[x]. Với x = α ta có f(αp ) = pG(α). Bieu dien G(α) trong dạng (1) ta có: pG (α) = f (αp ) = f αk = c(k) + c(k) α + · · · + c(k) αm−1 . chia het cho p thì tat cả các c(k) = 0. Ta đã chỉ ra f(αp ) = f(αk ) = 0. Tóm lại neu so nguyên to p thỏa mãn (p,n)=1 và p > c thì f(αp ) = 0. i i i
  • 28. 26 Viết đề tài giá sinh viên – ZALO:0973.287.149-TEAMLUANVAN.COM k Q n kq+r k q r r Q Hoàn toàn tương tự ta cũng chỉ ra được f(αk ) = 0 với (k,n)=1. Tà đó suy ra φn(x) = f(x). 1.2.2 V n dnng Bo đe 1.2. Cho so nguyên dương r với ước nguyên dương thực sự của n và so nguyên m. Neu so nguyên to p là m®t ước so chung của φn(m) và φr(m) thì p|n. Chfíng minh. Vì xn − 1 = φd(x) và r|n, r < n nên xn − 1 chia het d|n cho φr(x)φn(x) theo định lý 1.5. Vì φn(m) ≡ 0(modp) nên φn(x) ≡ (x − m)φ(x)(modp). Tương tự φr(x) ≡ (x − m) ψ (x) (modp). Vì các đa thác φd(x) là bat khả quy theo định lý 1.9 nên (φr(x), φn(x)) = 1. Tà đây suy ra xn − 1 ≡ 0(modp) có nghi m m b®i k ≥ 2. V y p|n. Bo đe 1.3. Cho so nguyên dương n và so nguyên m. Neu so nguyên to p chia het φn(m) thì p ≡ 1(modn) ho¾c p|n. Chfíng minh. Giả sả so nguyên to p là m®t ước của φn(m). Tà p|φn(m) và φn(m)|mn − 1 suy ra p|mn − 1. V y (m, p) = 1. Theo Định lý nhỏ Fer- mat ta có mp−1 ≡ 1(modp). Chon so nguyên dương nhỏ nhat k thỏa mãn mk ≡ 1(modp). M t khác lại có mn ≡ 1(modp). Theo phép chia với dư, bieu dien n = ks + r với 0 ≤ r < k. Ta có 1 ≡ m ≡ m ≡ m m ≡ m (modp). Tà cách chon so nguyên dương k suy ra r = 0 và k|n. Tương tự ta cũng chỉ ra k|p − 1. V y neu k = n thì n|p − 1 hay p ≡ 1(modn). Neu k < n thì tà mk ≡ 1(modp) suy ra 0 ≡ m − 1 ≡ φd (m) (modp). Vì p là so nguyên to nên ta có d|k d|k đe p|φd(m). Tà d|k và k|n suy ra d|n và p|n theo bő đe 1.2. Đa thúc chia đường tròn có nhieu úng dựng, m®t trong nhũng úng dựng phő bien của nó là chúng minh đ nh lý Dirichlet. Định lj 1.10. [Dirichlet] Với so nguyên dương n có vô so so nguyên to p thóa mãn p ≡ 1(modn).
  • 29. 27 Viết đề tài giá sinh viên – ZALO:0973.287.149-TEAMLUANVAN.COM Y } } Q Q Q Y i=1 Chfíng minh. Với n = 1 ket quả đúng. Xét n > 1. Giả sả chỉ có hǎu hạn các so nguyên to p1, p2, ..., ps thỏa mãn Q k các ước nguyên to của n và đ t T = PR. Do n > 1 nên T > 1. Lay so nguyên dương đủ lớn k sao cho φn(Tk ) > 1. Goi q là m®t ước nguyên to của φn(Tk ). Vì q là ước của Tnk − 1 nên q không là ước của T và suy ra được ngay q không là ước của P và của R. Do v y q /≡ 1(modn) và q không là ước của n (mâu thuan theo bő đe 1.3). Ví dn 1.9. [Dự bị IMO 2002] Với các so nguyên to p1, p2, ..., pn > 3, so 2p1p2...pn+ + 1 có ít nhat 2 2n−1 ước. Bài giải. Theo định lý 1.5, với x = 2 ta nh n được h thác sau: (2p1p2...pn + 1) (2p1p2...pn − 1) = 22p1p2...pn − 1 = d|2p1p2...pn φd (2). Vì 2k − 1 và 2k + 1 chỉ có ước là so lẻ và (2k − 1, 2k + 1) = (2k + 1, 2) = 1 nên 2k − 1 và 2k + 1 chỉ có m®t ước dương chung bang 1. Bây giờ ta sě chỉ ra 2p1p2...pn +1 có ít nhat 2n−1 ước đôi m®t nguyên to cùng nhau và l p 22n−1 tích tà các ước đó ta sě nh n được ít nhat 22n−1 ước của so 2p1p2...pn + 1. Đ t A = d|d là ước của p1p2...pn , B = 2d|d là ước của p1p2...pn . Hien nhiên A ∩ B = ∅, Card(A) = Card(B) = 2n . Tà bieu dien dưới đây d|2p1 Q p2...pn φd (2) = d|p1p2...pn φd (2) d′|p1p2...pn φ2d′ (2) ta suy ra = (2p1p2...pn − 1) d′|p1p2...pn φ2d′ (2) (2p1p2...pn + 1) = d′|p1p2...pn φ2d′ (2). Goi A là t p tat cả các ước d của p1p2...pn sao cho d là tích m®t so chȁn các nhân tả tà t p p1p2...pn và goi B là t p tat cả các ước d của p1p2...pn pi ≡ 1(modn) với i = 1, 2, ..., s. Ký hi u P = pi và R là tích tat cả
  • 30. 28 Viết đề tài giá sinh viên – ZALO:0973.287.149-TEAMLUANVAN.COM a Q n ∈ N∗ |an − 1. p , k=1,(k,n)=1 sao cho d là tích m®t so lẻ các nhân tả tà t p p1p2...pn. khi đó A và B có cùng lực lượng và moi t p có 2n−1 phan tả. Giả sả a, b ∈ A, a /= b v à φa(2), φb(2) không nguyên to cùng nhau. Khi đó là lũy thàa nguyên b của m®t so nguyên to. Vì a, b đeu là tích của m®t so chȁn các thàa so a nguyên to phân bi t nên b không the là lũy thàa nguyên của m®t so nguyên to. V y φa(2) và φb(2) phải là nhǎng so nguyên to cùng nhau khi a, b ∈ A, a b . Vì A có 2n−1 phan tả nên 2p1p2...pn + 1 = d′|p1p2...pn φ2d′ (2) có không ít hơn 22n−1 ước. 1.3 Định lj Zsigmondy 1.3.1 Định lj Zsigmondy Bo đe 1.4. Với so nguyên to p và so nguyên a > 1, (a, p) = 1, luôn có so nguyên dương nhó nhat n đe an − 1 chia het cho p. Chfíng minh. Do p là so nguyên to và so nguyên a > 1 với (a, p) = 1 nên ap−1 − 1 chia het cho p theo Định lý nhỏ Fermat. Như v y, có so nguyên dương n đe an − 1 chia ,het cho p . Chon n là so nguyên dương nhỏ Tà bő đe này ta định nghĩa ước nguyên thủy của so nguyên dạng an ± 1. Định nghĩa 1.9. Cho hai so nguyên dương n và a > 1. So nguyên to p được goi là m®t ước nguyên thủy của an − 1 ho¾c (an + 1) neu n là so nguyên dương nhó nhat đe an ≡ 1(modp), ho¾c an ≡ −1(modp). Tà định nghĩa ta thay rang, neu p là m®t ước nguyên thủy của an − 1 thì Q n Định lj 1.11. [Zsigmondy] Neu so nguyên to p là m®t ước nguyên thủy của an − 1 thì ho¾c p|n ho¾c n|p − 1. Chfíng minh. Giả sả so nguyên to p là m®t ước của φn(m). Tà p|φn(m) và φn(m)|mn − 1 suy ra p|mn − 1. V y (m, p) = 1. Theo Định lý nhỏ nhat thu®c . Tính duy nhat của n là hien nhiên. p chỉ có the là m®t ước của so φn(a).Trong đó φn (a) = (a − αk).
  • 31. 29 Viết đề tài giá sinh viên – ZALO:0973.287.149-TEAMLUANVAN.COM Q k 2 Fermat ta có mp−1 ≡ 1(modp). Chon so nguyên dương nhỏ nhat k thỏa mãn mk ≡ 1(modp). M t khác, lại có mn ≡ 1(modp). Theo phép chia có dư, bieu dien n = ks + r với 0 ≤ r < k. Ta có 1 ≡ mn ≡ mkq+r ≡ (mk )q mr ≡ mr (modp). Tà cách chon so nguyên dương k suy ra r = 0 và k|n. Tương tự, ta cũng chỉ ra k|p − 1. V y, neu k = n thì n|p − 1 hay p ≡ 1(modp). Neu k < n thì tà mk ≡ 1(modp) suy ra 0 ≡ m − 1 ≡ φd (m) (modp). Vì p là so nguyên to nên có d|k d|k đe p|φd(m). Tà d|k và theo bő đe 1.2 ta suy ra d|n và p|n. Định lj 1.12. [Định lj Zsigmondy] Cho hai so nguyên dương a và b với (a,b)=1. Giả sủ so nguyên n ≥ 2. Khi đó: (1) an − bn có ít nhat m®t ước nguyên to không chia het ak − bk với moi so nguyên dương k<n, ngoài hai trường hợp: 26 −1 ho¾c n=2, a+b = 2s . (2) Neu a,b,n với a>b và n ≥ 2, là nhũng so nguyên dương thì an + bn có ít nhat m®t ước nguyên to không chia het ak + bk với moi so nguyên dương k<n, ngoài trường hợp: 23 + 1 = 9 chia het cho 3 = 21 + 1. H quả 1.7. Giả sủ so nguyên to p thóa mãn p|an + bn nhưng p / |ak + bk với moi k = 1, 2, ..., n − 1. Khi đó p / |aj + bj với j = n + 1, n + 2, ..., 2n. Chfíng minh. Đ t j = n + x với x = 1, 2, ..., n − 1. Ta sě chỉ ra p / |ab. Th t v y, neu p|ab thì p|a ho c p|b, chȁng hạn p|a. Khi đó p|an . Vì p|an và p|an + bn nên p|bn . Do p là so nguyên to nên p|b. Tà đây suy ra (a, b) = 1 và p|a, p|b, mâu thuan. Như v y p / |ab. Chú ý rang, p| (an + bn ) (ax + bx ) = aj + bj + ax bx (an−x + bn−x ) . Vì p / |ab, p / |(an−x + bn−x ) nên p / |(aj + bj ) với j = n + 1, n + 2, ..., 2n − 1 .Xét j = 2n, ta có p|(an + bn )2 = a2n + 2an bn + b2n . Neu p|a2n + b2n thì p|2an bn . Vì p / |ab nên p = 2. Do (a, b) = 1 và 2 = p|an + bn nên a, b cùng là hai so lẻ. Khi đó 2 = p|ak + bk với moi k = 1, 2, ..., n − 1, mâu thuan với giả thiet. V y p / |a2n + b2n . H quả 1.8. Giả sủ so nguyên p thóa mãn p|an + bn nhưng p / |ak + bk n với moi k = 1, 2, ..., n − 1. Khi đó p / |aj − bj với moi 1 ≤ j < .
  • 32. 30 Viết đề tài giá sinh viên – ZALO:0973.287.149-TEAMLUANVAN.COM | 2 Chfíng minh. Xuat phát tà p (an + bn ) an−2j + bn−2j ta suy ra ket quả p|a2n−2j + b2n−2j + an−2j a2j + b2j , (1). M t khác ta lại có p / |(an−j+bn−j)2 =a2n−2j + b2n−2j + 2an−jbn−j, (2). Lay (1) − (2) ta có p / |an−2j bn−2j (a2j +b2j − 2aj bj ). n Do v y p / |an−2j bn−2j (aj − bj )2 và suy ra p / |aj − bj với 1 ≤ j < . 1.3.2 V n dnng Định lj Zsigmondy Trong phan này giới thi u vi c v¾n dựng đ nh lý Zsigmondy đe giải các bài toán trong các đe thi hoc sinh giói và các bài toán liên quan đen so nguyên to. Ví dn 1.10. [Japanese MO 2011] Xác đ nh tat cả các b® năm so nguyên dương a, n, p, q, r thóa mãn an − 1 = (ap − 1)(aq − 1)(ar − 1). Bài giải. Hien nhiên n > p, q, r. Neu a > 2, n > 2 thì an − 1 có m®t ước nguyên to không chia het ap − 1, aq − 1, ar − 1 theo định lý Zsigmondy. V y a ≤ 2 ho c n ≤ 2. Kiem tra a = 1 . Khi đó moi m,n,p,r đủ thỏa mãn. Kiem tra a = 2. Khi đó 2n − 1 = (2p − 1) (2q − 1) (2r − 1). Tà đây suy ra {p, q, r} = {n, 1, 1}. Kiem tra n = 2, n = 1, a > 1 không thỏa mãn. Ví dn 1.11. [IMO Shortlist 2000] Xác đ nh tat cả các b® ba so nguyên dương a, m, n thóa mãn (am + 1)|(a + 1)n . Bài giải. Kiem tra (a, m, n) = (2, 3, n) với n ≥ 2 thỏa mãn đe bài. Neu a > 1, m ≥ 2 và (a, m) = / (2, 3) thì theo Định lý Zsigmondy am +1 có ước nguyên to không chia het a + 1. Do v y, am + 1 không chia het (a + 1)n và bài toán không có lời giải. Neu a = 1 ho c m = 1 thì bài toán là tam thường. Ví dn 1.12. [Polish MO 2010] Giả sủ p, q là hai so nguyên to thóa mãn q > p > 2. Chúng minh rang 2pq − 1 có ít nhat ba nhân tủ nguyên to khác nhau.
  • 33. 31 Viết đề tài giá sinh viên – ZALO:0973.287.149-TEAMLUANVAN.COM Bài giải. Hien nhiên 2pq − 1 chia het cho 2p − 1 và 2q − 1. Theo định lý Zsigmondy 2pq − 1 có m®t ước nguyên to p1 không chia het 2p − 1, 2q − 1. Hơn nǎa, 2q − 1 có m®t ước nguyên to p2 không chia het 2p − 1. Cuoi cùng, 2p − 1 có nhân tả nguyên to p3. Do v y 2pq − 1 có ít nhat ba nhân tả nguyên to phân bi t p1, p2, p3. Ví dn 1.13. Tìm tat cả b® bon so nguyên dương x, r, p, n đe p là so nguyên to và xr − 1 = p n với n, r > 1. Bài giải. Neu xp − 1 có m®t ước nguyên to q không chia het x − 1, (theo Định lý Zsigmondy luôn có q), thì xr − 1 có hai ước nguyên to phân bi t. Tà đó suy ra bieu dien xr − 1 = pn là không the xảy ra. Xét các trường hợp đ c bi t: Trường hợp x = 2; r = 6: Khi đó 26 − 1 = pn hay 32 .7 = pn . Đieu này không the xảy ra. Trường hợp x + 1 = 2s , r = 2 : khi đó x2 − 1 = pn hay (x − 1)(x + 1) = pn . Ta có 2s (2s − 2) = pn . Tà đây suy ra s = 2. V y x = 3, r = 2, p = 2, n = 3. Ví dn 1.14. [Czech - Slovak MO 1996] Xác đ nh tat cả các nghi m nguyên dương x, y, p, trong đó p là so nguyên to, thóa mãn phương trình: px − yp = 1. Bài giải. Ta có px = yp + 1. Kiem tra y = 1; khi đó px = 1p + 1 = 2. V y p = 2, x = 1. Kiem tra y = 2; p = 3: Khi đó 3x = 23 + 1 = 9 = 32 . V y x = 2. Xét trường hợp y > 1: Theo Định lý Zsigmondy, so yp + 1 có ước nguyên to không chia het y + 1. Vì yp + 1, p /= 2, có hai ước nguyên to khác nhau nên px = yp + 1 không the xảy ra. Xét trường hợp p = 2: Khi đó y2 + 1 = 2x . Như v y y là so lẻ. Đ t y = 2k + 1. Ta có 2k2 + 2k + 1 = 2x−1 . Tà đây suy ra x = 1, k = 0 và nh n được x = 1, y = 1, p = 2.
  • 34. 32 Viết đề tài giá sinh viên – ZALO:0973.287.149-TEAMLUANVAN.COM Ví dn 1.15. [Romania TST 1994] Chúng minh, dãy so (an = 3n − 2n ) không chúa ba so hạng l¾p thành cap so nhân. Bài giải. Giả sả dãy so (an) có cháa ba so hạng với chỉ so x, y, z và x < y < z, tương áng l p thành m®t cap so nhân. Khi đó ta có h thác (3y − 2y )2 = (3x − 2x ) (3z − 2z ) (∗). Theo Định lý Zsigmondy, so 3z − 2z có ước nguyên to p không chia het cho 3y − 2y và (*) không the xảy ra. Như v y đieu giả sả là sai. Ví dn 1.16. [Italy TST 2003] Xác đ nh tat cả các b® ba so nguyên dương (a, b, p) thóa mãn 2a + pb = 19a . Bài giải. Giả sả có b® ba so nguyên dương (a, b, p) thỏa mãn 2a + pb = 19a . De dàng suy ra 17 = 19 − 2 là m®t ước nguyên to của 19a − 2a = pb . Tà 17|pb suy ra p = 17. Khi đó ta có h thác 19a − 2a = 17b . Với a = 1 ta có b = 1 . Với a ≥ 2, theo định lý Zsigmondy so 19a − 2a có ước nguyên to p không chia het 17 = 19 − 2. Do v y ước nguyên to p không chia het cho 17p , vô lý. V y a không the khác 1. Ví dn 1.17. Xác đ nh tat cả các so nguyên không âm m, n đe 3m − 5n là so chính phương. Bài giải. Lay theo modulo 4 ta có (−1)m − 1 ≡ x2 (mod4). Do v y m phải là so chȁn. Đ t m = 2k với k ∈ N. Khi đó ta có h thác 32k − 5n = x2 hay (3k − x)(3k + x) = 5n . Đ t 3k − x = 5r và 3k + x = 5s với s > r . Ta cũng có 2.3k = 3k − x + 3k + x = 5r + 5s = 5r (1 + 5s − r ). Tà đây suy ra r = 0. Khi đó 2.3k = 1 + 5s . Neu k = 0 thì m = 0, s = 0 và ta có m = n = 0, x = 0. Neu k > 0 thì ta xét 2.3k = 1 + 5s . Với s ≥ 3, theo Định lý Zsigmondy 2.3k = 1 + 5s có ước nguyên to p không chia het 1 + 5 = 6|2.3k , vô lý. V y s = 1, k = 1 và suy ra m = 2, 32 − 51 = 4 = 22 . Với s = 2 có 2.3k = 1 + 52 = 26, vô nghi m. Tóm lại m = 2, n = 1 ho c m = n = 0 và coi 0 cũng là m®t so chính phương.
  • 35. 33 Viết đề tài giá sinh viên – ZALO:0973.287.149-TEAMLUANVAN.COM Ví dn 1.18. [Vi t Nam TST 2016] Tìm hai so nguyên dương a,n với a > 2 sao cho mői ước nguyên to của an − 1 cũng đeu là ước nguyên to của a32016 − 1. Bài giải. Có ước chung lớn nhat an − 1, a32016 − 1 = a(n,32016 ) − 1 ; ước chung lớn nhat của xm − 1 và xn − 1 đúng bang x(m,n) − 1 suy ra ket lu n đúng cho moi so nguyên a > 2 khi n = 1. Xét n > 1. Đ t n = 3s m với (m, 3) = 1. Khi đó ta có: an − 1, a32016 − 1 = a3r − 1, ở đó r = min {s, 2016}. Xét trường hợp: s > 0. Neu m > 1 thì n > 3r . Khi đó an − 1 có m®t ước nguyên to p không là ước của a3r − 1 . Do v y, trường hợp này không the xảy ra . Tà đó suy ra m = 1. Ta có an − 1 = a3r − 1. Đe moi ước nguyên to của an − 1 cũng là ước của a32016 − 1 thì r ≤ 2016. V y (a, n) = (a, 3r ) với so nguyên a > 2 và r ≤ 2016. nguyên to của an − 1 cũng là ước của a32016 − 1 thì n = 1. V y (a, n) = (a, 1) với a > 2. Xét trường hợp n = 2 và a = 2s − 1 với s > 1. Trường hợp này yêu cau bài toán luôn thỏa mãn. = a − 1. Đe moi ước an − 1, a32016 − 1 Xét trường hợp s = 0. Khi đó
  • 36. 34 Viết đề tài giá sinh viên – ZALO:0973.287.149-TEAMLUANVAN.COM Σ Chương 2 Tính chat so hoc của đa thfíc Trong chương 2 này, dự kien trình bày m®t so tính chât so hoc của đa thác. Các kien thác trong chương này sě được tham khảo tà tài li u [1] và [2]. 2.1 Tính chat đ c bi t của đa thfíc thu c Z[x] 2.1.1 Định lj Bézout Định lj 2.1. Cho đa thúc f(x) ∈ Z[x]. Khi đó, với moi a, b ∈ Z ta luôn có a − b|f(a) − f(b). Chfíng minh. Giả sả f(x) = a0xn + a1xn−1 + · · · + an ∈ Z[x]. Với a, b ∈ Z ta có f(a) − f(b) = n−1 k=0 ak(a k — bk ) chia het cho a − b. Chý ý rang, khi a = b ta có a − b = 0 và f(a) − f(b) = 0. Do 0 = 0.0 nên f(a) − f(b) cũng van chia het cho a − b = 0. Giả sả đa thác f (x) = a0xn + a1xn−1 + · · · + an ∈ Z[x]. Khi đó, có nhieu ket quả thường được sả dụng. Trước tiên, ta công nh n định lý Định lj 2.2. [Dirichlet] Cho đa thúc b¾c nhat f (x) = ax+b với a, b ∈ N∗ và (a, b) = 1. Khi đó, t¾p giá tr {f(n)|n ∈ N∗ } chúa nhieu vô hạn so nguyên to. Định lj 2.3. Cho đa thúc f(x) ∈ Z[x] với f(0) /= 0 và deg f(x) ≥ 1. Khi đó, với mői so nguyên dương k luôn luôn có so nguyên to p > |f(0)|
  • 37. 35 Viết đề tài giá sinh viên – ZALO:0973.287.149-TEAMLUANVAN.COM đe so f(p) có ít nhat k thùa so nguyên to phân bi t. Chfíng minh. Với k = 1, ton tại so nguyên to p1 > |f(0)| đe |f(p1)| = 1 bởi vì neu khác đi thì f(p)2 = 1 cho moi so nguyên to p. Khi đó đa thác f(x)2 − 1 có nhieu vô hạn nghi m. Đa thác này phải là đa thác 0 và ta có f(x) ≡ 1 ho c f(x) ≡ −1, mâu thuan với đieu ki n deg f(x) ≥ 1. Do f (p1) là so nguyên khác ±1 nên f (p1) có ít nhat m®t ước nguyên to và ket lu n đúng cho k = 1. Giả sả so nguyên to pk thỏa mãn pk > |f(0)| và f(pk) có ít nhat k thàa so nguyên to phân bi t. Chú ý rang, (pk, f(pk) = 1 vì f(pk) ≡ f(0)(mod pk) theo Định lý 2.1 và vì pk > |f(0)| là so nguyên to. Xét cap so c®ng {f(pk)2 n + pk|n ∈ N∗ }. Theo Định lý Dirichlet, Định lý 2.2, dãy này có cháa nhieu vô hạn so nguyên to. Giả sả pk+1 = f (pk)2 m + pk là m®t so nguyên to. Khi đó f(pk+1) − f(pk) chia het cho pk+1 − pk = f(pk)2 m theo Định lý 2.1. Do v y f(pk+1) ≡ f(pk)(mod f(pk)2 ). Tà đây suy ra sự ton tại của so nguyên r đe f (pk+1) = f (pk)(1 + rf (pk)). Tích này chỉ ra rang, f (pk+1) có ít nhat m®t ước nguyên to nhieu hơn so ước nguyên to của f(pk) hay f (pk+1) có ít nhat k + 1 ước nguyên to. Tóm lại, ket lu n đã được cháng minh. Định lj 2.4. [Schur] Giả sủ đa thúc f(x) b¾c n ≥ 1 thu®c vành Z[x]. Khi đó có vô hạn so nguyên to chia het toi thieu m®t so hạng khác không trong dãy (f(n)) với n ∈ N∗ . Chfíng minh. Không hạn che ta có the giả thiet f (0) = 1. Vì f (x) = 1 chỉ có m®t so hǎu hạn nghi m nên ton tại so nguyên M đe f (n) /= 1 với moi so nguyên n > M. Hien nhiên f (n!) ≡ 1( mod n!). Khi lay so nguyên dương n đủ lớn, ta có the xây dựng nhǎng so nguyên to tùy ý chia het f(n!). Neu f(0) = 0 thì ket quả là hien nhiên. f(xf(0)) Neu f(0) /= 0 thì ta xét g(x) = và ta có g(0) = 1 và v n dụng f(0) l p lu n trên ta chỉ ra được ket quả nêu ra ở trên của Schur.
  • 38. 36 Viết đề tài giá sinh viên – ZALO:0973.287.149-TEAMLUANVAN.COM M®t ket quả khá női tieng, Hensel’s Lemma, cũng được nêu ra (không cháng minh) sau đây. Định lj 2.5. [Hensel’s Lemma] Giả sủ a0, a1, . . . , an là nhũng so nguyên và đa thúc f (x) = anxn + · · · + a1x + a0 với đạo hàm f ′(x). Giả sủ so nguyên to p và so nguyên x1 thóa mãn f (x1) ≡ 0(modp), f ′(x1) /≡ 0(modp). Khi đó, với mői so nguyên k có duy nhat m®t th¾ng dư xk(modpk ) đe f(xk) ≡ 0(modpk ) và xk ≡ x1(modp). 2.1.2 V n dnng Ví dn 2.1. [RMC 2010] Xác đ nh tat cả các đa thúc f (x) ∈ Z[x] thóa mãn đieu ki n: Ton tại so nguyên dương k đe với moi so nguyên to p, so f(p) có nhieu nhat k thùa so nguyên to phân bi t. Bài giải. Giả sả đa thác f(x) ∈ Z[x] thỏa mãn đieu ki n đau bài, có nghĩa: Có so nguyên k đe f(p) có nhieu nhat k thàa so nguyên to phân bi t cho moi so nguyên to p. Giả sả f(0) = / 0. Với so nguyên m > k có so nguyên to q > |f(0)| đe f(q) có ít nhat m > k ước so nguyên to phân bi t theo Định lý 2.3, mâu thuan. Do v y, đieu giả sả là sai và ta có f (0) = 0. Bieu dien f (x) = xr g(x) với g(0) /= 0. Neu g(x) không l à hang so thì ta l p lại quá trình trên và suy ra mâu thuan. Tà đó suy ra f(x) = axm với m ∈ N và a ∈ N∗ . Ví dn 2.2. Giả sủ đa thúc f (x) ∈ Z[x] nh¾n giá tr ±1 tại 3 điem nguyên khác nhau. Chúng minh rang, f(x) không the có nghi m nguyên. Bài giải. Giả sả có bon so nguyên phân bi t a, b, c, d đe f (a), f (b), f (c) thu®c t p {−1, 1} và f (d) = 0. Khi đó a − d, b − d, c − d là ba so nguyên phân bi t đôi m®t cùng là ước của 1, vô lý. Ví dn 2.3. Cho dãy các so nguyên (an) với tính chat m − n|am − an với moi m, n ∈ N và m n. Giả sủ có đa thúc p(x) đe |an| < p(n) với moi n ∈ N. Chúng minh, có đa thúc q(x) thóa mãn an = q(n) với moi n.
  • 39. 37 Viết đề tài giá sinh viên – ZALO:0973.287.149-TEAMLUANVAN.COM 2 2 2 2 Bài giải. Đ t d = deg p(x). De dàng chỉ ra m®t đa thác q(x) với deg q(x) ≤ d thỏa mãn q(k) = ak với moi k = 1, 2, . . . , d + 1. Ta sě chỉ ra q(n) = an với moi n ∈ N∗ . Xét n > d + 1. Đa thác q(x) có the không thu®c Z[x], nhưng chac chan thu®c Q[x]. Chon so nguyên M đe g(x) = Mq(x) ∈ Z[x]. Tích M(an − q(n)) = Man − Mq(n) = M(an − ak) − (g(n) − g(k)) chia het cho n − k theo giả thiet cho k = 1, 2, . . . , d + 1. Như v y, ho c an = q(n) ho c b®i chung nhỏ nhat Ln của các so n−1, n−2, . . . , n−d−1 không vượt quá M (an − q(n)) hay Ln ≤ M (an − q(n)) < cnd , trong đó hang so c không phụ thu®c n. Ví dn 2.4. Giả sủ đa thúc f(x) = x2018 + x2017 + 2 2019 = a + a1x + a2x2 + · · · + anxn . Hãy tính tőng T = a a1 a2 — − + a a4 a5 — − + a — · · · . 2π Bài giải. Với α = cos 3 2π + i sin 3 /= 1 ta có α3 = 1, α2 + α = −1. Tính 1 = f(α) = a0 + a1α + a2α2 + a3 + a4α + a5α2 + a6 + · · · 1 = f(α2 ) = a0 + a1α2 + a2α + a3 + a4α2 + a5α + a6 + · · · . V y 2 = 2a0 −a1 −a2 +2a3 −a4 −a5 +2a6 −· · · và suy ra tőng T = 1. Ví dn 2.5. Giả sủ f (x) = x3 − 3x. Xây dựng dãy truy hoi qua vi c đ¾t f1(x) = f (x) và fn+1(x) = f1(fn(x)) với n = 1, 2, 3, . . . Chúng minh rang, phương trình đa thúc fn(x) = x có 3n nghi m thực phân bi t. Bài giải. Trước tiên ta chỉ ra fn(x) là đa thác với h so nguyên b c 3n . Vì f1(x) = x3 − 3x nên ket lu n đúng cho n = 1. Giả sả fn(x) là đa thác với h so nguyên b c 3n . Khi đó fn+1(x) = f1(fn(x)) = (fn(x))3 − 3fn(x) là đa thác với h so nguyên b c 3n+1 và ket quả đã được cháng minh. Xét x = x(t) = 2 cos t với 0 ≤ t ≤ π. Ta có −2 ≤ x ≤ 2. De dàng kiem tra f (x) = 2 cos 3t và f2(x) = 2 cos 32 t. Qua quy nạp theo n, ta nh n được ket quả fn(x) = 2 cos 3n t. 0 0 3 6
  • 40. 38 Viết đề tài giá sinh viên – ZALO:0973.287.149-TEAMLUANVAN.COM ∈ − Xét phương trình fn(x) = x với x = 2 cos t. Ta có cos 3n t = cos t. 2kπ Giải ra t = 3n + 1 2kπ và t = 3n − 1 với k = 0, 1, 2, . . . , 3n với chú ý 2.0.π 3n + 1 2.0.π = 3n − 1 . Ta nh n được 3n nghi m phân bi t. Ví dn 2.6. Giả sủ đa thúc f(x) Z[x] thóa mãn f(f(. . . f(m) . . .)) = m n lan f với so nguyên m và so nguyên n ≥ 2. Chúng minh rang f(f(m)) = m. Tù đó suy ra so nghi m nguyên của phương trình f(f(. . . f(x) . . .)) = x 2017 lan f với f(x) = x2 + 3x − 4 không lớn hơn 2. Bài giải. Xét dãy (mk) với m0 = m và mk+1 = f(mk) với so nguyên k ≥ 0. Giả sả mr = m0 = m. Theo Định lý 2.1 ta có dk = mk+1 − mk|f(mk+1) − f(mk) = mk+2 − mk+1 = dk+1 cho moi so nguyên k. Ket hợp với mr = m0 = m ta suy ra dr = d0 và nh n được |d0| = |d1| = · · · = |dr|. Giả sả d1 = d0 = d 0. Khi đó d2 = d. Tương tự d3 = d, . . . V y mk = m0 + kd /= m0 cho moi k, mâu thuan. Tà đó suy ra d1 = −d0 và suy ra m2 = m0 = m. Giả sả x = m là nghi m nguyên của phương trình f(f(. . . f(x) . . .)) = x. 2017 lan f Khi đó m phải là nghi m nguyên của phương trình f(f(x)) = x. Ta có x4 + 6x3 + 4x2 − 16x = 0. Phương trình có hai nghi m nguyên x = 0 và x = 4. V y, so nghi m nguyên của phương trình f(. . . f(x) . . .) = x 2017 lan f với f(x) = x2 + 3x − 4 không lớn hơn 2. Ví dn 2.7. Cho so nguyên m và đa thúc f(x) = anxn + · · · + a1x + a0 ∈ R[x] thóa mãn đieu ki n f(x) là so nguyên chia het cho m khi x nh¾n giá tr nguyên. Chúng minh rang, n!an chia het cho m. 1 Bài giải. Xét đa thác 1 f(x). Vì f(x) là so nguyên chia het cho m khi m x là so nguyên nên f(x) nh n giá trị nguyên khi x là so nguyên. Theo m
  • 41. 39 Viết đề tài giá sinh viên – ZALO:0973.287.149-TEAMLUANVAN.COM n 0 Định lý ??, có các so nguyên b0, b1, . . . , bn đe 1 f(x) viet được thành m 1 f(x) = b0 m x + b x n − 1 + · · · + bn x . H so cao nhat của đa thác này bang b0 + nb1 + · · · + n!bn. Do v y, n!an chia het cho m. Ví dn 2.8. Xác đ nh tat cả các so nguyên dương n đe sao tat cả các đa thúc f (x) = (x − a1)(x − a2) . . . (x − an) + 1 là bat khả quy với moi n so nguyên phân bi t đôi m®t a1, a2, . . . , an. Bài giải. Với n = 1, đa thác f (x) = x − a + 1 luôn là bat khả quy. Với n = 2, đa thác f (x) = (x − a)(x − a − 2) + 1 = (x − a − 1)2 là khả quy. Với n = 4, đa thác f (x) = (x − a)(x − a − 1)(x − a − 2)(x − a − 3) + 1 hay f(x) = [(x − a − 1)(x − a − 2) − 1]2 là khả quy. Xét n = 3 ho c n ≥ 5. Giả sả đa thác f(x) là khả quy trong Z[x], có nghĩa: Có hai đa thác g(x), h(x) b c lớn hơn 0 và g(x), h(x) ∈ Z[x] đe f(x) = g(x)h(x). Khi đó g(ai)h(ai) = 1 với moi i = 1, 2, . . . , n. Như v y, g(ai) = h(ai) = ±1 với moi i = 1, 2, . . . , n. Do b c của g(x), h(x) đeu lớn hơn 0 và bang nhau tại n điem phân bi t ai, trong đó n > deg g(x), deg h(x), nên g(x) = h(x). Ta có f(x) = g(x)2 . Tà đây suy ra n phải là so chȁn, n = 2m, và khi n là so lẻ thì f(x) là bat khả quy. Viet [g(x) + 1][g(x) − 1] = (x − a1)(x − a2) . . . (x − a2m), m ≥ 3. Sap xep lại, neu can, ta có the viet g(x) + 1 = (x − a1)(x − a3) . . . (x − a2m−1) g(x) − 1 = (x − a2)(x − a4) . . . (x − a2m) với a1 > a3 > · · · > a2m−1 và a2 > a4 > · · · > a2m. Trà hai ve, ta có 2 = (x − a1)(x − a3) . . . (x − a2m−1) − (x − a2)(x − a4) . . . (x − a2m). V y 2 = (a2k − a1)(a2k − a3) . . . (a2k − a2m−1). 1
  • 42. 40 Viết đề tài giá sinh viên – ZALO:0973.287.149-TEAMLUANVAN.COM Σ Σ n n k n Σ = c(n, k) k k Ta có the coi a2 là so lớn nhat. Vì 2 = 1.2 ho c 2 = (−1)(−2) nên vi c bieu dien 2 = (a2 − a1)(a2 − a3) . . . (a2 − a2m−1) bang tích m ≥ 3 thàa so khác nhau là không the xảy ra. Đieu này chỉ ra đieu giả sả là sai. Đa thác f(x) là bat khả quy. Tóm lại, với n = 1, n = 3, n ≥ 5, đa thác f (x) = (x − a1)(x − a2) . . . (x − an) + 1 là bat khả quy với moi n so nguyên phân bi t đôi m®t a1, a2, . . . , an. Ví dn 2.9. Giả sủ xn n > 2017. n = k=0 n c(n, k) Σ n x . Tính x c(n, 2017) + c(n, 2018) khi c(n + 1, 2018) xn+1 = n k=0 Σ c(n, k) x x = x Σ k=0 c(n, k) Σ x (x + 1 − 1) x = k=0 c(n, k)(k + 1) x + 1 k + 1 — Σ k=0 c(n, k) x . Do v y xn+1 = Σ k=0 c(n, k)(k + 1) x k + 1 + x i − Σ k=0 c(n, k) x = k=0 c(n, k − 1)k + (k + 1 − 1)c(n, k) x = k k=0 c(n, k − 1) + c(n, k) x . Tà đây suy ra c(n + 1, k) = k c(n, k − 1) + c(n, k) . Ta nh n được c(n, k − 1) + c(n, k) c(n + 1, k) 1 = . Khi so nguyên n > 2017 ta nh n được ket k quả c(n, 2017) + c(n, 2018) = c(n + 1, 2018) 1 . 2018 Ví dn 2.10. Cho so tự nhiên n > 1. Chúng minh rang, đa thúc thuan nhat hai bien f(y, z) = yn + 5yn−1 z + 3zn là bat khả quy trong Z[y, z]. k n n k k n k k k=0 k k=0 k n k k=0 n n h Bài giải. Với bieu dien x = c(n, k) ta có the bien đői tőng (x + 1) − c(n, k) Σ Σ
  • 43. 41 Viết đề tài giá sinh viên – ZALO:0973.287.149-TEAMLUANVAN.COM − αi (αi +5) = = g( 5) . Tà g( 5)h( 5) = k( i=1 Bài giải. Bieu dien f(y, z) = zn xn + 5xn−1 + 3 với x = y . Ta chỉ z can cháng minh k(x) = xn + 5xn−1 + 3 là bat khả quy trong Z[x]. Với n = 2, đa thác k(x) = x2 + 5x + 3 không có nghi m trong Z. Đa thác này là bat khả quy. Với n > 2, giả sả đa thác k(x) = g(x)h(x), trong đó g(x), h(x) ∈ Z[x] và deg g(x), deg h(x) > 1 do k(x) không có nghi m trong Z. Do g(0)h(0) = k(0) = 3 nên ho c |g(0)| = 1 ho c |h(0)| = 1, chȁng hạn |g(0)| = 1. Bieu dien đa thác g(x) = xs + a1xs−1 + · · · + as, s > 1, |g(0)| = 1. Goi α1, . . . , αs là nghi m của g(x) = 0, trong đó các nghi m có the là Q s 0 ta suy ra k(αi) = 0 với moi i = 1, 2, . . . , s. De dàng chỉ ra 3s = Q s | n−1 | Q s | | | − | − − − nên |g(−5)| = 1 ho c |g(−5)| = 3. Do s > 1 nên |g(−5)| = 3s là sai. V y đieu giả sả k(x) khả quy là sai. Tà đó suy ra k(x) bat khả quy. Ví dn 2.11. Cho so tự nhiên n > 1. Chúng minh, đa thúc thuan nhat hai bien f(y, z) = y5 − yz4 + nz5 là bat khả quy trong Z[y, z] khi 5 / |n. Bài giải. Bieu dien f(y, z) = z5 x5 x+n với x = y . Ta chỉ can cháng z minh k(x) = x5 − x + n là bat khả quy trong Z[x] khi 5 / |n. Giả sả k(x) = (x − m)h(x). Khi đó k(m) = 0 hay m5 − m = −n. Vì m ∈ N∗ và 5 là so nguyên to nên m5 − m ≡ 0( mod 5) theo Định lý nhỏ Fermat. Tà đây suy ra 5|n, mâu thuan với giả thiet. Giả sả k(x) = (x2 −bx−c)h(x). Khi đó x5 −x+n chia het cho x2 −bx−c. Thực hi n phép chia x5 − x + n cho x2 − bx − c ta nh n được thương x3 + bx2 +(b2 + c)x + b3 + 2bc và đa thác dư (b4 + 3b2 c + c2 − 1)x +(b3 c + 2bc2 + n). Do đa thác dư phải bang 0 nên b4 + 3b2 c + c2 − 1 = 0 b3 c + 2bc2 + n = 0. i=1 thực hay phác. Ta có |αi| = |g(0)| = 1. Tà k(αi) = g(αi)h(αi) = i=1 (αi +5) 5) = 3
  • 44. 42 Viết đề tài giá sinh viên – ZALO:0973.287.149-TEAMLUANVAN.COM − − — − z − — − Σ t − z |z|2 1 Suy ra b(b4 + 3b2 c + c2 − 1) − 3(b3 c + 2bc2 + n) = 0 hay b5 − b − 5bc2 = 3n. Do b, c nguyên và b5 − b chia het cho 5 nên n˙:5, vô lý. V y, đieu giả sả k(x) khả quy là sai. Tà đó suy ra k(x) bat khả quy. Ví dn 2.12. Chúng minh rang, đa thúc f (x, y) = xn − xyn−1 − yn là bat khả quy trong Q[x, y] cho moi so nguyên dương n. Bài giải. Bieu dien f(x, y) = yn tn t 1] với t = x . Do v y, ta chỉ y can cháng minh g(t) = tn t 1 là đa thác bat khả quy trong Q[t]. Nh n xét: Neu z là nghi m của tn − t − 1 thì 2Re z − 1 > 1 − 1. Th t v y, viet z = reiu với r ≥ 0. Ta thay ngay r = 0 là không thỏa mãn. Neu r = 1 thì cos nu = cos u + 1 và sin nu = sin u. Tà đó suy ra 1 = cos2 nu + sin2 nu = (1 + cos u)2 + sin2 u. Tà đây suy ra mâu thuan. Bat đȁng thác can xét sě trở thành (1 + 2r cos u)(r2 − 1) > 0. Do r2n = |z|2n = |z + 1|2 = 1 + 2r cos u + r2 nên r2n − r2 = 1 + 2r cos u, và ta nh n được (1 + 2r cos u)(r2 − 1) = (r2n − r2 )(r2 − 1) > 0. Ta có 2Re z − = 2r cos u. và 2Re z − > 1 r2 1 1 r2 1 1 tương đương (1 + 2r cos u)(r2 1) > 0, đúng. |z|2 Giả sả F(t) = tn − t − 1 = g(t)h(t) trong Q[t]. Theo Bő đe Gauss, ta có the xét g(t), h(t) ∈ Z[t]. Hien nhiên tn − t − 1 = 0 không có nghi m trong Q. Ta có F ′(t) = ntn−1 − 1. Neu t là nghi m của tn − t − 1 = 0 thì tn = t + 1. Giả sả t cũng là nghi m của F ′(t) = ntn−1 − 1 = 0. Khi đó n(t + 1) = t và suy ra t ∈ Q, vô lý. Do v y, tat cả các nghi m của F (t) = 0 là khác nhau đôi m®t. Giả sả F(t) có n nghi m phác phân bi t t1, . . . , tn và giả sả t1, . . . , ts là nghi m của g(t) = 0. Ta có |t1| . . . |ts| = |g(0)| = 1. Viet g(t) = ts + a1ts−1 + · · · + as−1t ± 1. Ta có t1 +· · ·+ts = −a1 và 1 1 t +· · ·+ t = ±as−1. V y s j=1 1 j t1 ∈ Z. Theo z s
  • 45. 43 Viết đề tài giá sinh viên – ZALO:0973.287.149-TEAMLUANVAN.COM ‚ .Y , Σ − − Σ t − tj − t — u = −1. V y tj − t Σ Σ j nh n xét trên, qua bat đȁng thác Cauchy ta có the đánh giá tőng 2 Re j=1 1 tj − t > j=1 1 |tj|2 s — s ≥ s s j=1 1 |tj|2 — s = s − s = 0. Vì các tj phân bi t nên dau = không xảy ra. Vì g(t) là đa thác monic Σ s 1 với h so nguyên nên Σ n Re j=1 tj − t 1 ≥ 1. Tương tự, l p lu n cho đa Hien nhiên, với n = 2 đa thác t2 − t − 1 = 0 là bat khả quy. Xét n ≥ 3. Vì t1 , . . . , tn là nghi m của tn t 1 nên tj j=1 1 = 0 và tj là nghi m của n−1 n 1 Σ n 1 thuan với ket quả n Re j=1 1 j tj ≥ 1 + 1 = 2. Tà đây suy ra, đieu giả sả tn − t − 1 khả quy là sai. V y, đa thác tn − t − 1 là bat khả quy. Ví dn 2.13. Cho so tự nhiên n > 1. Chúng minh rang, đa thúc thuan nhat hai bien f(y, z) = (y2 + z2 )n + pz2n là bat khả quy trong Z[y, z] khi p = 4k + 3 là so nguyên to với so nguyên không âm k. Bài giải. Bieu dien f(y, z) = z2n (x2 + 1)n + p với x = y . Ta chỉ can z cháng minh k(x) = (x2 +1)n +p là bat khả quy trong Z[x] khi p = 4k+3 là so nguyên to. Giả sả k(x) = g(x)h(x) với đa thác g(x), h(x) ∈ Z[x] và deg g(x) ≥ 1, deg h(x) ≥ 1. Ký hi u g(x), h(x) là đa thác g(x), h(x) xét trong Zp[x]. Khi đó g(x).h(x) = (x2 + 1)n . Đa thác x2 + 1 là bat khả quy do −1 không là th ng dư b c hai trong Zp. V y, có so nguyên dương s đe g(x) = (x2 +1)s và h(x) = (x2 +1)n−s , trong đó 1 ≤ s ≤ n−1. Tà đây suy ra, có đa thác với h so nguyên g1(x) và h1(x) đe g(x) = (x2 +1)s +pg1(x) và h(x) = (x2 + 1)n−s + ph1(x). Nhân đa thác ta có (x2 + 1)n + p = g(x)h(x) = [(x2 + 1)s + pg1(x)][(x2 + 1)n−s + ph1(x)] j j=1 j t j=1 j s s thác h(t). Như v y j=1 Re ≥ 1 + 1 = 2. 1 + u nên = 1. Đieu này mâu j Σ
  • 46. 44 Viết đề tài giá sinh viên – ZALO:0973.287.149-TEAMLUANVAN.COM 2 2 — − hay (x2 +1)s h1(x)+(x2 +1)n−s g1(x)+pg1(x)h1(x) = 1. Lay theo modulo p ta thay ngay đa thác x2 + 1 là ước của 1 trong Zp[x]. Tà đieu mâu thuan này suy ra rang, giả sả k(x) khả quy là sai. Tà đó suy ra k(x) bat khả quy. Định nghĩa 2.1. Hàm Z(n) = min , m|m ∈ N∗ , n| m(m + 1), được goi là hàm pseudo Smarandache. Ví dụ Z(1) = 1, Z(2) = 3, Z(3) = 2, Z(4) = 7, Z(5) = 4, Z(6) = 3, Z(7) = 6, Z(8) = 15, Z(9) = 8, Z(10) = 4, Z(11) = 10, Z(12) = 8, Z(13) = 12, Z(14) = 7, Z(15) = 5, Z(16) = 31, Z(17) = 16, Z(18) = 8, Z(19) = 18, Z(20) = 15. Bo đe 2.1. Với moi so nguyên lé n ≥ 3 ta có Z(n) ≤ n − 1 và với moi so nguyên to p > 2 có Z(p) = p − 1. Chfíng minh. Vì (n − 1)n 2 chia het cho n nên Z(n) ≤ n − 1 với moi so nguyên lẻ n ≥ 3. Vì p > 2 và p là so nguyên to nên p là so lẻ. V y 1 + 2 + · · · + p − 1 = p(p − 1) là so nguyên dương chia het cho p. V y Z(p) ≤ p 1. Kiem tra, với moi so nguyên dương k < p 1 có k(k + 1) 1 + 2 + · · · + p − 1 = đeu không chia het cho so nguyên to p 2 vì k, k + 1 đeu không chia het cho p. Do v y Z(p) = p − 1 cho moi so nguyên to p > 2. Định lj 2.6. Với so dương M đủ lớn luôn có nhieu vô hạn so nguyên dương n đe |Z(n + 1) − Z(n)| > M và Z(n + 1) > M. Z(n) Chfíng minh. Cho so dương bat kỳ M đủ lớn, ta chon m ∈ N∗ đe 2m > M. Vì (22m+1 , 2m + 1) = 1 nên có nhieu vô hạn so nguyên to dạng 22m+1 k + 2m + 1 theo Định lý 2.1. Như v y, có so nguyên dương k0 đe p = 22m+1 k0 + 2m + 1 là m®t so nguyên to. Với so nguyên to p ta có Z(p) = p − 1 = 22m+1 k0 + 2m theo Bő đe 2.2. Bien đői
  • 47. 45 Viết đề tài giá sinh viên – ZALO:0973.287.149-TEAMLUANVAN.COM n n n n! Σ 2m+1k0 m+1 m+1 Z(p − 1) = Z 2m (2m+1 k0 + 1) và i Σ =1 i = 2 k0(2 k0 + 1) . 2 2m+1k0 Hien nhiên 2m (2m+1 k0 + 1) chia het 2m k0(2m+1 k0 + 1) = i. i=1 Tà đó suy ra Z(p − 1) ≤ 2m+1 k0. Với các đánh giá này ta nh n được ket quả Z(p) Z(p − 1) 22m+1k0 + 2m 2m+1k0 > 2m > M |Z(p) − Z(p − 1)| ≥ Z(p) − Z(p − 1) = 22m+1 k0 + 2m − 2m+1 k0 = 2m+1 k0(2m − 1) + 2m > 2m > M. Z(n + 1) Đieu này chỉ ra |Z(n + 1) − Z(n)| và không bị ch n trên. Z(n) 2.2 Đa thfíc Hilbert và bieu dien Mahler Xét đa thúc X = X(X − 1)(X − 2) · · · (X − n + 1) , được goi là các đa thúc Hilbert thú n. Các đa thúc này có các h so hũu ty (không nhat thiet nguyên) và nó nh¾n các giá tr nguyên tại moi so nguyên. Chú ý x(x − 1) · · · (x − n + 1) = n! x neu x ≥ 1, x(x − 1) · · · (x − n + 1) = (−1)n −x + n − 1 neu x < 0. Các đa thác đó đóng m®t vai trò cơ bản trong lý thuyet các đa thác nh n giá trị nguyên do ket quả sau đây của Polya: Định lj 2.7. Giả sủ f là m®t đa thúc b¾c n với các h so thực. Khi đó f (X) nh¾n giá tr nguyên neu và chí neu ton tại các so nguyên a0, a1, a2, . . . , an sao cho ≥
  • 48. 46 Viết đề tài giá sinh viên – ZALO:0973.287.149-TEAMLUANVAN.COM n k n k k k d k 1 1 2 n 1 2 n k f(X) = a + a X + a X + · · · + a X . Chfíng minh. Giả sả rang f(Z) ⊂ Z. Các đa thác X , X , · · · , X có b c 0, 1, . . . , n, do đó chúng tạo thành m®t cơ sở của R-không gian vectơ của các đa thác với h so lớn nhat là n. Do đó ton tại duy nhat các so thực a0, a1, a2, . . . , an sao cho f(X) = Σ k=0 ak · X . Xét toán tả ∆f(X) = f(X + 1) − f(X) và quan sát rang ∆ X = X . n n − 1 Tác đ®ng toán tả ∆ liên tiep vào quan h f(X) = Σ k=0 ak · X . ta suy ra aj = ∆j f(0) với moi j. Bang quy nạp toán hoc ta có ∆k f(X) = Σ j=0 (−1)k−j k f(X + j). Do đó, neu f(0), f(1), . . . , f(n) là các so nguyên, thì a0, a1, a2, . . . , an cũng là các so nguyên. Phép cháng minh của định lý trên chỉ ra rang ak = Σ j=0 (−1)k−j k f(j). Ta goi aj là các h so Mahler của f. Tà Định lý 2.7 ta có m nh đe sau. M nh đe 2.1. Giả sủ d = deg f. Tù Đ nh lý 2.7 ta có the viet f(X) = n Σ k=0 a X j j 0 2 n
  • 49. 47 Viết đề tài giá sinh viên – ZALO:0973.287.149-TEAMLUANVAN.COM · · D i d i i i−1 i phải cháng minh. với các so nguyên ai nào đó. Xem xét các h so đau trong đȁng thúc này cho thay d! là b®i của n. M¾t khác, neu d! là m®t b®i của n, thì ta có the lay f (X) = X(X + 1) · · · (X + d − 1). Do đó, d là so nguyên nhó nhat sao cho d! là m®t b®i của n. 2.3 V n dnng giải bài toán thi hoc sinh giỏi Ví dn 2.14. [MOSP 2001] Giả sủ f là m®t đa thúc sao cho f(n) ∈ Z với moi n ∈ Z. Chúng minh rang với bat kỳ các so nguyên m, n thì so lcm[1, 2, . . . , deg(f)] f(m) − f(n) m − n là nguyên. Bài giải. Co định các so nguyên phân bi t m và n, giả sả d = m − n và g(X) = f (n + X). Khi đó g có các h so hǎu t , gải các so nguyên đen các so nguyên và deg(f) = deg(g). Do đó ta cháng minh rang lcm[1, 2, . . . , deg(g)] g(d) − g(0) d là nguyên. Giả sả D = deg g, do đó, sả dụng Định lý 2.8, ton tại các so nguyên a0, a1, . . . , aD sao cho: g(X) = Σ i=0 a X . Nó cháng tỏ raơng với bat kỳ 1 ≤ i ≤ D ta có lcm[1, 2, . . . , D] · 1 d ∈ Z. Nhưng ve trái bang lcm[1, 2, . . . , D] d−1 , là m®t so nguyên. Suy ra đieu Ví dn 2.15. [Holden Lee] Giả sủ f là m®t đa thúc b¾c d sao cho f (Z) ⊂ Z và với f (m)− f (n) là m®t b®i của m− n với moi 0 ≤ m, n ≤ d. Chúng minh rang f(m) − f(n) là m®t b®i của m − n với moi so nguyên m, n mà m /= n.
  • 50. 48 Viết đề tài giá sinh viên – ZALO:0973.287.149-TEAMLUANVAN.COM d k d k i k k k d k k k k k k Σ k Bài giải. Do f(Z) ⊂ Z, Định lý 2.7 chỉ ra sự ton tại của các so nguyên a0, a1, . . . , ad sao cho f(X) = Σ k=0 a X . Yêu cau bài toán được suy ra tà đây, Ví dụ 2.14 và ket quả tőng quát dưới đây. Bo đe 2.2. Giả sủ ai ∈ Z và f(X) = Σ k=0 a X và Lk = lcm(1, 2, . . . , k) (với quy ước L0 = 1). Khi đó các phát bieu sau đây là tương đương: (a) m − n chia het f(m) − f(n) với moi 0 ≤ m /= n ≤ d = deg f. (b) Lk chia het ak với moi 0 ≤ k ≤ d. (c) m − n chia het f(m) − f(n) với moi m /= n ∈ Z. Chfíng minh. Giả sả (a) xảy ra. Ta sě cháng minh bang quy nạp toán hoc theo i rang Li chia het ai. Đieu này rõ ràng với i = 0, do đó giả sả rang a0, a1, . . . , ai−1 là các b®i của L0, L1, . . . , Li−1 và co định 0 ≤ j < i. Khi đó j − i chia het f(i) − f(j) = Σ k=0 a i − j = ai + a k 0≤k<i i − j . Tà ví dụ 2.14 và giả thiet quy nạp, moi so trong các so ak i − j với 0 ≤ k < i là m®t b®i của i − j. Suy ra i − j chia het ai và do đó j < i với bat kỳ, suy ra Li chia het ai. Do v y (a) kéo theo (b). Ví dụ 2.14 chỉ ra rang (b) kéo theo (c) và do đó rat hien nhiên rang (c) kéo theo (a). Phép cháng minh được ket thúc. Nh n xét 2.1. Đa thúc f(X) = Σ k=0 a X k
  • 51. 49 Viết đề tài giá sinh viên – ZALO:0973.287.149-TEAMLUANVAN.COM n i ≤ ≤ i i thóa mãn f(a) − f(b) a − b ∈ Z với moi a /= b ∈ Z neu và chí neu lcm(1, 2, . . . , k) | ak với moi k . Bài toán sau đây gợi ý sả dụng Định lý n®i suy Lagrange, nhưng có m®t so khó khăn trong vi c l p lu n, do các đa thác xuat hi n trong công thác Lagrange không có h so nguyên. Bài toán là khá phác tạp và sả dụng lại các đa thác Hilbert. Ví dn 2.16. [Chinese TST 2004] (a) Chúng minh rang với moi so nguyên dương n ton tại m®t đa thúc f ∈ Z[X] sao cho moi so f (1) < f(2) < . . . < f(n) là lũy thùa của 2. (b) Giả sủ a > 1 là m®t so nguyên và n nguyên dương. Chúng minh rang ton tại m®t đa thúc f b¾c n có h so nguyên, sao cho f(0), f(1), . . . , f(n) là các c¾p phân bi t nguyên dương, tat cả có dạng 2ak + 3 với m®t so nguyên k nào đó. Bài giải. (a) Ta sě chon f có dạng f(X) = A. Σ i=0 X Bi với các so nguyên A, B phù hợp, sě được chon sau. Tà công thác nhị thác, với bat kỳ 0 i n ta có f (i) = A(1 + B)i . Ta can f có h so nguyên nhưng X không có các h so nguyên. Tuy nhiên, n! X có h so nguyên với moi 1 ≤ i ≤ n. Do A(1 + B)i không phải là m®t lũy thàa của 2 nên ta không the lay cho B m®t b®i của n!. Ta có the t n dụng sự xuat hi n của A: chon A là 2v2(n!) và B là m®t b®i lẻ của ước lẻ lớn nhat của n!. Khi đó các chú ý trên cháng tỏ rang f có h so nguyên. Cuoi cùng, ta muon 1 + B là m®t lũy thàa của 2. Do đó ta có the chon (chȁng hạn) 1 + B = 2ϕ(d) , trong đó d là ước lẻ lớn nhat của n!. Với nhǎng lựa chon đó, f (1) < f (2) < . . . < f (n) là lũy thàa của 2, theo yêu cau bài toán. (b) Sả dụng ý tưởng như cháng minh (a). Ta viet n! = m · q, với moi nhân tả nguyên to của m là m®t trong so a và trong đó gcd(q, a) = 1.
  • 52. 50 Viết đề tài giá sinh viên – ZALO:0973.287.149-TEAMLUANVAN.COM n Giả sả b = aϕ(q) − 1, do đó q chia het b. Cuoi cùng, định nghĩa: f(X) = 2am Σ i=0 X bi + 3. Nó có h so nguyên bởi vì i! | n! | am · b với moi 0 ≤ i ≤ n. Hơn nǎa, với 1 ≤ k ≤ n ta có P(i) = 2am · (b + 1)i + 3 = 2am+ϕ(q)i + 3. Nh n xét 2.2. Với moi so nguyên dương n có m®t đa thúc f ∈ Z[X] sao cho với moi so f(1) < f(2) < . . . < f(n) là nguyên to. Ta sě tìm m®t đa thác như v y dưới dạng: f(X) = a1(X − 2)(X − 3) · · · (X − n) + a2(X − 1)(X − 3) · · · (X − n)+ + . . . + an(X − 1) · · · (X − n + 1) + 1, với các so nguyên ai thích hợp nào đó. Mà f(i) chỉ phụ thu®c vào ai, do đó ta có the chon ai đe f(i) là nguyên to. Chú ý rang: f(i) = (−1)n−i (n − i)!(i − 1)!ai + 1. bây giờ ta sě chon ai m®t cách quy nạp sao cho f(1) < . . . < f(n) là các so nguyên to. Chúng tôi sě sả dụng Định lý 9.6 trong tài li u tham khảo so [2], theo đó với moi n ton tại vô so so nguyên to p ≡ 1(modn), các so nguyên to này có dạng 1 + kn. Do đó, ton tại m®t so nguyên a1 sao cho 1 + (−1)n−1 (n − 1)!a1 = f (1) là m®t so nguyên to. Co định a1 như v y và chon (m®t lan nǎa bởi ket quả đã trích dan) m®t so nguyên a2 sao cho 1 + a2(n − 2)!(−1)n−2 = f (2) nguyên to lớn hơn f (1). Tiep tục như v y, ta tìm a1, a2, . . . , an sao cho f (1) < f (2) < . . . < f (n) là các so nguyên to và bài toán được giải quyet. i
  • 53. 51 Viết đề tài giá sinh viên – ZALO:0973.287.149-TEAMLUANVAN.COM Ket lu n Trong lu n văn "Định lý Zsigmondy và tính chat so hoc của đa thác" chúng tôi đã trình bày được nhǎng van đe sau: 1. Đa thác và so phác. 2. Đa thác chia đường tròn 3. Định lý Zsigmondy và v n dụng. 4. Tính chat đ c bi t của đa thác thu®c Z[x]. 5. Đa thác Hilber bieu dien Mahler. Lu n văn có the làm tài li u tham khảo cho giáo viên và hoc sinh trong công tác boi dương hoc sinh giỏi
  • 54. 52 Viết đề tài giá sinh viên – ZALO:0973.287.149-TEAMLUANVAN.COM Tài li u tham khảo Tieng Vi t [1] Đàm Văn Nhỉ, Văn Đác Chín, Tran Thị Hong Dung, Lê Xuân Dũng, Đào Ngoc Dũng, Đ ng Xuân Sơn, Tran Trung Tình, Nguyen Anh Tuan (2017), Đa thúc-Chuői lũy thùa và chuyên đe nâng cao, NXB thông tin và truyen thông . Tieng Anh [2] T. Andreescu and G. Dospinescu (2010), Straight from the Book, XYZ Press . [3] Lawrence Sun (February 17, 2013), Cyclotomic Polynomials in Olympiad Number Theory, lala-sun@hotmail.com.